Вы находитесь на странице: 1из 85

TOPNOTCH MEDICAL BOARD PREP SURGERY SUPPLEMENT HANDOUT - Jules Lopez,MD-MBA,Teddy

Carpio,MD-MBA
For inquiries visit www.topnotchboardprep.com.ph or email us at topnotchmedicalboardprep@gmail.com

SURGERY REVIEW Table 1. Cytokines And Their Responses to Injury


Cytokines and their Responses to Injury
This review material is a synthesis of the first few chapters of Among earliest responders after
injury
Schwartzs Principles of Surgery, 9th edition, Absite and Board Tumor Necrosis Factor
Induces muscle breakdown and
Review of Schwartzs Principles of Surgery, 9th edition, 2009 Absite Alpha (TNF-)
cachexia through increased
Clinical Review of Surgery and RUSH integrated review of surgery. catabolism
Integration with other basic subjects as well as review questions Induces fever through
per section are also included to facilitate mastery of the course. prostaglandin activity in anterior
Interleukin 1 (IL-1) hypothalamus
Ready? God bless and enjoy Surgery! Promotes -endorphin release
from pituitary
- Dr. Jules Lopez and Dr. Teddy Carpio Promotes lymphocyte
proliferation, immunoglobulin
P.S. production, gut barrier integrity
Interleukin 2 (IL-2)
Attenuated production after major
Ophthalmology, ENT, Orthopedics, and Gynecology topics are not included blood loss leads to
in this surgery handout. immunocompromise
Elicited by all immunogenic cells
as mediator of acute phase
Any statements, tables, figures marked with a means that the
Interleukin 6 (IL-6) response
information highlighted was previously tested in previous board exams.
Prolongs activated neutrophil
You should master/memorize those. Pay close attention to those in bold,
survival
italicized, underlines as these are very important facts to remember for the
subject. Master topics written in our quick review, subject cross overs and Chemoattractant for neutrophils,
end of review question boxes because they provide high yield information, Interleukin 8 (IL-8) basophils, eosinophils,
not just for surgery but for the rest of the other subjects as well! lymphocytes
Activates macrophages via TH1
cells that demonstrate enhanced
General Outline: Interferon (IFN-) phagocytosis and microbial killing
I. Basic Principles in Surgery Found in wounds 5-7 days after
a. Systemic Response to Injury and Metabolic injury
support
b. Fluid and Electrolyte management of the 2. Heat Shock Proteins
Surgical Patient Group of intracellular proteins that are increasingly
c. Hemostasis, Surgical bleeding and transfusion expressed during times of stress
d. Surgical Infections and Shock Bind both autologous and foreign proteins and thereby
e. Trauma function as intracellular chaperones for ligands such as
f. Burns bacterial DNA and endotoxin
g. Wound healing Protect cells from the deleterious effects of traumatic
II. Organ System Pathologies stress and, when released by damaged cells, alert the
a. Skin and soft tissues immune system of the tissue damage
b. Breast
c. Head & Neck 3. Reactive Oxygen Species (ROS)
d. Esophagus
Small molecules that are highly reactive due to the
e. Stomach
presence of unpaired outer orbit electrons
f. Small Intestine
Cause cellular injury to both host cells and invading
g. Appendix
pathogens through the oxidation of unsaturated fatty
h. Colon, Rectum & Anus
acids within cell membranes
i. Abdominal Wall & Hernia
j. Liver, Portal Venous System & Gallbladder
4. Eicosanoids (Refer to Figure 1 )
Derived primarily by oxidation of membrane
phospholipid,arachidonic acid
PART I: BASIC PRINCIPLES IN SURGERY
Composed of subgroups including prostaglandins,
prostacyclins, hydroxyeicosatetraenoic acid,
SYSTEMIC RESPONSE TO INJURY AND thromboxanes, and leukotrienes
METABOLIC SUPPORT Generate a proinflammatory response

A. Mediators of Inflammation Figure 1. Arachidonic acid metabolism . Cycloxygenase catalyzes the


B. Central Nervous System Regulation of Inflammation formation of PG and TXA2from arachidonic acid. LT = leukotriene;
C. Hormonal Response to Injury PG = prostaglandin; TXA2 = thromboxane A2
D. Surgical Metabolism
E. Nutrition in the Surgical Patient

A. MEDIATORS OF INFLAMMATION
1. Cytokines(Refer to Table 1 )
Protein signaling compounds that are essential for
both innate and adaptive immunity
Mediate cellular responses, including cell migration and
turnover, DNA replication, and immunocyte
proliferation

TOPNOTCH MEDICAL BOARD PREP SURGERY SUPPLEMENT HANDOUT Page 1 of 85


For inquiries visit www.topnotchboardprep.com.ph or email us at topnotchmedicalboardprep@gmail.com
TOPNOTCH MEDICAL BOARD PREP SURGERY SUPPLEMENT HANDOUT - Jules Lopez,MD-MBA,Teddy
Carpio,MD-MBA
For inquiries visit www.topnotchboardprep.com.ph or email us at topnotchmedicalboardprep@gmail.com
Release is increased during times of stress and may be
chronically elevated in certain disease processes (e.g.
burn-injured patients may exhibit elevated levels for
PHARMACOLOGY a 4 weeks)
Wound healing is impaired because it reduces
Aspirin (Acetylsalicylic acid, ASA) is one of the non- transforming growth factor-beta (TGF-B) and insulin-
steroidal anti-inflammatory drug (NSAID) prototypes like growth factor I (IGF-I) in the wound
Mechanism of action ||
Figure 2. Steroid synthesis from cholesterol . ACTH is a principal
o Nonselective AND irreversible inhibitor of
regulator of steroid synthesis. The end products are mineralocorticoids,
the enzyme cyclooxygenase (COX), inhibiting glucocorticoids, and sex steroids.
both cylooxygenase-1 (COX-1) and
cyclooxygenase-2 (COX-2) isoenzymes
o Anti-inflammatory effect is mediated by COX-2
inhibition via decreasing platelet production of
TXA2, a potent stimulator of platelet
aggregation
Side effect ||Gastrointestinal (GI) toxicity due to
inhibition of COX-1 and therebyPG synthesis
Notes ||Uncoupler of oxidative phosphorylation and
is associated with Reye syndrome in children
Ketorolac and Indomethacin are bothnonselective
BUT reversible COX inhibitors
Celecoxib is a selective COX-2 inhibitor with a reduced
risk of GI toxicity

5. Fatty Acid Metabolites BIOCHEMISTRY a


Omega-3 fatty acids have anti-inflammatory effects,
including inhibition of TNF release from hepatic Cholesterol is a steroid alcohol
Kupffer cells, leukocyte adhesion and migration It is a precursor of the following:
6. Kallikrein-Kinin System 1. Cell membranes
Group of proteins that contribute to inflammation, 2. Vitamin D (7-dehydrocholesterol)
BP control, coagulation, and pain responses 3. Bile salts (cholic and chenodeoxycholic acid)
Kallikrein levels are increased during gram negative 4. Adrenal hormones (aldosterone and cortisol)
bacteremia, hypotension, hemorrhage, endotoxemia, 5. Sex hormones (testosterone and estradiol)
and tissue injury It is very hydrophobic (which means, it can cross lipid
Kinis mediate vasodilation, increased capillary predominant barriers), composed of 4 fused
permeability, tissue edema, pain pathway activation, hydrocarbon rings (A-D) and 8-membered branched
inhibition of gluconeogenesis, and increased hydrocarbon chain (20-27) attached to the D-ring
bronchoconstriction It has a single hydroxyl group located at carbon 3 of the
Elevated levels of both has been associated with the A-ring to which a fatty acid can be attached to form
magnitude of injury and mortality cholesterol esters

7. Serotonin
Released at the site of injury, primarily by platelets
Stimulates vasoconstriction, bronchoconstriction, and
platelet aggregation
Ex vivo study showed that serotonin receptor blockade
is associated with decreased production of TNF andIL-1
in endotoxin-treated monocytes

8. Histamine (H4)
Associated with eosinophil and mast cell chemotaxis
Increased release has been documented in hemorrhagic
shock, trauma, thermal injury, endotoxemia, and sepsis

B. CNS REGULATION OF INFLAMMATION 2. Macrophage Migration-Inhibiting Factor


Vagus nerve is highly influential in mediating afferent Neurohormone that is stored and secreted by the
sensory input to the CNS anterior pituitary and by intracellular pools within
o Parasympathetic nervous system transmits macrophages
its efferent signals via acetylcholine A counter regulatory mediator that potentially reverses
o Exerts homeostatic influences such as the anti-inflammatory effects of cortisol
enhancing gut motility, reducing heart rate,
and regulating inflammation 3. Growth Hormones (GH)
o Allows for a rapid response to inflammatory Neurohormone expressed primarily by the pituitary
stimuli and also for the potential regulation of gland that has both metabolic and immunomodulatory
early proinflammatory mediator release, effects
specifically tumor necrosis factor (TNF) Exerts its downstream effects through direct interaction
o Inhibit cytokine activity and reduce injury with GH receptors and secondarily through the
from disease process enhanced hepatic synthesis of IGF-I
GH and IGF-I promote protein synthesis and insulin
C. HORMONAL RESPONSE TO INJURY resistance, and enhances mobilization of fat stores
1. Cortisol(Refer to Figure2 )
Glucocorticoid steroid hormone released by the adrenal 4. Catecholamines
cortex in response to adrenocorticotropic hormone
(ACTH)

TOPNOTCH MEDICAL BOARD PREP SURGERY SUPPLEMENT HANDOUT Page 2 of 85


For inquiries visit www.topnotchboardprep.com.ph or email us at topnotchmedicalboardprep@gmail.com
TOPNOTCH MEDICAL BOARD PREP SURGERY SUPPLEMENT HANDOUT - Jules Lopez,MD-MBA,Teddy
Carpio,MD-MBA
For inquiries visit www.topnotchboardprep.com.ph or email us at topnotchmedicalboardprep@gmail.com
Include epinephrine, norepinephrine, and dopamine, This metabolic pathway involving conversion of lactate
which have metabolic, immunomodulatory, and to glucose; which, in turn, is brought back to the muscle
vasoactive effects for utilization is the Cori cycle
After severe injury, plasma catecholamine levels are Energy expense: 4 ATP molecules
increased threefold to fourfold, with elevations lasting
24 to 48 hours before returning to baseline levels

5. Insulin
Mediates an overall host anabolic state
Insulin resistance and hyperglycemia are hallmarks
of critical illness due to the catabolic effects of
circulating mediators, including catecholamines, cortisol, Lactate production is insufficient to maintain systemic
glucagon, and GH glucose needs during short-term fasting; therefore,
Hyperglycemia during critical illness has significant amounts of protein must be degraded
immunosuppressive effects, and thus is associated daily (75 g/d for a 70 kg adult) to provide the amino
with an increased risk for infection acid substrate for hepatic gluconeogenesis
Insulin therapy (to manage hyperglycemia) decreased Proteolysis during starvation, which results from
mortality and reduced in infectious complications in decreased insulin and increased cortisol release, is
select patient populations associated with elevated urinary nitrogen excretion
from the normal 7-10 g/day up to 30 g or more/day
QUICK REVIEW a
2. Metabolism During Prolonged Fasting
Burn patients may exhibit elevated levels of cortisol for Systemic proteolysis is reduced approximately 20 g/d
4 weeks and urinary nitrogen excretion stabilizes at 2 to 5 g/d
Plasma catecholamine levels are increased 3-4x lasting due to adaptation by vital organs (e.g. myocardium,
for 24 to 48 hours before returning to baseline brain, renal cortex, and skeletal muscle) to using
ketone bodies as their principal fuel source
Ketone bodies become an important fuel source for the
D. SURGICAL METABOLISM brain after 2 days and gradually become the principal
To maintain basal metabolic needs (i.e. at rest and fuel source by 24 days
fasting), a normal healthy adult requires ~22 to 25
kcal/kg/dayfrom carbohydrate, lipid, and protein 3. Metabolism After Injury
sources Injuries or infections induce unique neuroendocrine
Initial hours after surgical or traumatic injury are and immunologic responses that differentiate injury
metabolically associated with a reduced total body metabolism from that of unstressed fasting
energy expenditure and urinary nitrogen wasting Magnitude of metabolic expenditure appears to be
directly proportional to the severity of insult, with
1. Metabolism During Short-term Fasting (<5 days) thermal injuries and severe infections having the
In the healthy adult, principal sources of fuel are highest energy demands (Refer to Figure 4)
derived from muscle protein and lipids, with lipids
Figure 4. Influence of injury severity on resting metabolism
being the most abundant source of energy(40% or
(resting energy expenditure or REE)
more of caloric expenditure)
Hepatic glycogen stores are rapidly and preferentially
depleted fall of serum glucose concentration within
hours (<16 hours)
Hepatic gluconeogenesis is then activated using lactate
from skeletal muscle as the main precursor(Refer to
Figure 3 )

Figure 3. The recycling of peripheral lactate and pyruvate for hepatic


gluconeogenesis is accomplished by the Cori cycle. Alanine within skeletal
muscles can also be used as a precursor for hepatic gluconeogenesis .

Lipids(Triglyceride) become the primary source of


energy (50-80%) during critical illness and stressed
states
o Minimize protein catabolism in the injured
BIOCHEMISTRY a patient
o Lipolysis occurs mainly in response to
Lactate is generated from the skeletal muscle during catecholamine stimulus of the hormone-
anaerobic metabolism sensitive triglyceride lipase
Muscle CANNOT reconvert lactate to glucose Injury and severe infections induce a state of
Lactate MUST first be transported to the liver, where it peripheral glucose intolerance (insulin resistance),
is converted to glucose via hepatic gluconeogenesis despite ample insulin production at levels several-fold
Glucose is then brought back to musclesfor usage above baseline
o Occur in part due to reduced skeletal muscle
pyruvate dehydrogenase activity after injury,

TOPNOTCH MEDICAL BOARD PREP SURGERY SUPPLEMENT HANDOUT Page 3 of 85


For inquiries visit www.topnotchboardprep.com.ph or email us at topnotchmedicalboardprep@gmail.com
TOPNOTCH MEDICAL BOARD PREP SURGERY SUPPLEMENT HANDOUT - Jules Lopez,MD-MBA,Teddy
Carpio,MD-MBA
For inquiries visit www.topnotchboardprep.com.ph or email us at topnotchmedicalboardprep@gmail.com
which diminishes the conversion of pyruvate
to acetyl-CoA and subsequent entry into the Provision of 30 kcal/kg/d will adequately meet energy
TCA cycle requirements in most postsurgical patients, with low
o Increase in plasma glucose levels is risk of overfeeding
proportional to the severity of injury, and this o Overfeedingusually results from
net hepatic gluconeogenic response is under overestimation of caloric needs because actual
the influence of glucagon body weight is used to calculate BEE,
After injury, the initial systemic proteolysis, mediated especially in special patients (e.g. critically ill
primarily by cortisol, increases urinary nitrogen with significant fluid overload and the obese)
excretion to levels in excess of 30 g/d, which roughly o Overfeeding may contribute to clinical
corresponds to a loss in lean body mass of 1.5%/d deterioration via the following: increased O2
(Refer to Figure 5) consumption, increased CO2 production and
prolonged need for ventilatory support,
Figure 5. Effect of injury severity on nitrogen wasting suppression of leukocyte function,
hyperglycemia, and increased risk of infection

1. Enteral Nutrition
Generally preferred over parenteral nutrition due to:
o Lower cost
o Associated risks of the intravenous route
o Beneficial effects of luminal nutrient contact as
it reduces intestinal mucosal atrophy
Initiation should occur immediately after adequate
resuscitation (adequate urine output)
Presence of bowel sounds and the passage of flatus or
stool are NOT absolute prerequisites to start enteral
nutrition, EXCEPT in the setting of gastroparesis,
feedings should be administered distal to the pylorus
Gastric residuals of 200 ml or more in a 4 to 6 hour
period or abdominal distention requires cessation of
feeding and adjustment of infusion rate
QUICK REVIEW a The following are options for enteral feeding access
(Refer to Table 3 ):
Normal energy requirement: 22 to 25 kcal/kg/day
Initial hours after surgical or traumatic injury results to Table 3. Options for Enteral Feeding Access
a reduced total body energy expenditure and Options for Enteral Feeding Access
urinary nitrogen wasting Short-term use
Fat/lipid is the primary source of calories during acute Nasogastric tube Aspiration risks
starvation (<5 days fasting) and after acute injury (NGT) Nasopharyngeal trauma
Frequent dislodgement
Ketone bodies is the primary fuel source in prolonged
Short-term use
starvation
Nasoduodenal / Lower aspiration risks in jejunum
Ketone bodies becomes an important fuel source for Nasojejunal tube Placement challenges (radiographic
brain after 2 days and eventually become the principal assistance often necessary)
fuel source by 24 days Endoscopy skills required
May be used for gastric decompression
Percutaneous
or bolus feeds
Endoscopic
E. NUTRITION IN THE SURGICAL PATIENT Aspiration risks
Gastrostomy
Goals of nutritional support in the surgical patient are Can last 12-24 months
(PEG)
as follows: Slightly higher complication rates with
o To meet the energy requirements for placement and site leaks
Requires general anesthesia and small
metabolic processes, core temperature
laporotomy
maintenance, and tissue repair Surgical
Procedure may allow placement of
o To meet the substrate requirements for Gastrostomy
extended duodenal/jejunal feeding ports
protein synthesis Laparoscopic placement possible
Energy requirement may be measured by indirect Commonly carried out during
calorimetry and trends in serum markers (e.g. laparotomy
prealbumin level) and estimation from urinary nitrogen General anesthesia, laparoscopic
Surgical
excretion, which is proportional to resting energy placement usually requires assistant to
Jejunostomy
expenditure thread catheter
Basal energy expenditure (BEE) may also be Laparoscopy offers direct visualization
of catheter placement
estimated using Harris-Benedict equations, adjusted for
Jejunal placement with regular
the type of surgical stress (Refer to Table 2)
endoscope is operator dependent
o BEE (men) = 66.47 + 13.75 (weight in kg) + 5 Jejunal tube often dislodges retrograde
(height in cm) 6.76 (age in years) kcal/d PEG-jejunal tube Two-stage procedure with PEG
o BEE (women) = 655.1 + 9.56 (weight in kg) + placement, followed by fluoroscopic
1.85 (height in cm) 4.68 (age in years) kcal/d conversion with jejunal feeding tube
The BEE is then multiplied by the type of surgical stress through PEG
(Refer to Table 2 ) that the patient has to determine
the total daily caloric need 2. Parenteral Nutrition
Continuous infusion of hyperosmolar solution
Table 2. Caloric Adjustment Above BEE in Hypermetabolic Conditions containing carbohydrates, proteins, fat, and other
Caloric Adjustments Above Basal Energy Expenditures in necessary nutrients through an indwelling catheter
Hypermetabolic Conditions inserted into the superior vena cava
Normal or Moderate Malnutrition 25-30 kcal/kg/day Principal indications include malnutrition, sepsis, or
Mild Stress 25-30
surgical or traumatic injury in seriously ill patients for
Moderate Stress 30
whom use of the gastrointestinal tract for feedings is
Severe Stress 30-35
Burns 35-40
not possible
TOPNOTCH MEDICAL BOARD PREP SURGERY SUPPLEMENT HANDOUT Page 4 of 85
For inquiries visit www.topnotchboardprep.com.ph or email us at topnotchmedicalboardprep@gmail.com
TOPNOTCH MEDICAL BOARD PREP SURGERY SUPPLEMENT HANDOUT - Jules Lopez,MD-MBA,Teddy
Carpio,MD-MBA
For inquiries visit www.topnotchboardprep.com.ph or email us at topnotchmedicalboardprep@gmail.com
Total (Central) Parenteral Nutrition (TPN)requires Answer: A
access to a large-diameter vein to deliver the nutritional
requirements of the individual Prostacyclin is a member of the eicosanoid family and
o Dextrose content of the solution is high (15- is primarily produced by endothelial cells. It is an
25%) effective vasodilator and also inhibits platelet
o All other macronutrients and micronutrients aggregation. During systemic inflammation,
are deliverable by this route prostacyclin expression is impaired and thus the
Peripheral Parenteral Nutrition (PPN) uses lower endothelium favors a more procoagulant profile.
osmolarity of the solution to allow its administration via
peripheral veins 2. Sepsis increases metabolic needs by approximately
o Reduced levels of dextrose (5-10%) and what percentage?
protein (3%) a. 25%
o Some nutrients cannot be supplemented b. 50%
because they cannot be concentrated into c. 75%
small volumes d. 100%
o Not appropriate for repleting patients with
severe malnutrition Answer: B
o Used for short periods (<2 weeks); beyond
this, TPN should be instituted Sepsis increases metabolic needs to approximately
Complications are as follows (Refer to Table 4 ): 150-160% of resting energy expenditure, or 50%
above normal (Refer to Figure 4). This is mediated in
Table 4. Complication of Parenteral Nutrition part by sympathetic activation and catecholamine
Complications of Parenteral Nutrition release.
Rare occurrences if IV vitamin
preparations are used 3. Which of the following is the initial enteric formula for
Vitamin However, Vitamin K is not part of any the majority of surgical patients?
Deficiencies commercially prepared vitamin a. Low-residue isotonic formula
solution so it should be supplemented
on a weekly basis
b. Elemental formula
Clinically apparent during prolonged
c. Calorie dense formula
parenteral nutrition with fat-free d. High protein formula
solutions
Essential Fatty Acid Manifests as dry, scaly dermatitis Answer: A
(EFA) Deficiency and loss of hair
Prevented by periodic infusion of a fat Most low-residue isotonic formulas provide a caloric
emulsion at a rate equivalent to 10 to density of 1.0 kcal/ml, and approximately 1500 to
15% of total calories 1800 ml are required to meet daily requirements.
Essential trace minerals may be These provide baseline carbohydrates, protein,
required after prolonged TPN
electrolytes, water, fat, and fat-soluble vitamins. These
Zinc deficiency is the most common
that manifests as diffuse eczematoid
solutions usually are considered to be the standard or
Trace Mineral first-line formulas for stable patients with an intact GI
rash at intertriginous areas
Deficiencies tract.
Copper deficiency is associated with
Microcytic anemia
Chromium deficiency is associated with
Glucose intolerance FLUID AND ELECTROLYTE MANAGEMENT
May occur after initiation of parenteral OF THE SURGICAL PATIENT
nutrition
Manifests as glycosuria
If blood glucose levels remain elevated A. Body Fluids and Compartments
or glycosuria persists, dextrose B. Body Fluid Changes
concentration may be decreased, C. Fluid Therapy
Relative Glucose
Intolerance
infusion rate slowed, or regular insulin D. Special Case: Refeeding Syndrome
added to each bottle E. Electrolyte Abnormalities
Rise in blood glucose may be F. Acid-Base Disorders
temporary, as the normal pancreas
increases its output of insulin in
response to the continuous
carbohydrate infusion A. BODY FLUIDS AND COMPARTMENTS
Due to large glucose infusion, a Water constitutes ~50-60% of total body weight
Hypokalemia
significant shift of potassium from Relationship between total body weight and total body
extracellular to intracellular space may water (TBW) is relatively constant for an individual
(and Metabolic
take place and is primarily a reflection of body fat
Acidosis)
Manifests as glycosuria, which is
o Lean tissues (e.g. muscle and solid organs)
treated with potassium, NOT insulin
have higher water content than fat and bone
Lack of intestinal stimulation is
associated with intestinal mucosal o TBW of average young adult male and
atrophy, diminished villous height, female is 60% and 50%, respectively of
Intestinal Atrophy bacterial overgrowth, reduced total body weight
lymphoid tissue size, reduced Estimates of %TBW should be adjusted downward
immunoglobulin A production, and ~10-20% for obese individuals and upward by ~10%
impaired gut immunity for malnourished individuals
Highest percentage of TBW is found in newborns
REVIEW QUESTIONS a (~80%)

1. Prostacyclin has which of the following effects in


QUICK REVIEW a
systemic inflammation?
a. Inhibition of platelet aggregation
TBW is ~50-60% of total body weight
b. Vasoconstriction
c. Increased adhesion molecules TBW (Male): 60%of total body weight
d. Decreased cardiac output TBW (Female): 50%of total body weight

TOPNOTCH MEDICAL BOARD PREP SURGERY SUPPLEMENT HANDOUT Page 5 of 85


For inquiries visit www.topnotchboardprep.com.ph or email us at topnotchmedicalboardprep@gmail.com
TOPNOTCH MEDICAL BOARD PREP SURGERY SUPPLEMENT HANDOUT - Jules Lopez,MD-MBA,Teddy
Carpio,MD-MBA
For inquiries visit www.topnotchboardprep.com.ph or email us at topnotchmedicalboardprep@gmail.com
Young lean males have a higher proportion of TBW than A healthy person consumes water an average of 2L/d,
elderly or obese individuals ~75% from oral intake and the rest extracted from solid
Lower percentage of TBW in females generally foods (Refer to Table 6)
correlates with a higher percentage of adipose tissue Daily water losses include 800-1200 ml in urine, 250 ml
and lower percentage of muscle mass in stool, and 600 ml in insensible losses through both
the skin (75%) and lungs (25%)
Sensible water losses such as sweating or pathologic
TBW is divided into 3 functional fluid compartments loss of GI fluids vary widely, but these include loss of
(Refer to Table 5): electrolytes as well
o Plasma (extracellular) o Sweat is hypotonic and sweating usually
o Interstitial fluid (extracellular) results in only a small sodium loss
o Intracellular fluid o Pathologic GI losses are isotonic to slightly
hypotonic and contribute little to net gain or
loss of free water

Table 5. Functional Body Fluid Compartments


Extracellular fluid PLASMA (1)
Total (1/3 of TBW or (5% of total body weight)
Body 20% of total body INTERSTITIAL FLUID (2)
Water weight) (15% of total body weight) Table 6. Normal Fluid Balance
(TBW) INTRACELLULAR FLUID (3) Water Gain Water Loss
(2/3 of TBW or 40% of total body weight) Urine
Oral fluids 800-1200 ml
1,500 ml Stool
Extracellular fluid compartment (ECF) is balanced Sensible
250 ml
between sodium (Na2+), the principal cation, and Solid fluids Sweat
chloride (Cl-) and bicarbonate (HCO3-), the principal 500 ml 0 ml
anions(Refer to Figure 6) Water of oxidation Skin
o Composition of the plasma and interstitial fluid 250 ml 450 ml
Insensible
differs only slightly in ionic composition Water of solution Lungs
0 ml 150 ml
o Slightly higher protein content (anions) in
plasma results in a higher plasma cation
composition relative to the interstitial fluid
1. Extracellular Volume Deficit
Intracellular fluid compartment (ICF) is comprised
Most common fluid disorder in surgical patients
ofcations, potassium (K+) and magnesium (Mg2+), and
theanions, phosphate(HPO4-) and proteins Can either be acute or chronic (Refer to Table 7)
o Acute volume deficit is associated with
Concentration gradient between compartments is
cardiovascular and central nervous system
maintained by adenosine triphosphate (ATP) driven
signs
sodium-potassium pumps located with the cell
o Chronic deficit displays tissue signs such as
membranes
decrease in skin turgor and sunken eyes, in
Figure 6. Chemical composition of body fluid compartments
addition to acute signs

Table 7. Signs and Symptoms of Volume Disturbances


System Volume Deficit Volume Excess
Weight loss Weight gain
General
Decreased skin turgor Peripheral edema
Tachycardia Increased cardiac output
Orthostasis / Increased central venous
Cardio Hypotension pressure
Collapsed neck veins Distended neck veins
Murmur
Oliguria --
Renal
Azotemia
GI Ileus Bowel edema
Pulmo -- Pulmonary edema

Most common cause of volume deficit in surgical


patients is a loss of GI fluids from nasogastric suction,
vomiting, diarrhea, or enterocutanous fistula
(Refer to Table 8 )
Third-space or nonfunctional ECF losses that occur with
sequestration secondary to soft tissue
injuries/infections, burns, and intraabdominal
processes such as peritonitis, obstruction, or prolonged
surgery can also lead to massive volume deficits

Water is freely diffusible and distributed evenly Table 8. Composition of GI Secretions


throughout all fluid compartments of the body Volume Volume Na+ K+ Cl- HCO3
Sodiumis confined to ECF and is associated with water (ml/24h) (ml/24h) (mEq/L) (mEq/L) (mEq/L) (mEq/L)

o Sodium-containing fluids are distributed Saliva 1000 10 26 10 30


throughout the ECF and add to bothplasma 1000- 60-90 10 130 0
Stomach
2000
(intravascular) and interstitial spaces
1500 120- 5-10 90-120 0
o Sodium-containing fluids expand the Duodenum
140
interstitial space by ~3x as much as the plasma Ileum 3000 140 5 104 30
Colon 750 60 30 40 0
B. BODY FLUID CHANGES 600-800 135- 5-10 70-90 115
Pancreas
145
300-800 135- 5-10 90-110 30-40
Bile
145
TOPNOTCH MEDICAL BOARD PREP SURGERY SUPPLEMENT HANDOUT Page 6 of 85
For inquiries visit www.topnotchboardprep.com.ph or email us at topnotchmedicalboardprep@gmail.com
TOPNOTCH MEDICAL BOARD PREP SURGERY SUPPLEMENT HANDOUT - Jules Lopez,MD-MBA,Teddy
Carpio,MD-MBA
For inquiries visit www.topnotchboardprep.com.ph or email us at topnotchmedicalboardprep@gmail.com
However, there also have been concerns of
2. Extracellular Volume Excess increased bleeding, because hypertonic saline
May be iatrogenic or secondary to renal dysfunction, is an arteriolar vasodilator
congestive heart failure, or cirrhosis
Both plasma and interstitial volumes are increased
1. Preoperative Fluid Therapy
Symptoms are primarily pulmonary and cardiovascular
(Refer to Table 7) Preoperative evaluation of a patients volume status and
pre-existing electrolyte abnormalities is an important
In healthy patients, edema and hyperdynamic
part of overall preoperative care
circulation are common and well tolerated
Administration of maintenance fluidsis required in an
However, the elderly and patients with cardiac disease
otherwise healthy individual on NPO before surgery
may quickly develop congestive heart failure and
pulmonary edema in response to only a moderate The following is the formula used for calculating
volume excess maintenance fluids in the absence of pre-existing
abnormalities (Refer to Table 11):
C. FLUID THERAPY
Table 11. Maintenance Fluid Computation
Most commonly used solutions are as follows:
First 0-10 kg Give 100 ml/kg/dor 4 ml/kg/hr
(Refer to Table 9)
Next 10-20 kg Give additional 50 ml/kg/dor 2 ml/kg/hr
Weight >20 kg Give additional 20 ml/kg/dor 1 ml/kg/hr

However, may surgical patients have volume and/or


electrolyte abnormalities associated with their surgical
disease
Table 9. Electrolyte Solutions for Parenteral Administration o Acute volume deficits should be corrected as
Solution Na+ Cl- K+ Ca2+ Other mOsm much as possible
ECF 142 103 4 27 -- 280 o Once a volume deficit is diagnosed, prompt
Lactated
Lactate
fluid replacement should be instituted, usually
Ringers 130 109 4 28 280 with an isotonic crystalloid
28 mEq/l
(LR) o Patients whose volume deficit is not corrected
0.9% after initial volume challenge and those with
Sodium
154 154 0 0 -- 308 impaired renal function and the elderly should
chloride
(PNSS) be considered for more intensive monitoring
Dextrose of central venous pressure or cardiac output in
D5 Lactated an ICU setting
50 g/l
Ringers 130 109 4 3 560
Lactate o If symptomatic electrolyte abnormalities
(D5LR)
28 mEq/l accompany volume deficit, the abnormality
D5 Sodium should be corrected to the point that the acute
Dextrose
chloride 154 154 0 0 588 symptom is relieved before surgical
50 g/l
(D5NS) intervention.
D5 0.45%
Dextrose
Sodium 77 77 0 0 434
50 g/l QUICK REVIEW a
chloride
D5 0.25%
Dextrose
Sodium 34 34 0 0
50 g/l
357 Extracellular volume deficit is the most common fluid
chloride disorder in surgical patients
Most common cause of volume deficit in surgical
patients is a loss of GI fluids
Type of fluid administered depends on the patients Both PLR and PNSS are considered isotonic and are
volume status and the type of concentration or useful in replacing GI losses and correcting extracellular
composition abnormality present (Refer to Table 10) volume deficits
Hypertonic saline solution is used as a treatment
Table 10. Fluid Therapy
modality in patients with closed head injuries
Solution Description
Considered isotonic BUT it is slightly
Lactated
hypotonic due to lactate
Ringers 2. Intraoperative Fluid Therapy
Useful in replacing GI losses and correcting
(PLR) With the induction of anesthesia, compensatory
extracellular volume deficits
Considered isotonic BUT it is mildly mechanisms are lost, and hypotension will develop if
hypertonic volume deficits are not appropriately managed
0.9% Sodium Also useful in replacing GI losses and To avoid hemodynamic instability intraoperatively, the
chloride correcting extracellular volume deficits, following should be ensured:
(PNSS) especially those associated with o Known fluid losses corrected
hyponatremia, hypochloremia, and metabolic
alkalosis
preoperatively
Useful for replacement of ongoing GI losses
o Adequate maintenance fluid therapy
as well as for maintenance fluid therapy in provided
D5 0.45% o Ongoing losses replaced intraoperatively
the postoperative period
Sodium
Provides sufficient free water for insensible Among the ongoing losses during surgery include
chloride
losses and enough sodium to aid the kidneys distributional shifts via third space or nonfunctional ECF
in adjustment of serum sodium levels losses seen in the following:
D5 3.5-5% Hypertonic saline solution o Major open abdominal surgeries in the form of
Sodium Used for correction of severe sodium bowel wall edema, peritoneal fluid, and the
chloride deficits wound edema during surgery
Hypertonic saline solution
o Large soft tissue wounds, complex fractures
Used as a treatment modality in patients
D5 7% with associated soft tissue injury, and burns
with closed head injuries
Sodium Replacement of ECF losses during surgery often
Shown to increase cerebral perfusion and
chloride requires 500 to 1000 ml/hr of a balanced salt solution
decrease intracranial pressure, thus
decreasing brain edema to support homeostasis

TOPNOTCH MEDICAL BOARD PREP SURGERY SUPPLEMENT HANDOUT Page 7 of 85


For inquiries visit www.topnotchboardprep.com.ph or email us at topnotchmedicalboardprep@gmail.com
TOPNOTCH MEDICAL BOARD PREP SURGERY SUPPLEMENT HANDOUT - Jules Lopez,MD-MBA,Teddy
Carpio,MD-MBA
For inquiries visit www.topnotchboardprep.com.ph or email us at topnotchmedicalboardprep@gmail.com
Addition of albumin or other colloid-containing
solutions to intraoperative fluid therapy is NOT Treatment ||Management of water deficit
necessary In hypovolemic patients, volume should be restored
with normal saline before concentration abnormality is
3. Postoperative Fluid Therapy addressed
Should be based on the patients current estimated Once adequate volume is achieved, water deficit is
volume status and projected ongoing fluid losses replaced using a hypotonic fluid
Any deficits from either preoperative or intraoperative Rate of fluid administration should be titrated to
losses should be corrected and ongoing requirements achieve a decrease in serum sodium concentration
should be included along with maintenance fluids of no more than 1 mEq/l/h
In the initial postoperative period, an isotonic solution Overly rapid correction can lead to cerebral edema
should be administered and herniation
o Adequacy of resuscitation should be based on
vital signs and urine output 2. Hyponatremia
o All measured losses, including losses through Occurs when there is an excess of extracellular water
vomiting, NGT, drains, and urine output as relative to sodium
well as insensible losses should be replaced Extracellular volume can be high, normal, or low (Refer
After the initial 24 to 48 hours, fluids can be changed to to Figure 8)
5% dextrose to 0.45% saline in patients unable to In most cases, sodium concentration is decreased as a
tolerate enteral nutrition consequence of either sodium depletion or dilution
If normal renal function and adequate urine output are Symptomatic hyponatremia does not occur until serum
present, potassium may be added to the IV fluids sodium level is 20 mEq/l
Clinical manifestations || Primarily central nervous
D. SPECIAL CASE: REFEEDING SYNDROME system in origin (headache, confusion, seizures, coma)
Refeeding syndrome potentially lethal condition that associated increases in intracranial pressure
can occur with rapid and excessive feeding of
patients with severe underlying malnutrition due to
starvation, alcoholism, delayed nutritional support,
anorexia nervosa, or massive weight loss in obese
patients
Shift in metabolism from fat to carbohydrate substrate Figure 8. Etiology of Hyponatremia
stimulates insulin release, which results in the cellular
uptake of electrolytes, particularly phosphate,
magnesium, potassium, and calcium
Severe hyperglycemia may result from blunted basal
insulin secretion
To prevent its development, the following measures
should be done:
o Underlying electrolyte and volume deficits
should be corrected
o Thiamine should be administered before the
initiation of feeding
o Caloric repletion should be instituted slowly,
at 20 kcal/kg per day, and should gradually
increase over the first week

E. ELECTROLYTE ABNORMALITIES Treatment ||Water restriction and, if severe, the


1. Hypernatremia administration of sodium
Results from either a loss of free water or a gain of If symptomatic, 3% normal saline should be used to
sodium in excess of water increase the sodium by no more than 1 mEq/l/huntil
Associated with either an increased, normal, or the serum sodium reaches 130 mEq/l or symptoms are
decreased extracellular volume (Refer to Figure 7) improved
Symptoms are rare until serum sodium concentration If asymptomatic, correction should increase the sodium
exceeds 160 mEq/l level by no more than 0.5 mEq/l/hr to a maximum
Clinical manifestations || Mostly central nervous increase of 12 mEq/l/d
system in nature (restlessness, irritability, seizures,
coma) due tohyperosmolarity
INTERNAL MEDICINE a
May lead to subarachnoid hemorrhage and death

Figure 7. Etiology of Hypernatremia Central Pontine Myelinosisis a consequence of rapid


correction of hyponatremia
Characterized with seizures, weakness, paresis, akinetic
movements, and unresponsiveness
May result in permanent brain damage and death
MRI may assist in the diagnosis

3. Hyperkalemia
Serum K+ concentration above the normal range of
3.5-5 mEq/l
Caused by excessive K+ intake, increased release of K+
from cells, or impaired K+ excretion by the kidneys
(Refer to Table 12)
Clinical manifestations || Mostly GI (nausea/vomiting,
diarrhea), neuromuscular (weakness, paralysis), and
cardiovascular (arrhythmia, arrest)

TOPNOTCH MEDICAL BOARD PREP SURGERY SUPPLEMENT HANDOUT Page 8 of 85


For inquiries visit www.topnotchboardprep.com.ph or email us at topnotchmedicalboardprep@gmail.com
TOPNOTCH MEDICAL BOARD PREP SURGERY SUPPLEMENT HANDOUT - Jules Lopez,MD-MBA,Teddy
Carpio,MD-MBA
For inquiries visit www.topnotchboardprep.com.ph or email us at topnotchmedicalboardprep@gmail.com
ECG changes ||High peaked T waves (early),
widened QRS complex, flattened P wave, prolonged PR K+sparing Diuretics are competitive antagonists that
interval (first-degree block), sine wave formation and either block the actions of aldosterone at the distal
ventricular fibrillation convoluted tubule, or directly inhibit sodium channels
Treatment ||Reducing total body K+, shifting K+ from o Aldosterone antagonists: Spironolactoneand
extracellular to intracellular space, and protecting cells Eplerenone
from the effects of increased K+ o Epithelial sodium channel blockers: Amiloride
Exogenous sources of potassium should be removed, and Triamterene
including K+ supplementation in IV fluids Non K+-sparing Diureticsinclude loop diuretics and
K+ can be removed from the body using a cation- thiazides, which both inhibit Na+ and Cl- reabsorption
exchange resin such as Kayexalate that binds K+ in o Loop diuretics (Furosemide) inhibit the Na+-
exchange for Na+ K+-2Cl- cotransporter in the thick ascending
Immediate measures also should include attempts to limb of the loop of Henle
shift K+ intracellularly with glucose, insulin and o Thiazidesinhibit the Na+-Cl-transporter in the
bicarbonate infusion and nebulized salbutamol(10- distal tubule
20 mg)
When ECG changes are present, calcium chloride or
calcium gluconate (5-10 ml of 10% solution) should be
administered immediately
All measures are temporary, lasting from 1 to 4 hours
Dialysis should be considered in severe hyperkalemia
when conservative measures fail

4. Hypokalemia
More common than hyperkalemia in the surgical patient
Caused by inadequate K+ intake, excessive renal K+
excretion, K+ loss in pathologic GI secretions, or
intracellular shifts from metabolic alkalosis or insulin
therapy (Refer to Table 12)
Clinical manifestations || Primarily related to failure of
normal contractility of GI smooth muscle (ileus,
constipation), skeletal muscle (decreased reflexes,
weakness, paralysis), and cardiac muscle (arrest)
ECG changes || U waves, T-wave flattening, ST- QUICK REVIEW a
segment changes, and arrhythmias (with digitalis
therapy) Normal Na+: 135-145 mEq/l
Symptomatichypernatremia are rare until serum
Table 12. Etiology of Potassium Abnormalities
Etiology of Potassium Abnormalities
sodium exceeds 160 mEq/l
Increased Intake Symptomatic hyponatremia does not occur until serum
Potassium supplementation sodium level is 20 mEq/l
Blood transfusions
Endogenous load/destruction: hemodialysis, Normal K+: 3.5-5 mEq/l
rhabdomyolysis, crush injury, GI hemorrhage Peaked T waves are the first ECG change seen in most
Increased Release patients with hyperkalemia
Hyperkalemia
Acidosis T-wave flattening is seen in hypokalemia
Rapid rise of extracellular osmolality
Hypokalemia causes decreased deep tendon reflexes
(hyperglycemia or mannitol)
Impaired Excretion
while hypomagnesemia and hypocalcemia causes
Potassium-sparing diuretics increased deep tendon reflexes
Renal insufficiency/failure
Inadequate Intake
Dietary, potassium-free IV fluids 5. Hypercalcemia
Potassium-deficient TPN Serum calcium level above the normal range of 8.5-
Excessive Potassium Excretion 10.5 mEq/l or an increase in ionized calcium above
Hyperaldosteronism 4.2-4.8 mg/dl
Hypokalemia
Medications (Non-K+ sparing diuretics) Caused by primary hyperparathyroidism in the
GI losses outpatient setting and malignancy in hospitalized
Direct loss of potassium from GI fluid
patients
(diarrhea)
Renal loss of potassium Clinical manifestations || Neurologic impairment,
musculoskeletal weakness and pain, renal dysfunction,
Treatment || Potassium repletion, the rate is and GI symptoms (Refer to Table 13)
determined by the symptoms ECG changes || Shortened QT interval, prolonged PR
Mild, asymptomatic hypokalemia: oral repletion is and QRS intervals, increased QRS voltage, T-wave
adequate (KCl 40 mEq per enteral access x 1 dose) flattening and widening, and atrioventricular block
Asymptomatic hypokalemia, not tolerating enteral Treatment is required when hypercalcemia is
nutrition: KCl 20 mEq IV q2h x 2 doses symptomatic, which usually occurs when the serum
If IV repletion is required, usually no more than 10 level exceeds 12 mEq/l
mEq/h is advisable in an unmonitored setting Critical level for serum calcium is 15 mEq/l, when
K+ supplementation can be increased to 40 mEq/h symptoms noted earlier may rapidly progress to death
when accompanied by continuous ECG monitoring, and Treatment || Aimed at repleting the associated volume
even more in the case of imminent cardiac arrest from a deficit and then inducing a brisk diuresis with normal
malignant arrhythmia associated hypokalemia saline
Caution should be done when oliguria or impaired renal
6. Hypocalcemia
function is coexistent
Serum calcium level below 8.5 mEq/l or a decrease in
the ionized calcium level below 4.2 mg/dl
PHARMACOLOGY a
TOPNOTCH MEDICAL BOARD PREP SURGERY SUPPLEMENT HANDOUT Page 9 of 85
For inquiries visit www.topnotchboardprep.com.ph or email us at topnotchmedicalboardprep@gmail.com
TOPNOTCH MEDICAL BOARD PREP SURGERY SUPPLEMENT HANDOUT - Jules Lopez,MD-MBA,Teddy
Carpio,MD-MBA
For inquiries visit www.topnotchboardprep.com.ph or email us at topnotchmedicalboardprep@gmail.com
Causes include pancreatitis, malignancies associated Treatment || Eliminate exogenous sources of
with increased osteoclastic activity (breast and prostate magnesium, correct concurrent volume deficits and
cancer), massive soft tissue infections such as correct acidosis if present
necrotizing fasciitis, renal failure, pancreatic and small To manage acute symptoms, calcium chloride(5-10ml)
bowel fistulas, hypoparathyroidism, toxic shock should be administered to immediately antagonize the
syndrome, and tumor lysis syndrome cardiovascular effects
Transient hypocalcemia also occurs after removal of a If persistently elevated or with symptoms, dialysis may
parathyroid adenoma due to atrophy of the remaining be necessary
gland and avid bone remineralization
Neuromuscular and cardiac symptoms do not occur 8. Hypomagnesemia
until the ionized fraction falls below 2.5 mg/dl Magnesium depletion is a common problem in
Clinical manifestations || Neuromuscular symptoms hospitalized patients, particularly in the critically ill
with decreased cardiac contractility (Refer to Table 13) Result from alterations of intake, renal excretion and
ECG changes || Prolonged QT interval, T-wave pathologic losses
inversion, heart block and ventricular fibrillation o Poor intake may occur in cases of starvation,
alcoholism, prolonged IV fluid therapy, and TPN
MICROBIOLOGY a with inadequate supplementation of
Magnesium
Toxic Shock Syndromeis due to the Staphylococcus o Losses are seen in cases of increased renal
aureus toxin, Toxic shock syndrome toxin (TSST-1) excretion from alcohol abuse, diuretic use,
Clinical manifestations ||Fever, hypotension, sloughing administration of amphotericin B, and primary
of the filiform papillae (strawberry tongue), aldosteronism, as well as GI losses from
desquamating rash, andmulti-organ involvement diarrhea, malabsorption, and acute pancreatitis
Usually no site of pyogenic inflammation blood CS Clinical manifestations || Neuromuscular and central
negative nervous system hyperactivity, similar to those of
calcium deficiency
Common in tampon-using menstruating women or in
patients with nasal packing for epistaxis ECG changes || Prolonged QT and PR intervals, ST-
segment depression, flattening or inversion of P waves,
Treatment || Remove the offending agent and to start
torsades de pointes, and arrhythmias
antibiotics (Clindamycin and Vancomycin)
Can produce hypocalcemia and lead to persistent
hypokalemia

Table 13. Clinical Manifestations of Abnormalities in Ca2+ and Mg+


Increased Serum Levels
System Calcium Magnesium
PATHOLOGY a Anorexia Nausea/vomiting
Gastrointestinal Nausea/vomiting
Tumor Lysis Syndromeconsists of multiple electrolyte Abdominal pain
abnormalities that may be seen after initiation of Weakness Weakness
Bone pain Lethargy
cancer treatment Neuromuscular
Confusion Decreased reflexes
Chemotherapy causes release of break-down products Coma
of dying cancer cells Hypertension Hypotension Arrest
Among the electrolyte abnormalities include Arrhythmia
Cardiovascular
hyperkalemia, hyperphosphatemia, hyperuricemia, Worsening of digitalis
and hypocalcemia toxicity
Clinical consequences are acute uric acid nephropathy Renal Polyuria -
and acute renal failure Decreased Serum Levels
System Calcium Magnesium
Hyperactive reflexes Hyperactive reflexes
Paresthesias Muscle tremors
Treatment || Calcium supplementation and correction Muscle cramps Tetany
of other metabolic derangements Carpopedal spasm Positive Chvosteks
Neuromuscular
Asymptomatic hypocalcemia can be treated with oral Seizures and Trousseaus
or IV calcium Tetany signs
Acute symptomatic hypocalcemia should be treated Trousseaus sign1 Delirium and
with IV 10% calcium gluconate to achieve a serum Chvosteks sign2 seizures (severe)
Cardiovascular Heart failure Arrhythmia
concentration of 7-9 mg/dl 1Spasm resulting from pressure applied to the nerves and vessels of the
Associated deficits in magnesium, potassium, and pH
upper extremity with a blood pressure cuff
must also be corrected 2 Spasm resulting from tapping over the facial nerve
Hypocalcemia will be refractory to treatment if
coexisting hypomagnesemia is not corrected first
Treatment || Magnesium supplementation
7. Hypermagnesemia Correction of magnesium depletion can be oral if
asymptomatic and mild or IV if symptomatic and severe
Rare but can be seen with severe renal insufficiency and
parallel changes in potassium excretion For those with severe deficits (<1 mEq/L) or those who
o Magnesium-containing antacids and laxatives are symptomatic, 1 to 2 g of magnesium sulfate may
can produce toxic levels in patients with renal be administered IV over 15 minutes or 2 minutes if
insufficiency/failure under ECG monitoring to correct torsades
o Excess intake in conjunction with TPN, or To counteract the adverse side effects of a rapidly rising
rarely massive trauma, thermal injury, and magnesium level and correct hypocalcemia (frequently
severe acidosis, may be associated with associated with hypomagnesemia), simultaneous
symptomatic hypermagnesemia administration of calcium gluconateis done
Clinical manifestations || Mainly GI with
neuromuscular dysfunction and impaired cardiac QUICK REVIEW a
conduction (Refer to Table 13)
ECG changes || (similar to hyperkalemia) Increased PR Normal Ca2+: 8.5-10.5 mEq/l
interval, widened QRS complex, elevated T waves Normal ionized Ca2+: 4.2-4.8 mg/dl

TOPNOTCH MEDICAL BOARD PREP SURGERY SUPPLEMENT HANDOUT Page 10 of 85


For inquiries visit www.topnotchboardprep.com.ph or email us at topnotchmedicalboardprep@gmail.com
TOPNOTCH MEDICAL BOARD PREP SURGERY SUPPLEMENT HANDOUT - Jules Lopez,MD-MBA,Teddy
Carpio,MD-MBA
For inquiries visit www.topnotchboardprep.com.ph or email us at topnotchmedicalboardprep@gmail.com
Treatment is required when hypercalcemia is hydrogen ions and allow the hydrogen ions to buffer
symptomatic, when the serum level exceeds 12 mEq/l excess HCO3
Symptomatic hypocalcemia do not occur until the Treatment || Includes replacement of the volume
ionized fraction falls below 2.5 mg/dl deficit with isotonic saline and then potassium
Hypocalcemia will be refractory to treatment if replacement once adequate urine output is achieved
coexisting hypomagnesemia is not corrected first
Table 15. Etiology of Metabolic Alkalosis
F. ACID-BASE DISORDERS Increased bicarbonate generation
Chloride losing (urinary chloride > 20 mEq/l)
1. Metabolic Acidosis
Mineralocorticoid excess
Results from an increased intake of acids, an increased Profound potassium depletion
generation of acids, or an increased loss of bicarbonate Chloride sparing (urinary chloride < 20 mEq/l)
Body compensates by producing buffers (extracellular Loss from gastric secretions (emesis or nasogastric
bicarbonate and intracellular buffers from bone and suction)
muscle), increasing ventilation (Kussmaul's Diuretics
respirations),increasing renal reabsorption and Excess administration of alkali
generation of bicarbonate, and increasing renal secretion Acetate in parenteral nutrition
of hydrogen Citrate in blood transfusions
Evaluation of a patient with metabolic acidosis includes Antacids
determination of the anion gap (AG), an index of Bicarbonate
Milk-alkali syndrome
unmeasured anions
Impaired bicarbonate excretion
o AG = Na+ (Cl- + HCO3-)
Decreased glomerular filtration
o Normal: <12 mmol/l Increased bicarbonate reabsorption (hypercarbia or potassium
Etiology of metabolic acidosis is listed inTable 14 depletion)

Table 14. Etiology of Metabolic Acidosis


3. Respiratory Acidosis
High Anion Gap Metabolic Acidosis (HAGMA)
Associated with retention of CO2 secondary to
decreased alveolar ventilation
Exogenous acid ingestion Mnemonic: MUDPILES
Ethylene glycol
Principal causes are listed in Table 16
Methanol Because compensation is primarily a renal mechanism,
Salicylate Uremia (Renal failure)
Methanol it is a delayed response
Diabetic ketoacidosis
Propylene glycol In the chronic form, partial pressure of arterial CO2
Paraldehyde remains elevated and the bicarbonate concentration
Endogenous acid production Infection, Iron, Isoniazid rises slowly as renal compensation occurs
Ketoacidosis Lactic acidosis Treatment || Directed at the underlying cause
Lactic acidosis Ethylene glycol Measures to ensure adequate ventilation through
Renal insufficiency Salicylates bilevel positive airway pressure or endotracheal
intubationare also initiated
Norma Anion Gap Metabolic Acidosis (NAGMA)
Acid administration (HCl)
Mnemonic: HARD UP Table 16. Etiology of Respiratory Acidosis
Loss of bicarbonate Hyperalimentation Etiology of Respiratory Acidosis
Acetazolamide (Carbonic Narcotics
GI losses (diarrhea, fistulas) anhydrase inhibitor) Central nervous system injury
Renal tubular acidosis Pulmonary (secretions, atelectasis, mucus plug, pneumonia, pleural
Ureterosigmoidoscopy Diarrhea effusion)
Ureteroenteric fistula Pain from abdominal or thoracic injuries or incisions
Renal tubular acidosis Pancreticoduodenal Limited diaphragmatic excursion from intra-abdominal
fistula pathology (abdominal distention, abdominal compartment
syndrome, ascites)
Carbonic anhydrase inhibitor

Lactic acidosis is a common cause of severe metabolic 4. Respiratory Alkalosis


acidosis in surgical patients In the surgical patient, most cases are acute and
In circulatory shock, lactate is produced in the presence secondary to alveolar hyperventilation
of hypoxia from inadequate tissue perfusion Causes include pain, anxiety, neurologic disorders
Treatment || Restore perfusion with volume (central nervous system injury and assisted ventilation),
resuscitation rather than to attempt to correct with drugs (salicylates), fever, gram-negative bacteremia,
exogenous bicarbonate thyrotoxicosis, and hypoxemia
With adequate perfusion, lactate is rapidly metabolized Acute hypocapnia can cause an uptake of potassium and
by the liver and the pH level returns to normal phosphate into cells and increased binding of calcium to
Administration of bicarbonate for the treatment of albumin, leading to symptomatic hypokalemia,
metabolic acidosis is controversial hypophosphatemia, and hypocalcemia with
o Overzealous administration of bicarbonate can subsequent arrhythmias, paresthesias, muscle cramps,
lead to metabolic alkalosisand can be and seizures
associated with arrhythmias Treatment || Directed at the underlying cause
o An additional disadvantage is that sodium Direct treatment of the hyperventilation using
bicarbonate actually can exacerbate controlled ventilation may also be required
intracellular acidosis
QUICK REVIEW a
2. Metabolic Alkalosis
Results from the loss of fixed acids orgain of Evaluation of a patient with metabolic acidosis includes
bicarbonate(Refer to Table 15) determination of the anion gap (AG) to differentiate
Majority of patients will have hypokalemia, because HAGMA from NAGMA (TIP: Memorize the mnemonics!)
extracellular potassium ions exchange with intracellular Normal AG is <12 mmol/l

TOPNOTCH MEDICAL BOARD PREP SURGERY SUPPLEMENT HANDOUT Page 11 of 85


For inquiries visit www.topnotchboardprep.com.ph or email us at topnotchmedicalboardprep@gmail.com
TOPNOTCH MEDICAL BOARD PREP SURGERY SUPPLEMENT HANDOUT - Jules Lopez,MD-MBA,Teddy
Carpio,MD-MBA
For inquiries visit www.topnotchboardprep.com.ph or email us at topnotchmedicalboardprep@gmail.com
Treatment of metabolic acidosis is to restore perfusion D. Special Cases
with volume resuscitation rather than exogenous E. Transfusion
bicarbonate
Metabolic alkalosis is associated with hypokalemia
A. HEMOSTASIS
Function is to limit blood loss from an injured vessel
Four major physiologic events participate in the
REVIEW QUESTIONS a hemostatic process(Refer to Figure 9):
o Vascular constriction
1. A patient develops a high output fistula following o Platelet plug formation
abdominal surgery. The fluid is sent for evaluation o Fibrin formation
with the following results: Na+ 135, K+ 5, Cl- 70. Which o Fibrinolysis
of the following is the most likely source of the fistula? Figure 9. Biology of Hemostasis
a. Stomach
b. Small bowel
c. Pancreas
d. Biliary tract

Answer: C
The composition of pancreatic secretions is marked
by high level of bicarbonate (Refer to Table 8),
compared to other GI secretions. In this example, the
patient has a total of 140 mEq of cation (Na+ + K+) and
only 70 mEq of anion (Cl-). The remaining 70 mEq (to
balance the 140 mEq of cation) must be bicarbonate.

2. A postoperative patient with a potassium of 2.9 is


given 1 mEq/kg replacement with KCl (potassium
chloride). Repeat tests after the replacement show the
serum K to be 3.0. The most likely diagnosis is:
a. Hypomagnesemia
b. Hypocalcemia
c. Metabolic acidosis
d. Metabolic alkalosis

Answer: A 1. Vascular Constriction


In cases in which potassium deficiency is due to Initial response to vessel injury
magnesium depletion, potassium repletion is Dependent on local contraction of smooth muscle
difficult unless hypomagnesemia is first corrected. o Thromboxane A2 (TXA2) ,potent constrictor
Alkalosis will change serum potassium (a decrease in of smooth muscle,is produced locally at the
0.3 mEq/l for every 0.1 increase in pH above normal). site of injury
This is not enough to explain the lack of response to o Endothelin ,also a potent vasoconstrictor, is
repletion in the patient. Metabolic acidosis would not synthesized by injured endothelium and
decrease potassium. Calcium does not play a role in serotonin
potassium metabolism. o Bradykinin and fibrinopeptidesare capable
of contracting vascular smooth muscle.
3. Which of the following is a cause of acute Extent of vasoconstriction varies with the degree of
hypophosphatemia? vessel injury (more pronounced in vessels with medial
a. Chronic ingestion of magnesium containing smooth muscles)
laxatives
b. Insulin coma 2. Platelet Plug Formation
c. Refeeding syndrome Platelets do not normally adhere to each other or to the
d. Rhabdomyolosis vessel wall but during vascular disruption, they form a
hemostatic plugthat aids in cessation of bleeding
Answer: C Injury to the intimal layer in the vascular wall exposes
Acute hypophosphatemia is usually caused by an von Willebrand's factor (vWF), a subendothelial
intracellular shift of phosphate in association with protein, where platelets adhere via glycoprotein I/IX/V
respiratory alkalosis, insulin therapy, refeeding After adhesion, platelets initiate a release reaction that
syndrome, and hungry bone syndrome. Clinical recruits other platelets to seal the disrupted vessel
manifestations include cardiac dysfunction or muscle The aforementioned process, mediated by adenosine
weakness but are usually absent until levels fall diphosphate (ADP) and serotonin,is reversible and is
significantly. Refer to page 8 for a discussion on known as primary hemostasis
refeeding syndrome. In the second wave of platelet aggregation,
Magnesium containing laxatives can cause anotherrelease reaction occurs that results in
hypermagnesemia in patients with renal failure but compaction of the platelets viaglycoprotein IIb/IIIa into
does not affect phosphorous. Patients with insulin a plug
coma (hypoglycemia) are not at risk for With fibrinogen as a cofactor, this process, mediated by
hypophosphatemia. Rhabdomyolosis is associated ADP, Ca2+, serotonin, TXA2, is irreversible
with hyperkalemia and hyperphosphatemia
3. Fibrin Formation / Coagulation
As a consequence of the release reaction, alterations
HEMOSTASIS, SURGICAL BLEEDING, occur in the phospholipids of the platelet membrane
AND TRANSFUSION that initiates coagulation
Coagulation cascade typically has been depicted as two
A. Hemostasis intersecting pathways
B. Evaluation of Hemostatic Risk o Intrinsic pathway begins with factor XII and
C. Surgical Bleeding through a series of enzymatic reactions, which
TOPNOTCH MEDICAL BOARD PREP SURGERY SUPPLEMENT HANDOUT Page 12 of 85
For inquiries visit www.topnotchboardprep.com.ph or email us at topnotchmedicalboardprep@gmail.com
TOPNOTCH MEDICAL BOARD PREP SURGERY SUPPLEMENT HANDOUT - Jules Lopez,MD-MBA,Teddy
Carpio,MD-MBA
For inquiries visit www.topnotchboardprep.com.ph or email us at topnotchmedicalboardprep@gmail.com
is intrinsic to the circulating plasma and no Vitamin B12 or Folate deficiency
surface is required to initiate the process Chemotherapy or radiation therapy
o Extrinsic pathwayrequires exposure of Acute alcohol intoxication
tissue factor on the surface of the injured Viral infections
vessel wall to initiate the arm of the cascade Immune-mediated disorders (Idiopathic
thrombocytopenia, Heparin-induced
beginning with factor VII
thrombocytopenia, Autoimmune disorders or B-
o The two arms of the coagulation cascade
Decreased cell maligancies, Secondary thrombocytopenia)
merge to a common pathway at factor X, and Survival Disseminated intravascular coagulation
activation of factors II (prothrombin) and I Disorders related to platelet thrombi
(fibrinogen)proceeds in sequence (Thrombocytopenic purpura, Hemolytic uremic
Secondary hemostasis or fibrin clot formation syndrome)
occurs after conversion of fibrinogen to fibrin Portal hypertension
Sarcoid
Sequestration
4. Fibrinolysis Lymphoma
During the wound-healing process, the fibrin clot Gauchers disease
Platelet Abnormalities: Qualitative
undergoes fibrinolysis, which permits restoration of
Massive transfusion
blood flow
Therapeutic administration of platelet inhibitors
This is initiated at the same time as the clotting Myeloproliferative disorders
mechanism under the influence of circulating kinases, Disease states Monoclonal gammopathies
tissue activators, and kallikrein, which are present in Liver disease
the vascular endothelium
Plasmindegrades the fibrin mesh at various places,
which leads to the production of circulating fragments PATHOLOGY a
that are cleared by proteases or by the kidney and liver
Bernard-Soulier Syndromeis caused by a defect in the
B. EVALUATION OF HEMOSTATIC RISK glycoprotein Ib/IX/V receptor for vWF, leading to
1. Preoperative Evaluation of Hemostasis defective platelet adhesion
Most important component of the bleeding risk o Decreased platelet count
assessment is a directed bleeding history o Treatment || Platelet transfusion
When history is unreliable or incomplete or when Glanzmann Thrombastheniais caused by a defect in
abnormal bleeding is suggested, a formal evaluation of the platelet glycoprotein IIb/IIIa complex , leading to
hemostasis should be performed before surgery defective platelet aggregation
o Hemoglobin levels below 7 or 8 g/dl appear to o Normal platelet count
be associated with significantly more o Treatment || Platelet transfusion
perioperative complications
o Determination of the need for preoperative
transfusion must consider factors other than
the absolute hemoglobin level, including the
presence of cardiopulmonary disease, type of
surgery, and likelihood of surgical blood loss
Laboratory tests of hemostatic parameters in patients
with low risk of bleeding are NOT required

2. Evaluation of Intraoperative or Postoperative Bleeding


Excessive bleeding during or after a surgical procedure
may be the result of ineffective hemostasis, blood
transfusion, undetected hemostatic defect, disseminated
intravascular coagulation (DIC) or consumptive
coagulopathy, and/or fibrinolysis Treatment || Depends on the extent and cause of
platelet reduction
C. SURGICAL BLEEDING o Platelets are given preoperatively to rapidly
1. Systemic Bleeding Disorders increase the count in surgical patients
Thrombocytopenia, secondary to any platelet o A count of >50,000/L generally requires no
pathology, is the most common abnormality of specific therapy
hemostasis that results in bleeding in surgical patients o One unit of platelet concentrate is expected
Systemic causes of surgical bleeding can either be to increase the circulating platelet count by
inherited or acquired (Refer to Table 17) ~10,000/L in the average 70-kg person
Inherited platelet functional defects include o In patients whose thrombocytopenia is
abnormalities of platelet surface proteins, abnormalities refractory to standard platelet transfusion,
of platelet granules, and enzyme defects the use of human leukocyte antigen (HLA)
Acquired abnormalities of platelets may be quantitative compatible plateletshas proved effective
or qualitative, although some patients have both types
o Quantitative defects may be a result of failure 2. Local Hemostasis
of production due to bone marrow disorders, Significant surgical bleeding usually is caused by
shortened survival, or sequestration ineffective local hemostasis
o Qualitative defects include massive Goal is to prevent further blood loss from a disrupted
transfusionand drugs that interfere with vessel that has been incised or transected
platelet function Hemostasis may be accomplished by interrupting the
flow of blood to the involved area or by direct closure of
Table 17. Etiology of Surgical Bleeding the blood vessel wall defect
Congenital Factor Deficiencies
o Mechanical procedure: When pressure is
Coagulation Factor Deficiencies
applied (whether through direct digital
von Willibrands Disease
Platelet Functional Defects
pressure, hemostatic clamp, or tourniquet) to
Acquired Hemostatic Defects an artery proximal to an area of bleeding,
Platelet Abnormalities: Quantitative profuse bleeding may be reduced so that more
Failure of Leukemia definitive action is permitted
Production Myeloproliferative disorders
TOPNOTCH MEDICAL BOARD PREP SURGERY SUPPLEMENT HANDOUT Page 13 of 85
For inquiries visit www.topnotchboardprep.com.ph or email us at topnotchmedicalboardprep@gmail.com
TOPNOTCH MEDICAL BOARD PREP SURGERY SUPPLEMENT HANDOUT - Jules Lopez,MD-MBA,Teddy
Carpio,MD-MBA
For inquiries visit www.topnotchboardprep.com.ph or email us at topnotchmedicalboardprep@gmail.com
o Thermal agents: Heat (via cautery or Mnemonic: Inhibitors Stop Cyber Kids
harmonic scalpel) achieves hemostasis by from Eating GRApefruit Q
denaturation of protein that results in Isoniazid
coagulation of large areas of tissue Cytochrome P450 Sulfonamides
o Topical hemostatic agents: Include physical inhibitors
Cimetidine
or mechanical, caustic, biologic, and reduce clearance
Ketoconazole
physiologic agents that works either by and increase the
Erythromycin
inducing protein coagulation and precipitation anticoagulant
effect of warfarin Grapefruit juice
or activating biologic responses to bleeding Ritonavir
Amiodarone
D. SPECIAL CASES
Quinidine
1. Disseminated Intravascular Coagulation (DIC)
An acquired syndrome characterized by intravascular
activation of coagulation Bleeding complications can be manifested
Can originate from and cause damage to the throughhematuria, soft tissue bleeding, intracerebral
microvasculature, which if sufficiently severe, can bleeding, skin necrosis, and abdominal bleeding
produce organ dysfunction Bleeding into the abdominal cavity is the most
Excessive thrombin generation leads to microthrombus common complication of warfarin therapy
formation, followed by consumption and depletion of Intramural bowel hematoma is the most common
coagulation factors and platelets, which leads to the cause of abdominal pain in patients receiving
classic picture of diffuse bleeding anticoagulation therapy
Causes include the following: Certain surgical procedures should not be performed
o Central nervous system injuries with such as procedures involving the central nervous
embolization of brain matter system or the eye
o Fractures with embolization of bone marrow
o Malignancy Table 18. Reversal of anticoagulation for patients undergoing surgery
o Organ injury (severe pancreatitis, liver failure) Reversal of Heparin Therapy
Not indicated when aPTT is <1.3 times the control value
o Certain vascular abnormalities (aneurysms)
Emergency Discontinue drug and use ofprotamine sulfate
o Others: snakebites, illicit drugs, transfusion
surgery for more rapid reversal of anticoagulation
reactions, transplant rejection, and sepsis Reversal of Warfarin Therapy
Diagnosis is made on the basis of an inciting cause with Not indicated when the INR is <1.5
associated thrombocytopenia, prolonged PT, low Discontinue drug several days before the
fibrinogen level, and elevated levels of fibrin operation with monitoring of prothrombin
markers (fibrin degradation products, D-dimer, soluble concentration (>50% is safe)
fibrin monomers) Parenteral administration of vitamin K is
Elective
Treatment || Relieving the patient's causative primary indicated in patients with biliary obstruction or
surgery
medical or surgical problem and maintaining adequate malabsorption who may be vitamin K deficient
Low molecular weight heparin should be
perfusion
administered while the INR is decreasing in
If there is active bleeding, hemostatic factors should be patients with high risk of thrombosis
replaced using fresh frozen plasma (FFP), which Emergency Rapid reversal of anticoagulation can be
generally is sufficient to correct the hypofibrinogenemia surgery accomplished with FFP

2.Anticoagulation and Bleeding


Spontaneous bleeding can be a complication of Other drugs that interfere with platelet function are
anticoagulant therapy with either heparin, warfarin,or aspirin, clopidogrel, dipyridamole, and glycoprotein
low molecular weight heparin IIb/IIIa inhibitors
o Risk of spontaneous bleeding with heparinis o Aspirin inhibit platelet function through
relatively high but reduced with continuous irreversible acetylation of platelet
infusion technique prostaglandin synthase
o Therapeutic anticoagulation is more reliably o Clopidogrelinhibit platelet function through
achieved withlow molecular weight selective irreversible inhibition of ADP-
heparinbecause laboratory testing is not induced platelet aggregation
routinely done, which makes them attractive General recommendation is that a period of ~7 days is
options for outpatient anticoagulation required from the time the drug is stopped until an
o Warfarin is used for long-term outpatient elective procedure can be performed
anticoagulation in various clinical conditions Timing of urgent and emergent surgeries is unclear
including deep vein thrombosis, valvular heart Preoperative platelet transfusions may be beneficial
disease (with or without prosthetic valves),
atrial fibrillation, and recurrent myocardial 3.Coagulopathy of Liver Disease
infarction Liver plays a key role in hemostasis because it
synthesizes manycoagulation factors
PHARMACOLOGY a Most common coagulation abnormalities associated
with liver dysfunction are thrombocytopenia and
Warfarin inhibits vitamin K epoxide reductase and impaired humoral coagulation function manifested
thereby interferes with production of functional vitamin as prolonged PT and increase in the International
K-dependent clotting and anticlotting factors Normalized Ratio (INR)
Side effect||bleeding, warfarin-induced skin necrosis Thrombocytopenia is related to hypersplenism, reduced
production of thrombopoietin, and immune-mediated
Mnemonic: Ethel Booba takes Phen- destruction of platelets
phen and Refuses Greasy Carb Shakes Before any therapy for thrombocytopenia is initiated,
Cytochrome P450 Ethanol the actual need for correction should be strongly
Inducers considered
Barbiturates
increase clearance Treatment || Platelet transfusions; however, the
Phenytoin
and reduce the effect typically lasts only several hours
Rifampicin
anticoagulant
Griseofulvin Potential alternative strategy is administration of
effect of warfarin
Carbamazepine interleukin-11, a cytokine that stimulates proliferation
St. Johns Wort / Smoking
TOPNOTCH MEDICAL BOARD PREP SURGERY SUPPLEMENT HANDOUT Page 14 of 85
For inquiries visit www.topnotchboardprep.com.ph or email us at topnotchmedicalboardprep@gmail.com
TOPNOTCH MEDICAL BOARD PREP SURGERY SUPPLEMENT HANDOUT - Jules Lopez,MD-MBA,Teddy
Carpio,MD-MBA
For inquiries visit www.topnotchboardprep.com.ph or email us at topnotchmedicalboardprep@gmail.com
of hematopoietic stem cells and megakaryocyte Most common indication for blood transfusion in
progenitors surgical patients is volume replacement
Less well accepted option is splenectomy or splenic
embolization to reduce hypersplenism but reduced
splenic blood flow can reduce portal vein flow with Complications of transfusionis primarily related to
subsequent development of portal vein thrombosis. blood-induced proinflammatory responses
Complications (discussed below) occur in
4. Coagulopathy of Trauma approximately 10% of all transfusions, but <0.5% are
Recognized causes of traumatic coagulopathy include serious
acidosis, hypothermia, and dilution of coagulation factors
Significant proportion of trauma patients arrive at the 1. Febrile Non-hemolytic Reactions
ER coagulopathic, and this early coagulopathy is Defined as an increase in temperature [>1C (1.8F)]
associated with increased mortality associated with a transfusion
Shockhas been postulated to induce coagulopathy Approximately 1% of all transfusions
through systemic activation of anticoagulant and Preformed cytokines in donated blood and recipient
fibrinolytic pathways antibodies reacting with donated antibodies are
Hypoperfusion causes activation of thrombomodulin(on postulated causes
the surface of endothelial cells), which complexes with Can be reduced by the use of leukocyte-reduced blood
circulating thrombin thereby inducing not only an products with
anticoagulant statebut also enhancing fibrinolysis Pretreatment with paracetamol reduces the severity of
the reaction
5. Massive Transfusion Rare but potentially lethal febrile reaction is secondary
Well-known cause of thrombocytopenia due to to bacterial contamination of infused blood
hypothermia, dilutional coagulopathy, platelet o Gram-negative organisms, especially
dysfunction, fibrinolysis, or hypofibrinogenemia Yersinia enterocolitica and Pseudomonas
Impaired ADP-stimulated aggregation occurs with species are the most common cause
massive transfusion(>10 units of packed RBC) leading to o Most cases are associated with the
surgical bleeding administration of platelets
o Pathogenesis is related to lability of factor V,
D. TRANSFUSION which appears necessary for this interaction
General indications for transfusion is listed in Table 19 o Results in sepsis and death in 25% of patients
o Clinical manifestations ||fever and chills,
Table 19. Indications for Replacement of Blood and its Elements tachycardia, and hypotension, GI symptoms
General Indications for Transfusion (abdominal cramps, vomiting, and diarrhea),
Improvement in Oxygen-carrying capacity is primarily a and hemorrhagic manifestations such as
Oxygen function of RBC
hemoglobinemia, hemoglobinuria, and DIC
Carrying Therefore, transfusion of RBC should
Capacity augment oxygen-carrying capacity
o If suspected, transfusion should be
Critically ill patients frequently receive discontinued and the blood cultured
Treatment of o Treatment ||Administration of oxygen,
transfusions at a hemoglobin level
Anemia adrenergic blocking agents, and antibiotics
approaching 9 g/dl
Most common indication for blood
transfusion in surgical patients is 2. Allergic Reaction
thereplenishment of the blood volume Occurs in ~1% of all transfusions
Measurements of hemoglobin levels or Reactions usually are mild
hematocrit are frequently used to assess
Clinical manifestations ||rash, urticaria, and fever
blood loss misleading in acute loss,
because levels can be normal in spite of within 60 to 90 minutes of the start of the transfusion
severely contracted blood volume In rare instances, anaphylactic shock develops
Volume Caused by transfusion of antibodies from
Estimated total blood volume is 7-8% of
Replacement
TBW hypersensitive donors or the transfusion of antigens to
Blood loss of up to 20% of total blood which the recipient is hypersensitive
volume: Replaced with crystalloid Can occur after the administration of any blood product
solution
Treatment ||Administration of antihistamines or in
Blood loss >20% of total blood
volume:Addition of packed RBC, and in
more serious cases, use of epinephrine or steroids may
the case of massive transfusion, the be indicated
addition of FFP
3. Respiratory Complications
Circulatory overload can occur with rapid infusion of
QUICK REVIEW a blood, plasma expanders, and crystalloids, particularly
in older patients with underlying heart disease
Thrombocytopenia is the most common abnormality Clinical manifestations ||dyspnea, rales,and cough
of hemostasis Treatment || Initiate diuresis, slow the rate of blood
Significant surgical bleeding usually is caused by administration, and minimize delivery of fluids while
ineffective local hemostasis blood products are being transfused
Most important management of DIC is treatment of the Another significant respiratory complication is
underlying cause Transfusion-related Acute Lung Injury (TRALI)
Bleeding into the abdominal cavity is the most o Defined as noncardiogenic pulmonary edema
common complication of warfarin therapy related to transfusion
Intramural bowel hematoma is the most common o Can occur with the administration of any
cause of abdominal pain in patients receiving plasma-containing blood product
anticoagulation therapy o Clinical manifestations ||similar to those of
A period of ~7 days is required from the time aspirin circulatory overload and often accompanied
and/or clopidogrel is stopped until an elective procedure by fever, rigors, and bilateral pulmonary
can be performed infiltrates on chest radiograph
o Most commonly occurs within 1 to 2 hours
Most common coagulation abnormalities associated
after the onset of transfusion, but virtually
with liver dysfunction are thrombocytopenia and
always before 6 hours
impaired humoral coagulation function
TOPNOTCH MEDICAL BOARD PREP SURGERY SUPPLEMENT HANDOUT Page 15 of 85
For inquiries visit www.topnotchboardprep.com.ph or email us at topnotchmedicalboardprep@gmail.com
TOPNOTCH MEDICAL BOARD PREP SURGERY SUPPLEMENT HANDOUT - Jules Lopez,MD-MBA,Teddy
Carpio,MD-MBA
For inquiries visit www.topnotchboardprep.com.ph or email us at topnotchmedicalboardprep@gmail.com
o Etiology is not well established, but TRALI is
thought to be related to anti-HLA or anti- REVIEW QUESTIONS a
human neutrophil antigen antibodies in
transfused blood that primes neutrophils in 1. What percentage of platelets can be sequestered in
the pulmonary circulation. the spleen?
o Treatment || Discontinuation of any a. 15%
transfusion, notification of the transfusion b. 30%
service, and provision of pulmonary support c. 45%
(from supplemental oxygen to mechanical d. 60%
ventilation)
Answer: B
4. Hemolytic Reactions Platelets are anucleate fragments of megakaryoctes.
Can be classified as either acute ordelayed(Refer to The normal circulating number of platelets ranges
Table 20) between 150,000 and 400,000/L. Up to 30% of
circulating platelets may be sequestered in the
Table 20. Classification of Hemolytic Reactions spleen. If not consumed in a clotting reaction,
Classification of Hemolytic Reactions platelets are normally removed by the spleen and have
Occur with the administration of ABO- an average life span of 7 to 10 days.
incompatible blood
Fatal in up to 6% of cases 2. A patient on chronic warfarin therapy presents with
Contributing factors include technical or
acute appendicitis. INR is 1.4. Which of the following
clerical errors in the laboratory
andadministration of wrong blood type is the most appropriate management?
Characterized by intravascular hemolysis a. Proceed immediately with surgery without
and consequent hemoglobinemia and stopping the warfarin
hemoglobinuria b. Stop the warfarin, give FFP, and proceed
Clinical manifestations ||pain at the site of with surgery
Acute transfusion, facial flushing, and back and chest c. Stop the warfarin and proceed with surgery
Hemolytic pain, associated with fever, respiratory in 8-12 hours
Reaction distress, hypotension, and tachycardia d. Stop the warfarin and proceed with surgery
In anesthetized patients, diffuse bleeding and
in 24-36 hours
hypotension are the hallmarks
Positive Coombs' test is diagnostic
Treatment || stop transfusion, get a sample of Answer: A
the recipient's blood and send along with the When the INR <1.5 in a patient taking warfarin,
suspect unit to the blood bank for comparison reversal of anticoagulation therapy may not be
with the pretransfusion samples necessary. (Refer to Table 18). However, meticulous
Urine output should be monitored and surgical technique is mandatory, and the patient must
adequate hydration maintained to prevent be observed closely throughout the postoperative
precipitation of hemoglobin within the tubules period.
Reactions occur 2 to 10 days after
transfusion
3. What percent of the population is Rh negative?
Occur when an individual has a low antibody
titer at the time of transfusion
a. 5%
Characterized by extravascular hemolysis, b. 15%
Delayed c. 25%
mild anemia, indirect hyperbilirubinemia,
Hemolytic d. 35%
decreased haptoglobin levels, low-grade
Reaction
hemoglobinemia and hemoglobinuria
Clinical manifestations ||fever and jaundice Answer: B
Coombs' test usually yields a positive result Rh negative recipients should receive transfusion only
Treatment || Do not usually require specific of Rh negative blood. However, this groups represents
intervention only 15% of the population. Therefore, the
administration of Rh positive blood is acceptable if Rh
negative blood is not available. However, Rh positive
5. Transmission of Disease blood should not be transfused to Rh negative females
Among the diseases that have been transmitted by who are of childbearing age.
transfusion are malaria, Chagas' disease, brucellosis,
and, very rarely, syphilis
Transmission of hepatitis C virus and HIV-1has been
dramatically minimized by the introduction of better
antibody and nucleic acid screening for these pathogens
SURGICAL INFECTIONS AND SHOCK
Hepatitis B virus transmission may still occur in about 1
in 100,000 transfusions in nonimmune recipients
A. Definitions
B. Surgical Wounds Classification
QUICK REVIEW a C. Prevention and Treatment of Surgical Infections
D. Infections of Significance in Surgical Patients
Gram-negative organismsare the most common cause E. Shock
of bacterial contamination of infused blood, especially
with platelet administration
A. DEFINITIONS
Transfusion-related Acute Lung Injury (TRALI) most
1. Infection (Refer to Figure 10 )
commonly occurs within 1 to 2 hours after the onset of
Identifiable source of microbial insult
transfusion, but virtually always before 6 hours
2. Systemic Inflammatory Response Syndrome (SIRS)
Two or more of the following criteria met:
Acute hemolytic reaction is characterized by o Temperature 38C or 36C
intravascular hemolysis while delayed reaction is o Heart rate 90 beats per minute
characterized by extravascular hemolysis o Respiratory rate 20 breaths per minute or
PaCO2 32 mmHg or mechanical ventilation
o White blood cell count 12,000/uL or
4,000/uL or 10% band forms
TOPNOTCH MEDICAL BOARD PREP SURGERY SUPPLEMENT HANDOUT Page 16 of 85
For inquiries visit www.topnotchboardprep.com.ph or email us at topnotchmedicalboardprep@gmail.com
TOPNOTCH MEDICAL BOARD PREP SURGERY SUPPLEMENT HANDOUT - Jules Lopez,MD-MBA,Teddy
Carpio,MD-MBA
For inquiries visit www.topnotchboardprep.com.ph or email us at topnotchmedicalboardprep@gmail.com
3. Sepsis mechanical, chemical, and antimicrobial modalities, or a
SIRS + Identifiable source of infection combination of these methods
4. Severe Sepsis These modalities are NOT capable of sterilizing the
Sepsis + Organ dysfunction hands of the surgeon or the skin or epithelial surfaces of
5. Septic Shock the patient BUT the inoculum can be reduced
Sepsis + Cardiovascular collapse (needs vasopressors) considerably
Thus, entry through the skin, into the soft tissue, and
Figure 10. Relationship between infection and SIRS. Sepsis is the into a body cavity or hollow viscus invariably is
presence both of infection and SIRS, shown here as the intersection of STILLassociated with the introduction of some
these two areas. Other conditions may cause SIRS as well (trauma, degree of microbial contamination
aspiration, etc.). Severe sepsis (and septic shock) are both subsets of
Therefore, antimicrobial agents should be given in
sepsis.
patients who undergo procedures that may be associated
with the ingress of significant numbers of microbes(e.g.,
colonic resection) or in whom the consequences of any
type of infection due to said process would be dire (e.g.,
prosthetic vascular graft infection)

1. Appropriate Use of Antimicrobial Agents


Prophylaxis is the administration of an antimicrobial
agent(s)before and during the operative procedure
to reduce the number of microbes that enter the tissue
or body cavity
o Only a single dose of antibiotic is required,
and only for certain types of surgical
procedures (Refer to Table 21)
B. SURGICAL WOUNDS CLASSIFICATION o Patients who undergo complex, prolonged
*Based on the magnitude of bacterial load at the time of surgery procedures in which the duration of the
**IR = Infection rate operation exceeds the serum drug half-life
1. Clean (Class I) should receive an additional dose(s)
Include those in which no infection is present o Administration of postoperative doses
Only skin microflora potentially contaminate the wound DOES NOT provide additional benefit, and
No hollow viscus (that contains microbes) is entered should be discouraged, as it is costly and is
Class ID wounds are similar except that a prosthetic associated with increased rates of microbial
device (e.g. mesh or valve) is inserted drug resistance
Example: Hernia repair, Breast biopsy(IR: 1-5.4%)
Table 21. Prophylactic therapy
Site Antibiotic Alternative
2. Clean/Contaminated (Class II) Cardiovascular Cefazolin or Cefuroxime Vancomycin
Include those in which a hollow viscus such as Cefazolin, Cefotetan
respiratory, GI, or GU tracts with inherent bacterial flora Gastroduodenal Cefoxitin Fluoroquinolone
is opened under controlled circumstances without Ampicillin-sulbactam
significant spillage of contents Biliary tract
Ampicillin-sulbactam Fluoroquinolone
Example: Cholecystectomy, Elective GI surgery(not colon) with active
Ticarcillin-clavulanate + Clindamycin or
(IR: 2.1-9.5%), Colorectal surgery(IR: 9.4-25%) infection
Piperacillin-tazobactam Metronidazole
(cholecystitis)
Cefazolin+Metronidazole Gentamicin or
3. Contaminated (Class III) Colorectal ,
Ertapenem Fluoroquinolone
Include open accidental wounds encountered early Obstructed small
Ticarcillin-clavulanate plus Clindamycin
after injury, those with extensive introduction of bowel
Piperacillin-tazobactam or Metronidazole
bacteria into a normally sterile area of the body due Aminoglycoside +
Head and neck Cefazolin
to major breaks in sterile technique (e.g. open cardiac Clindamycin
massage), gross spillage of viscus contents such as Neurosurgery Cefazolin Vancomycin
from the intestine, or incision through inflamed albeit Cefazolin
Orthopedics Vancomycin
nonpurulent tissue Ceftriaxone
Example: Penetrating abdominal trauma, large tissue Breast, Hernia Cefazolin Vancomycin
injury, enterotomy (IR: 3.4-13.2%)
Empiric therapycomprises the use of an antimicrobial
agent(s)when the risk of a surgical infection is high,
4. Dirty (Class IV) based on the underlying disease process (e.g. ruptured
appendicitis), or when significant contamination
Include traumatic wounds in which a significant
during surgery has occurred (e.g. inadequate bowel
delay in treatment has occurred and in which
preparation or considerable spillage of colon contents)
necrotic tissue is present, those created in the
o Prophylaxis merges into empiric therapy in
presence of overt infection as evidenced by the
situations in which the risk of infection
presence of purulent material, and those created to
increases markedly because of intraoperative
access a perforated viscus accompanied by a high
findings
degree of contamination
o Limited to a short course of drug (3-5 days),
Example: Perforated diverticulitis, necrotizing soft tissue
and should be curtailed based on
infections (IR: 3.1-12.8%)
microbiologic data (i.e. culture and sensitivity
pattern) coupled with improvements in the
C. PREVENTION AND TREATMENT OF SURGICAL INFECTIONS
clinical course of the patient
Resident microflora of the skin and other barrier
o Manner in which therapy is used differs
surfaces represent a potential source of microbes that
depending on whether the infection is
can invade the body during trauma, thermal injury, or
monomicrobial or polymicrobial
elective or emergent surgical intervention
Maneuvers to diminish the presence of exogenous Table 22. General principles in empiric therapy
(surgeon and operating room environment) and Empiric Therapy
endogenous (patient) microbes consist of the use of

TOPNOTCH MEDICAL BOARD PREP SURGERY SUPPLEMENT HANDOUT Page 17 of 85


For inquiries visit www.topnotchboardprep.com.ph or email us at topnotchmedicalboardprep@gmail.com
TOPNOTCH MEDICAL BOARD PREP SURGERY SUPPLEMENT HANDOUT - Jules Lopez,MD-MBA,Teddy
Carpio,MD-MBA
For inquiries visit www.topnotchboardprep.com.ph or email us at topnotchmedicalboardprep@gmail.com
Frequently are nosocomial infections Malnutrition
occurring in postoperative patients, such Peripheral vascular disease
asUTIs, pneumonia, or bacteremia Anemia
Therapy should be initiated in patients with Radiation
evidence of SIRS, coupled with evidence of Chronic skin disease
local infection (e.g., an infiltrate on chest X- Carrier state (e.g. chronic staphylococcus carriage)
ray plus a positive Gram's stain in BAL Recent operation
sample) Local Factors
Within 24 to 72 hours, culture and Poor skin penetration
Monomicrobial sensitivity reports will allow directed Contamination of instruments
antibiotic regimen Inadequate antibiotic prophylaxis
Empiric regimen for common infections are Prolonged procedure
as follows:
Local tissue necrosis
o UTI: 3-5 days
Hypoxia, hypothermia
o Pneumonia: 7-10 days
Microbial Factors
o Bacteremia: 7-14 days
o Osteomyelitis, endocarditis, or Prolonged hospitalization (leading to nosocomial organisms)
prosthetic infections: 6- Toxin secretion
12 weeks Resistance to clearance (e.g. capsule formation)
Primary therapeutic modality is source
control(discussed below) but antimicrobial Surgical site infections are classified into
agents play an important role as well incisionaland organ/space infections
Culture results are of lesser importance Incisional infections are further subclassified into
in managing these infections, as it has been superficial (limited to skin and subcutaneous tissue)
demonstrated that only a limited group of
Polymicrobial and deep incisional categories
microbes predominate in the established
infection, selected from a large number o Treatment ||Effective therapy for incisional
present at the time of initial contamination SSIs consists of incision and drainage
As such, antibiotic regimen should NOT without the addition of antibiotics
be modified solely on the basis of culture o Antibiotic therapy is reserved for patients in
information whom evidence of significant cellulitis is
present, or who manifest concurrent SIRS
o Open wound often is allowed to heal by
2. Source Control secondary intention, with dressings being
Primary precept of surgical infectious disease therapy changed twice a day
consists of drainage of all purulent material, o Use of topical antibiotics and antiseptics to
debridement of all infected, devitalized tissue, and debris, further wound healing remains unproven
and/or removal of foreign bodies at the site of infection, o Vacuum-assisted closure is increasingly used
plus remediation of the underlying cause of infection in management of problem wounds and can be
o Discrete, walled-off purulent fluid collection applied to complex wounds in difficult
(abscess) requires drainage via percutaneous locations
drain insertion or an incision and drainage Treatment of organ/space infections is discussed in
o Ongoing source of contamination (e.g. bowel Intra-Abdominal Infections section
perforation) or presence of an aggressive,
rapidly-spreading infection (e.g. necrotizing 2.Intra-Abdominal Infections/Peritonitis
soft tissue infection) requires aggressive Microbial contamination of the peritoneal cavity
operative intervention, both to remove Classified according to etiology (Refer to Table 24)
contaminated material and infected tissue (e.g.
radical debridement or amputation) and to
remove the initial cause of infection (e.g. Table 24. Intra-abdominal infections
bowel resection) Intra-abdominal Infections
Occurs when microbes invade the normally
D. INFECTIONS OF SIGNIFICANCE IN SURGICAL PATIENTS sterile peritoneal cavity via
1. Surgical Site Infection (SSI) hematogenous dissemination from a
Infections of the tissues, organs, or spaces exposed distant source of infection or direct
inoculation
by surgeons during surgery
More common among patients with ascites,
Development of SSI is related to three factors and in those individuals who are undergoing
(Refer to Table 23): peritoneal dialysis
o Patient factors Often monomicrobial and rarely require
o Local factors Primary surgical intervention
o Microbial factors Microbial Diagnosis is established based on physical
Treatment || Prophylactic antibiotics reduce the Peritonitis examination that reveals diffuse tenderness
incidence of SSI during certain types of procedures and guardingwithout localized findings,
absence of pneumoperitoneum, presence of
o Single dose of an antimicrobial agent
>100 WBCs/ml, and microbes with a single
should be administered immediately before morphology on Grams stain on fluid obtained
commencing surgery for class ID, II, III, and via paracentesis
IV types of wounds Treatment || Antibiotic therapy for 14 to 21
Surgical management of the wound is also a critical days and removal of indwelling devices (e.g.,
determinant of the propensity to develop an SSI peritoneal dialysis catheter or
o Class I and II woundsmay be closed primarily peritoneovenous shunt)
o Class III and IV wounds areallowed to heal by Occurs due to contamination of the
secondary intention where superficial aspects peritoneal cavity due to perforation or
severe inflammation and infection of an
of these wounds should be packed open only
intra-abdominal organ
Examples: Appendicitis, perforation of any
Table 23. Risk factors for development of surgical site infections Secondary
portion of the GI tract, or diverticulitis
Patient Factors Microbial
Treatment ||Effective therapy requires
Older age Peritonitis
source control to resect or repair the
Immunosuppression diseased organ, debridement of necrotic,
Obesity infected tissue and debris, and
Diabetes Mellitus administration of antimicrobial agents
Chronic inflammatory process directed against aerobes and anaerobe
TOPNOTCH MEDICAL BOARD PREP SURGERY SUPPLEMENT HANDOUT Page 18 of 85
For inquiries visit www.topnotchboardprep.com.ph or email us at topnotchmedicalboardprep@gmail.com
TOPNOTCH MEDICAL BOARD PREP SURGERY SUPPLEMENT HANDOUT - Jules Lopez,MD-MBA,Teddy
Carpio,MD-MBA
For inquiries visit www.topnotchboardprep.com.ph or email us at topnotchmedicalboardprep@gmail.com
Develops by leakage from a GI anastomosis Initial cellular injury that occurs is reversible; however,
or intra-abdominal abscess in patients in injury will become irreversible if tissue perfusion is
whom standard therapy fails prolonged or severe enough such that, at the cellular
Common in immunosuppressed patients level, compensation is no longer possible
Microbes such as E. faecalis and faecium, S.
epidermidis, C. albicans, and P. aeruginosa can
Figure 11. Pathways leading to decreased tissue perfusion and shock
be identified
Abscess is diagnosed via abdominal CT
Treatment || CT-guided percutaneous
drainage for intra-abdominal abscess
Surgical intervention is reserved for patients
with multiple abscesses, those with abscesses
in proximity to vital structures such that
Tertiary percutaneous drainage would be hazardous,
(persistent) and those in whom an ongoing source of
Peritonitis or contamination (e.g., enteric leak) is identified
Postoperative Necessity of antimicrobial agent therapy and
Peritonitis precise guidelines that dictate duration of
catheter drainage have NOT been established
Short course (3 to 7 days) of antibiotics that
covers for aerobic and anaerobic bacteria can
be given
Unfortunately, even with effective
antimicrobial agent therapy, this disease
process is associated with mortality rates of
more than 50%
Drainage catheter is left in situ until the
abscess cavity collapse, its output is less than
10-20 ml/d with no evidence of an ongoing
source of contamination and the patient's
clinical condition has improved
Clinical manifestations of several physiologic responses
are most often what lead practitioners to the diagnosis
3. Postoperative Nosocomial Infections of shock as well as guide the management of patients
Include SSIs, UTIs, pneumonia, and bacteremia Shock is classified into six types (Refer to Table 26)
Most infections are related to prolonged use of
indwelling tubes and catheters for the purpose of Table 26. Types of Shock
urinary drainage, ventilation, and venous and arterial Postoperative nosocomial infections
access, respectively (Refer to Table 25). Most common type
Results from loss of circulating blood
Table 25. Postoperative nosocomial infections volumedue to loss of whole blood
Postoperative nosocomial infections (hemorrhagic shock), plasma, interstitial
Should be considered based on urinalysis fluid (bowel obstruction)
with WBCs or bacteria, a positive test for Clinical and physiologic response is
leukocyte esterase, or a combination of classified according to the magnitude of
these elements volume loss (Refer to Table 27)
Diagnosis is established after more than Hypovolemic Treatment || Instituted with diagnostic
104 CFU/ml of microbes are identified by evaluation to identify a bleeding source
Postoperative culture techniques in symptomatic patients, Appropriate priorities are secure the
Urinary Tract or more than 105 CFU/ml in asymptomatic airway, control source of blood loss, and
Infection (UTI) individuals IV volume resuscitation
Treatment ||Single antibiotic therapy for 3 Patients who fail to respond to initial
to 5 days resuscitative efforts should be assumed to
Indwelling urinary catheters should be have ongoing active hemorrhage and
removed as quickly as possible, typically require prompt operative intervention
within 1 to 2 days, as long as the patients Results from decreased resistance within
are mobile capacitance vessels
Associated with prolonged mechanical Evaluationbegins with an assessment of the
ventilation and is frequently due to adequacy of airway and ventilation
pathogens common in the nosocomial Treatment ||Fluid resuscitation and
environment restoration of circulatory volume
Diagnosis should be made using the Empiric antibiotics must be chosen carefully
Vasogenic
presence of a purulent sputum, elevated (gram-negative rods, gram-positive cocci,
(Septic)
leukocyte count, fever, and new chest x-ray and anaerobes)
abnormality However, IV antibiotics without source
Pneumonia Presence of two of the clinical findings, plus control will be insufficient to adequately
chest x-ray findings, significantly increases treat patients with infected fluid collections,
the likelihood of ventilator-associated infected foreign bodies, and devitalized
pneumonia tissue
Treatment ||Antibiotic therapy for 7 to 10 Vasopressors may be necessary as well
days Form of vasogenic shock in which spinal
Surgical patients should be weaned from cord injury or spinal anesthesia causes
mechanical ventilation as soon as feasible, vasodilation due to acute loss of
based on oxygenation and inspiratory effort sympathetic vascular tone
Treatment ||After airway is secured and
ventilation is adequate, fluid resuscitation
E. SHOCK and restoration of intravascular volume
Neurogenic
Failure to meet the metabolic needs of the cell and often will improve perfusion
Administration of vasoconstrictors will
the consequences that ensue
improve peripheral vascular tone, decrease
Consists of inadequate tissue perfusion marked by vascular capacitance, and increase venous
decreased delivery of required metabolic substrates return
and inadequate removal of cellular waste products If the patient's blood pressure has not
(Refer to Figure 11) responded, dopamine may be used

TOPNOTCH MEDICAL BOARD PREP SURGERY SUPPLEMENT HANDOUT Page 19 of 85


For inquiries visit www.topnotchboardprep.com.ph or email us at topnotchmedicalboardprep@gmail.com
TOPNOTCH MEDICAL BOARD PREP SURGERY SUPPLEMENT HANDOUT - Jules Lopez,MD-MBA,Teddy
Carpio,MD-MBA
For inquiries visit www.topnotchboardprep.com.ph or email us at topnotchmedicalboardprep@gmail.com
Results from failure of the heart as a
pump, as in arrhythmias or acute
myocardial infarction (most common)
Hemodynamic criteria include sustained
hypotension (i.e. SBP <90 mmHg for at least
30 minutes), reduced cardiac index (<2.2
L/min/m2), and elevated pulmonary artery
wedge pressure (>15 mmHg)
Treatment ||Ensure adequate airway is
Cardiogenic present and ventilation is sufficient
Treatment of cardiac dysfunction includes
maintenance of adequate oxygenation to
ensure adequate myocardial O2 delivery and
judicious fluid administration to avoid fluid
overload and development of cardiogenic
pulmonary edema
Significant dysrhythmias and heart block
must be treated with antiarrhythmic drugs,
pacing, or cardioversion

Form of cardiogenic shock that results


from mechanical impediment to
circulation leading to depressed cardiac
output rather than primary cardiac failure
Causes include cardiac tamponade,
Obstructive Table 28. Endpoints in resuscitation
pulmonary embolism, tension pneumothorax,
Systemic/Global
IVC obstruction (DVT, gravid uterus),
increased intrathoracic pressure (neoplasm) Lactate
Treatment || Dependent on the etiology of Base deficit
the obstructive shock Cardiac output
Soft tissue and bony injury lead to the Oxygen delivery and consumption
activation of inflammatory cells and the Tissue Specific
release of circulating factors that modulate Gastric tonometry
the immune response Tissue pH, Oxygen, Carbon dioxide levels
These effects of tissue injury are combined Near infrared spectroscopy
with the effects of hemorrhage, creating a Cellular
more complex and amplified deviation from Membrane potential
homeostasis. Adenosine triphosphate
Traumatic
Treatment || Correction of the individual
elements to diminish the cascade of REVIEW QUESTIONS a
proinflammatory activation, and includes
prompt control of hemorrhage, adequate
volume resuscitation to correct O2 debt, 1. Which of the following is the most effective dosing of
debridement of nonviable tissue, antibiotics in a patient undergoing elective colon
stabilization of bony injuries, and resection?
appropriate treatment of soft tissue injuries a. A single dose given within 30 min prior to
skin incision
b. A single doe given at the time of skin incision
Table 27. Signs and symptoms of advancing stages of hypovolemic shock c. A single preoperative dose + 24 hours of

postoperative antibiotics
Class I Class II Class III Class IV d. A single preoperative dose + 48 hours of
Blood loss 1,500-
Up to 750 750-1,500 >2,000 postoperative antibiotics
(ml) 2,000
Blood loss
Up to Answer: A
(%blood 15-30% 30-40% >40%
15% Prophylaxis is the administration of an antimicrobial
volume)
Pulse rate <100 >100 >120 >140 agent(s) before and during the operative procedure to
Blood reduce the number of microbes that enter the tissue or
Normal Normal Decreased Decreased
pressure body cavity. Only a single dose of antibiotic is
Pulse Normal or required, and only for certain types of surgical
Decreased Decreased Decreased
pressure increased procedures. There is no evidence that administration
Respiratory of postoperative doses provides additional benefit.
14-20 20-30 30-40 >35
rate
Urine
output >30 20-30 5-15 Negligible
2. What percentage of the blood volume is normally in
(ml/h) the splanchnic circulation?
Anxious Confused a. 10%
CNS/mental Slightly Mildly b. 20%
and and
status anxious anxious
confused lethargic c. 30%
d. 40%

Answer: B
Most alterations in cardiac output in the normal heart
are related to changes in preload. Increases in
sympathetic tone have a minor effect on skeletal
muscle beds but produce a dramatic reduction in
splanchnic blood volume, which holds 20% of the
blood volume.

3. Which of the following best describes the


hemodynamic response to neurogenic shock?

INITIAL FLUID RESUSCITATION FOR HEMORRHAGIC SHOCK


TOPNOTCH MEDICAL BOARD
GOAL: To re-establish tissue PREP SURGERY
perfusion SUPPLEMENT HANDOUT
and oxygenation Page 20 of 85
For inquiries visit www.topnotchboardprep.com.ph or email us at topnotchmedicalboardprep@gmail.com
Crystalloid is the first fluid of choice for resuscitation.
Immediately administer 2 L of isotonic sodium
chloride solution or lactated Ringers solution in
TOPNOTCH MEDICAL BOARD PREP SURGERY SUPPLEMENT HANDOUT - Jules Lopez,MD-MBA,Teddy
Carpio,MD-MBA
For inquiries visit www.topnotchboardprep.com.ph or email us at topnotchmedicalboardprep@gmail.com
a. Increased cardiac index, unchanged venous Breathing
capacitance Tension pneumothorax
b. Increased cardiac index, decreased venous Open pneumothorax
capacitance Flail chest with underlying pulmonary contusion
c. Variable change in cardiac index (can Circulation
increase or decrease), increased venous Massive hemothorax or hemoperitoneum
capacitance Hemorrhagic shock Mechanically unstable pelvis fracture
d. Variable change in cardiac index (can
Extremity losses
increase or decrease), decreased venous
capacitance Cardiogenic shock: Cardiac tamponade
Neurogenic shock: Cervical spine injury
Disability
Answer: A
Intracranial hemorrhage/mass lesion
Choice B and D are most commonly associated with
septic shock. Choice C, on the other hand, is most likely
1. Airway management with cervical spine protection
seen in cardiogenic shock.
Ensuring a patent airway is the first priority in the
4. An unconscious patient with a systolic BP of 80 and a primary survey
HR of 80 most likely has? Efforts to restore cardiovascular integrity will be futile
a. Cardiogenic shock unless the oxygen content of the blood is adequate
b. Hemorrhagic shock All patients with blunt trauma require cervical spine
c. Neurogenic shock immobilization (hard collar or placing sandbags on
d. Obstructive shock both sides of the head with the patients forehead taped
across bags to the backboard) until injury is excluded
Answer: C Patients who are conscious, do not show tachypnea, and
Sympathetic input to the heart, which normally have a normal voice do not require early attention to the
increases heart rate and cardiac contractility, and airway EXCEPT the following:
input to the adrenal medulla, which increases o Patients with penetrating injuries to the neck
catecholamine release, may also be disrupted (with and an expanding hematoma
spinal cord injury), preventing the typical reflex o Evidence of chemical or thermal injury to the
tachycardia that occurs with hypovolemia. mouth, nares, or hypopharynx
The classic description of neurogenic shock consist of o Extensive subcutaneous air in the neck
decreased blood pressure associated with o Complex maxillofacial trauma
bradycardia (absence of reflex tachycardia due to o Airway bleeding
disrupted sympathetic discharge), warm extremities Elective intubation should be performed on the cases
(loss of peripheral vasoconstriction), motor and above before evidence of airway compromise
sensory deficits indicative of a spinal cord injury, and Altered mental status is the most common
radiographic evidence of a vertebral column fracture. indication for intubation
Options for endotracheal intubation include
nasotracheal, orotracheal, or surgical routes
TRAUMA o Nasotracheal: Only done in patients, who are
breathing spontaneously, requiring emergent
airway support in whom chemical paralysis
A. General Principle
cannot be used
B. Primary Survey
o Orotracheal: most common technique used
C. Resuscitation
to establish a definitive airway
D. Secondary Survey
o Surgical (cricothyroidotomy): Done in
E. Diagnostic Evaluation 2nd ICS
patients in whom attempts at intubation
F. Definitive Care
have failed or who are precluded from
intubation due to extensive facial injuries
o Surgical (emergent tracheostomy): Indicated
A. GENERAL PRINCIPLE in patients with laryngotracheal separation or
Trauma or injury is a cellular disruption caused by laryngeal fractures, in whom
an exchange with environmental energy that is cricothyroidotomy may cause further damage
beyond the bodys resilience or result in complete loss of airway
Most common cause of death for all individuals
between the ages of 1 and 44 years 2. Breathing and Ventilation
Third most common cause of death regardless of age Once a secure airway is obtained, adequate oxygenation
Most common cause of years of productive life lost and ventilation must be assured
Initial management of seriously injured patients All injured patients should receive supplemental oxygen
according to the Advanced Trauma Life Support (ATLS) and be monitored by pulse oximetry
consists of the following: The following conditions constitute an immediate threat
o Primary survey to life due to inadequate ventilation (Refer to Table 30)
o Concurrent resuscitation
o Secondary survey Table 30. Life-threatening injury identified due to inadequate ventilation
o Diagnostic evaluation Inadequate Ventilation
o Definitive care Diagnosis is implied by respiratory
distress and hypotension in combination
B. PRIMARY SURVEY with any of the following physical signs in
Goal is to identify and treat conditions that constitute an patients with chest trauma: tracheal
deviation away from the affected side, lack
immediate threat to life (Refer to Table 29)
of or decreased breath sounds on the
Assessment of the ABCDE (Airway with cervical Tension
affected side, and subcutaneous emphysema
spine protection, Breathing, Circulation, Disability, Pneumothorax
on the affected side(Refer to Figure
and Exposure) 10inset)
Treatment ||Needle thoracostomy
Table 29. Life-threatening injuries identified during the primary survey decompressionin the 2nd ICS in the MCL
Airway may be indicated in the acute setting(Refer
Airway obstruction to Figure 10)
Airway injury
TOPNOTCH MEDICAL BOARD PREP SURGERY SUPPLEMENT HANDOUT Page 21 of 85
For inquiries visit www.topnotchboardprep.com.ph or email us at topnotchmedicalboardprep@gmail.com
TOPNOTCH MEDICAL BOARD PREP SURGERY SUPPLEMENT HANDOUT - Jules Lopez,MD-MBA,Teddy
Carpio,MD-MBA
For inquiries visit www.topnotchboardprep.com.ph or email us at topnotchmedicalboardprep@gmail.com
Closed tube thoracostomy should be
performed immediately before a chest
radiograph is obtained
Occurs with full thickness loss of the chest
wall, permitting free communication
between the pleural space and the
atmosphere
Compromises ventilation due to
equilibration of atmospheric and pleural
pressures, which prevents lung inflation
Open Figure 14. Mechanism of a Flail Chest.Paradoxical movement of the flail
and alveolar ventilation, and results in
Pneumothorax chest during inspiration and expiration.
hypoxia and hypercarbia
(Sucking chest
Complete occlusion of the chest wall defect
wound)
WITHOUT a tube thoracostomy may
convert an open pneumothorax to a
tension pneumothorax
Treatment ||Definitive treatment is
closure of the chest wall defect and closed
tube thoracostomy remote from the
wound (Refer to Figure 11)
Occurs when 3 or more contiguous ribs
are fractured in at least 2 locations
(Refer to Figure 12)
Paradoxical movement of this free floating
segment of chest wall may be evident in
Flail chest with patients with spontaneous ventilation, due
underlying to the negative intrapleural pressure of
pulmonary inspiration
contusion Associated pulmonary contusion is
typically the source of postinjury
pulmonary dysfunction (Decreased
compliance and increased shunt fraction) 3. Circulation with hemorrhage control
Treatment ||May require presumptive Initial approximation of the patients cardiovascular
intubation and mechanical ventilation status can be obtained by palpating peripheral pulses
o Carotid pulse: 60 mmHg systolic BP
o Femoral pulse: 70 mmHg
Figure 12. Tension Pneumothorax (inset) with Needle Thoracostomy o Radial pulse: 80 mmHg to be palpable
Any hypotensive episode (SBP <90 mmHg) is assumed
to be caused by hemorrhage until proven otherwise
IV access for fluid resuscitation is obtained with 2
peripheral catheters, 16-gauge or larger in adults
In patients under 6 years old, an intraosseus needle can
be placed in the proximal tibia (preferred) or distal
femur of an unfractured extremity
External control of hemorrhage should be achieved
promptly while circulating volume is restored
The following conditions constitute an immediate threat
to life due to inadequate circulation (Refer to Table 31)

Table 31. Life-threatening injury identified due to inadequate circulation


Inadequate Circulation
Defined as >1,500 ml of blood or, in the
pediatrics, 1/3 of the patients blood
volume in the pleural space
After a blunt trauma, hemothorax is usually
due to multiple rib fractures with severed
Figure 13. Closed Tube Thoracostomy (CTT).A. Performed in the MAL at Massive
intercostal arteries, but occasionally
the 4th-5th ICS to avoid iatrogenic injury to the liver or spleen. B. Heavy Hemothorax
bleeding isfrom lacerated lung parenchyma
scissors are used to cut through the intercostal muscle into the pleural After a penetrating trauma, a systemic or
space done on top of the rib to avoid injury to the intercostal bundle pulmonary hilar vessel injury should be
located just beneath the rib. C. Incision is digitally explored to confirm presumed
intrathoracic location and identify pleural adhesions. D. A 36F chest tube Treatment || Operative intervention
is directed superiorly and posteriorly with the aid of a large clamp. Occurs most commonly after penetrating
thoracic injuries, although occasionally
blunt rupture of the heart, particularly the
atrial appendage, is seen
<100 ml of pericardial blood may cause
pericardial tamponade
(Refer to Figure 13 right)
Becks triad (dilated neck veins, muffled
heart tones, and a decline in arterial
pressure) is NOT often observed
Cardiac Diagnosis is best achieved by ultrasound of
Tamponade the pericardium
(Refer to Figure 13 left)
Early in the course of tamponade, blood
pressure and cardiac output will transiently
improve with fluid administration
Treatment ||Pericardiocentesis is
successful in decompressing tamponade in
approximately 80% of cases (Refer to
Figure 14)
Removing as little as 15 to 20 ml of blood
will often temporarily stabilize the patients
TOPNOTCH MEDICAL BOARD PREP SURGERY SUPPLEMENT HANDOUT Page 22 of 85
For inquiries visit www.topnotchboardprep.com.ph or email us at topnotchmedicalboardprep@gmail.com
TOPNOTCH MEDICAL BOARD PREP SURGERY SUPPLEMENT HANDOUT - Jules Lopez,MD-MBA,Teddy
Carpio,MD-MBA
For inquiries visit www.topnotchboardprep.com.ph or email us at topnotchmedicalboardprep@gmail.com
hemodynamic status, prevent
subendocardial ischemia, and associated
lethal arrhythmias, and allow transport to
the OR for sternotomy
Patients with a SBP <70 mmHg warrant
emergency department
thoracotomy(EDT) with opening of the
pericardium to address the injury (Refer to
Table 31)
EDT is best accomplished using aleft
anterolateral thoracotomy, with the
incision started to the right of the sternum
(Refer to Figure 15)

4. Disability and Exposure


Glasgow Coma Scale (GCS) score should be determined
ANATOMY a
for all injured patients
Closed Tube Thoracostomy (CTT)is done on the
Scores of 13 to 15 indicate mild head injury, 9 to 12
superior border of the lower ribon the 4th-5th ICS MAL
moderate injury, and <9 severe injury
o Directed superiorly for air drainage
Abnormal mental status should prompt an immediate
o Directed inferiorly for fluid drainage
re-evaluation of the ABCs and consideration of central
o Tube passes through the following: Skin
nervous system injury
Superficial fascia Serratus anterior
Figure 15. Cardiac Tamponade with ultrasound findings (*) on the left. External intercostals Internal intercostals
Innermost intercostals Endothoracic fascia
Parietal pleura

QUICK REVIEW a

Primary survey consists of the assessment of the


ABCDE (Airway with cervical spine protection,
Breathing, Circulation, Disability, and Exposure)
Ensuring a patent airway is the first priority in the
primary survey
Figure 16. Pericardiocentesis. Access to the pericardium is obtained
Altered mental status is the most common indication
through a subxiphoid approach, with the needle angled 45 degrees up
from the chest wall and toward the left shoulder. for intubation
Massive hemothorax is defined as >1,500 ml of blood
or, in the pediatrics, 1/3 of the patients blood volume
in the pleural space
Tension pneumothorax is the most common cause of
cardiogenic shock in trauma patients

C. RESUSCITATION
Quantity of acute blood loss correlates with
physiologicabnormalities (Refer to Table 26)
o Tachycardia is often the earliest sign of
ongoing blood loss but watch out for relative
tachycardia (HR<90 in patients with a resting
pulse rate in the 50s)
o Bradycardia, an ominous sign, occurs with
severe blood loss, often heralding impending
cardiovascular collapse
o Hypotension is NOT a reliable early sign of
hypovolemia, because blood volume must
decrease by >30% before hypotension occurs
Goal is to re-establish tissue perfusion
Table 32. Emergency Department Thoracotomy (EDT) Indications and o Urine output is a quantitative, reliable
Contraindications. CPR = Cardiopulmonary resuscitation indicator of organ perfusion
Indications o Adequate urine output is 0.5 ml/kg/hr in
Patients sustaining witnessed penetrating an adult, 1 ml/kg/hr in a child, and 2
Salvegeable trauma with <15 min of prehospital CPR ml/kg/hr in an infant <1 year of age
postinjury cardiac Patients sustaining witnessed blunt trauma
o Fluid resuscitation begins with a 2L (adult) or
arrest with <5 min of
prehospital CPR
20 ml/kg (child) IV bolus of isotonic
Persistent severe Cardiac tamponade crystalloid, typically Ringers lactate
postinjury Hemorrhage (intrathoracic, intra- o For persistent hypotension, this is repeated
hypotension abdominal, extremity, cervical) once in adult and twice in a child before RBCs
(SBP60 mmHg) Air embolism are administered
Contraindications Based on the initial response to fluid resuscitation,
Penetrating trauma: CPR >15 min and no signs of life (papillary hypovolemic injured patients can be separated into
response, respiratory effort, motor activity) three broad categories: responders, transient responders,
Blunt trauma: CPR >5 min and no signs of life or asystole and nonresponders
o Responders:Individuals who are stable or
Figure 17. Emergency department thoracotomy (EDT) is performed
have a good response to the initial fluid
through the 5th ICS using the anterolateral approach. Pericardium is
opened anterior to the phrenic nerve, and the heart is rotated out for therapy as evidenced by normalization of vital
repair signs, mental status, and urine output are
unlikely to have significant ongoing
TOPNOTCH MEDICAL BOARD PREP SURGERY SUPPLEMENT HANDOUT Page 23 of 85
For inquiries visit www.topnotchboardprep.com.ph or email us at topnotchmedicalboardprep@gmail.com
TOPNOTCH MEDICAL BOARD PREP SURGERY SUPPLEMENT HANDOUT - Jules Lopez,MD-MBA,Teddy
Carpio,MD-MBA
For inquiries visit www.topnotchboardprep.com.ph or email us at topnotchmedicalboardprep@gmail.com
hemorrhage, and further diagnostic evaluation covering stabilized patients with warm blankets, and
for occult injuries can proceed in an orderly
fashion (Secondary survey)
o Transient Responders: Those who respond
initially to volume loading by an increase in
blood pressure only to then hemodynamically
deteriorate once more
o Nonresponders: These patients have
persistent hypotension despite aggressive
resuscitation
Patients with ongoing hemodynamic instability,
whether nonresponders or transient responders, require
systematic evaluation and prompt intervention

D. SECONDARY SURVEY
Once the immediate threats to life have been addressed,
a thorough history is obtained and the patient is
examined in a systematic fashion
Patient (or surrogate) should be queried to obtain an
AMPLE (Allergies, Medications, Past illnesses or
Pregnancy, Last meal, and Events related to the
injury)
Physical examination should be head to toewith special
attention to the patient's back, axilla, and perineum,
because injuries here are easily overlooked
All potentially seriously injured patients should
undergo digital rectal examination to evaluate for administering warmed IV fluids and blood products.
sphincter tone, presence of blood, rectal perforation, or a o Hemorrhagic shock impairs perfusion and
high-riding prostate, which is particularly critical in metabolic activity throughout the body, with
patients with suspected spinal cord injury, pelvic resultant decrease in heat production and
fracture, or transpelvic gunshot wounds body temperature
Vaginal examination with a speculum also should be o Hypothermia causes coagulopathy and
performed in women with pelvic fractures to exclude an myocardial irritability
open fracture PRBC transfusion should occur once the patient's
hemoglobin level is <7 g/dl, in the acute phase of
E. DIAGNOSTIC EVALUATION resuscitation the endpoint is 10 g/dl
Selective radiography and laboratory tests are done FFP is transfused to keep theINR <1.5 and PTT <45 sec
early in the evaluation after the primary survey Target of 100,000/l is the target platelet count with
For patients with severe blunt trauma, lateral cervical massive transfusion
spine, chest, and pelvic radiographs should be
obtained, often termed the big three 1. Neck
For patients with truncal gunshot wounds, Divided into three distinct zones that is important in the
anteroposterior and lateral radiographs of the chest management of neck injuries (Refer to Figure 18 )
and abdomen are warranted
In critically injured patients, blood samples for a routine
trauma panel (type and cross-match, complete blood Figure 18. For the purpose of evaluating penetrating injuries, the neck is
count, blood chemistries, coagulation studies, divided into three zones. Zone I is up to the level of the cricoid and is also
known as the thoracic outlet.Zone II is located between the cricoid
lactate level, and arterial blood gas analysis) should
cartilage and the angle of the mandible. Zone III is above the angle of the
be sent to the laboratory mandible.
For less severely injured patients only a complete blood
count and urinalysis may be required

F. DEFINITIVE CARE
All injured patients undergoing an operation should
receive preoperative antibiotics
Extended postoperative antibiotic therapy is
administered only for open fractures or significant intra-
abdominal contamination
Tetanus prophylaxis is administered to all patients
Trauma patients particularly (a) those with multiple
fractures of the pelvis and lower extremities, (b) those
with coma or spinal cord injury, and (c) those requiring
ligation of large veins in the abdomen and lower
extremitiesare at risk for venous thromboembolism
and its associated complications
o Low molecular weight heparin is initiated as
Imaging options include CT scan or five plain
soon as bleeding has been controlled and radiograph views of the cervical spine: lateral view
there is no intracranial pathology with visualization of C7-T1, anteroposterior view,
o In high-risk patients, removable inferior transoral odontoid views, and bilateral oblique views
vena caval filters should be considered if Identification of penetrating injuries to the neck with
there are contraindications to administration exsanguination, expanding hematomas, and airway
of low molecular weight heparin obstruction is a priority during the primary survey
o Pulsatile compression stockings or Management algorithm for penetrating neck injury
sequential compression devices are used patients is based on the presenting symptoms and
routinely unless there is a fracture anatomic location of injury (Refer to Figure 19)
Another prophylactic measure is thermal protectionby All blunt trauma patients should be assumed to have
maintaining a comfortable ambient temperature, cervical spine injuries until proven otherwise
TOPNOTCH MEDICAL BOARD PREP SURGERY SUPPLEMENT HANDOUT Page 24 of 85
For inquiries visit www.topnotchboardprep.com.ph or email us at topnotchmedicalboardprep@gmail.com
TOPNOTCH MEDICAL BOARD PREP SURGERY SUPPLEMENT HANDOUT - Jules Lopez,MD-MBA,Teddy
Carpio,MD-MBA
For inquiries visit www.topnotchboardprep.com.ph or email us at topnotchmedicalboardprep@gmail.com
because there is up to a 50% chance of
Figure 19.1. Algorithm for the selective management of penetrating neck requiring laparotomy
injuries. CT = computed tomography; CTA = computed tomographic o Debate remains over whether the optimal
angiography; GSW = gunshot wound; IR Embo = interventional radiology
diagnostic approach is serial examination,
embolization
diagnostic peritoneal lavage
(Refer to Figure 20), or CT scanning
o Values representing positive findings for
diagnostic peritoneal lavage are summarized
in Table 33

Figure 21. Diagnostic peritoneal lavage (DPL) is performed through an


infraumbilical incision unless the patient has a pelvic fracture or is
pregnant. Linea alba is sharply incised, and the catheter is directed into
the pelvis. Abdominal contents (diagnostic peritoneal aspiration) is
considered positive if >10 ml of blood is aspirated. If <10 ml is obtained, a
liter of NSS is instilled. Effluent is withdrawn via siphoning and sent to
the laboratory for analysis

Figure 19.2. Laryngeal Trauma Management Protocol

2. Abdomen
Diagnostic approach differs for penetrating trauma
(i.e. gun shot/stab wound) and blunt abdominal trauma
Management algorithm for penetrating abdominal
injury patients is primarily based on the anatomic
location of injury (Refer to Figure 18)
As a rule, minimal evaluation is required before
laparotomy for abdominal gunshot or shotgun wounds
because over 90% of patients have significant internal
injuries EXCEPT those isolated in the liver by CT scan;
in hemodynamically stable patients where
nonoperative observation may be considered
Abdominal stab wounds are less likely to injure intra-
abdominal organs and thus, diagnostic evaluation can
be afforded

Figure 20. Algorithm for the evaluation of penetrating abdominal


injuries. AASW = anterior abdominal stab wound (from costal margin to Table 33. Criteria for positive finding on diagnostic peritoneal lavage.
inguinal ligament and bilateral MAL); CT = computed tomography; DPL = Between 1,000-10,000/ml, do laparoscopy/thoracoscopy
diagnostic peritoneal lavage; GSW = gunshot wound; LWE = local wound
Anterior
exploration; RUQ = right upper quadrant; SW = stab wound. Anterior Thoracoabdominal
Abdominal Stab
Abdominal Stab Wound
Wound
Red blood cell
>100,000/ml >10,000/ml
(RBC) count
White blood cell
>500/ml
(WBC) count
Amylase
>19 IU/l
level
Alkaline
phosphatase >2 IU/l
level
Bilirubin
>0.01 mg/dl
level

Blunt abdominal trauma initially is evaluated by FAST


(Refer to Figure 22) exam in major trauma centers
FAST is not 100% sensitive so diagnostic peritoneal
Anterior abdominal stab wounds (AASW) should be aspiration is still advocated in hemodynamically
explored under local anesthesia in the ED to determine unstable patients without a defined source of blood loss
if the fascia has been violated to rule out abdominal hemorrhage
o Injuries that do not penetrate the peritoneal
cavity do not require further evaluation, and
the patient is discharged from the ED
o Patients with fascial penetration must be
further evaluated for intra-abdominal injury,
TOPNOTCH MEDICAL BOARD PREP SURGERY SUPPLEMENT HANDOUT Page 25 of 85
For inquiries visit www.topnotchboardprep.com.ph or email us at topnotchmedicalboardprep@gmail.com
TOPNOTCH MEDICAL BOARD PREP SURGERY SUPPLEMENT HANDOUT - Jules Lopez,MD-MBA,Teddy
Carpio,MD-MBA
For inquiries visit www.topnotchboardprep.com.ph or email us at topnotchmedicalboardprep@gmail.com

Answer: D
In patients under the age of 8, cricothyroidotomy is
contraindicated due to the risk of subglottic stenosis,
and tracheostomy should be performed.

2. A patient presents with stable vital signs and


respiratory distress after a stab wound to the chest.
Chest tubes are placed and an air leak is noted. The
Figure 22. FAST is used to identify free intraperitoneal fluid in patient is electively intubated. The patient arrests
(1) subxiphoid/pericardium, (2) Morison's pouch/hepatorenal recess, after positive pressure ventilation is started. What is
(3) left upper quadrant/perisplenic, and (4) pelvis. Although this method the most likely diagnosis?
is sensitive for detecting intraperitoneal fluid of >250 ml, it does not
a. Unrecognized hemorrhage in the abdomen
reliably determine the source of bleeding nor grade solid organ injuries.
b. Tension pneumothorax
c. Pericardial tamponade
d. Air embolism

Answer: D
Air emboli can occur after blunt or penetrating
trauma, when air from an injured bronchus enters an
adjacent injured pulmonary vein and returns air to the
left heart. Air accumulation in the left ventricle
impedes diastolic filling, and during systole air is
pumped into the coronary arteries, disrupting
coronary perfusion. Patient should be placed in
Trendelenburgs position to trap the air in the apex of
the left ventricle. Emergency thoracotomy is followed
by cross clamping (left picture) of the pulmonary
hilum on the side of the injury to prevent further
introduction of air. Air is aspirated from the apex of
the left ventricle and the aortic root with an 18-g
needle and 50-ml syringe (right picture). Vigorous
massage is used to force air bubble through the
Patients with fluid on FAST examination, considered a
coronary arteries. If unsuccessful, a tuberculin syringe
"positive FAST," who do not have immediate indications
may be used to aspirate air from the right coronary
for laparotomy and are hemodynamically stable
artery. Once circulation is restored, patient should be
undergo CT scanning to quantify their injuries
kept in Trendelenburgs with the pulmonary hilum
Management algorithm for blunt abdominal injury
clamped until pulmonary venous injury is controlled
patients is shown in Figure 23
operatively.
Figure 23. Algorithm for the initial evaluation of a patient with suspected
blunt abdominal trauma. CT = computed tomography; DPA = diagnostic
peritoneal aspiration; FAST = focused abdominal sonography for
trauma/focused assessment with sonography for trauma; Hct=hematocrit

QUICK REVIEW a 3. Which of the following is the expected blood loss in a


patient with 6 rib fractures?
Tachycardia is the earliest sign of ongoing blood loss a. 240 ml
Adequate urine output is 0.5 ml/kg/hr in an adult, 1 b. 480 ml
ml/kg/hr in a child, and 2 ml/kg/hr in an infant <1 c. 750 ml
year of age d. 1500 ml
Secondary survey consists of AMPLE (Allergies,
Medications, Past illnesses or Pregnancy, Last meal, Answer: C
and Events related to the injury) For each rib fracture, there is ~100-200 ml of
blood loss; for tibial fractures, 300-500 ml; for femur
fractures, 800-1000 ml; and for pelvic fractures,
>1000 ml. Although no single injury may appear to
REVIEW QUESTIONS a cause a patients hemodynamic instability, the sum of
the injuries may result in life-threatening blood loss
1. Which of the following trauma patients with airway
compromise and failed endotracheal intubation
should undergo emergency tracheostomy (rather
than a cricothyroidotomy)? BURNS
a. 84 y/o male with blunt trauma to the neck
b. 65 y/o female with a stab wound to the A. Classification of Burns
submandibular region B. Burn Depth
c. 16 y/o male with a gun shot wound to the C. Initial Evaluation of Burns
neck D. Management of Burns
d. 6 y/o female with a crush injury to the face E. Inhalational Injury
TOPNOTCH MEDICAL BOARD PREP SURGERY SUPPLEMENT HANDOUT Page 26 of 85
For inquiries visit www.topnotchboardprep.com.ph or email us at topnotchmedicalboardprep@gmail.com
TOPNOTCH MEDICAL BOARD PREP SURGERY SUPPLEMENT HANDOUT - Jules Lopez,MD-MBA,Teddy
Carpio,MD-MBA
For inquiries visit www.topnotchboardprep.com.ph or email us at topnotchmedicalboardprep@gmail.com
o Stratum Corneum is a superficial stratum
later consisting of flat, anucleated and
A. CLASSIFICATION OF BURNS keratinized cells filled with keratin filaments
1. Thermal embedded in a dense matrix of proteins
Flame: Most common cause for hospital admission; o Stratum Lucidum is only found in regions of
highest mortality (due to association with inhalational thick stratum corneum of palms and soles;
injury and/or Carbon Monoxide (CO) poisoning) not found in thin skin
Contact o Stratum Granulosum is polygonal cells with
Scald basophilic keratohyalin granules; 1 layer in
thin skin while multiple layers in thin skin
2. Electrical o Stratum Spinosum is a multilaminar layer of
Potential for cardiac arrhythmias; do baseline ECG i cuboidal-like cells that are bound together by
Compartment syndromes with concurrent means of numerous desmosomal junctions
rhabdomyolysis is more common in high-voltage (tonofibrils) and they produce keratin
injuries; check for neurologic or vascular compromise o Stratum Basale/germinativum is a
Long-term neurologic and visual symptoms are also mitotically active, single layer of columnar or
common and thus, neurologic and ophthalmologic cuboidal cells attached to the dermis via
consultation should be done hemidesmosome
o Mnemonics: Californians Like Girls in String
3. Chemical Bikinis
Less common but usually severe Dermis is the connective tissue layer below the
epidermis and its basement membrane, consisting of 2
Offending agents can be systematically absorbed; may
layers:
cause specific metabolic derangements
o Papillary layer appears loose that fills the
Careful removal of toxic substance from patient and
hollows at the deep surface of the epidermis
irrigation of the affected area with water (~30 mins)
with frequent capillaries
EXCEPT in cases of concrete powder or powdered forms
o Reticular layer appears denser and contains
of lye, which should be swept from the patient instead
fewer cells
to avoid activation of AlOH with water
Hypodermis is a layer of loose vascular connective
tissue infiltrated by adipocytes
B. BURN DEPTH
Burn wounds are commonly stratified according to
depth as superficial, partial thickness, full thickness, and
fourth degree burns, which affect underlying soft tissue
They are also described according to zone of tissue
injury (Refer to Table 34)

1. Superficial (First degree burn)


Painful but DO NOT blister
2. Partial thickness (Second degree burn)
Extremely painful with weeping and blisters
Classified as either superficial or deep depending on the
depth of dermal involvement
o Superficial: Heals with expectant management
o Deep: Requires excision and skin grafting

3. Full thickness (Third degree burn)


Painless, hard, and non-blanching

4. Fourth degree burn


Affects underlying soft tissue

Table 34. Jacksons three zones of tissue injury following burn C. INITIAL EVALUATION OF BURNS
Jacksons three zones of tissue injury following burn
1. Airway management
Most severely burned area (typically the
Zone of center of the wound)
With direct thermal injury to the upper airway and/or
Coagulation Affected tissue is coagulated and sometimes smoke inhalation (perioral burns, signed nasal hairs),
necrotic, and will need excision and grafting rapid and severe airway edema is a potentially lethal
Between the first and third zones with local threat
response of vasoconstriction and ischemia Anticipating the need for intubation and establishing an
It has marginal perfusion and questionable early airway is critical
viability Signs of impending respiratory compromise: hoarse
Zone of Stasis
Resuscitation and wound care may help voice, wheezing, or stridor
prevent conversion to a deeper burn
Burn wounds evolve over 48-72 hours after
2. Evaluation of other injuries
injury
Outermost area, usually heals with minimal Burn patients should be first considered
Zone of traumapatients (especially when details of the injury
or no scarring
Hyperemia are unclear), as such, a primary survey should be
There is increased blood flow in this area
conducted
An early and comprehensive secondary survey must
also be performed in all burn patients
Urgent radiology studies (i.e. CXR) should be performed
ANATOMY a in the ER, but non urgent skeletal evaluation (i.e.
LAYERS OF THE SKIN extremity X-rays) can be done later to avoid
Epidermis is the outermost layer of the integument hypothermia and delays in burn resuscitation
composed of stratified squamous epithelial layer that is
devoid of blood vessels, consisting of 4-5 layers: 3. Estimation of burn size

TOPNOTCH MEDICAL BOARD PREP SURGERY SUPPLEMENT HANDOUT Page 27 of 85


For inquiries visit www.topnotchboardprep.com.ph or email us at topnotchmedicalboardprep@gmail.com
TOPNOTCH MEDICAL BOARD PREP SURGERY SUPPLEMENT HANDOUT - Jules Lopez,MD-MBA,Teddy
Carpio,MD-MBA
For inquiries visit www.topnotchboardprep.com.ph or email us at topnotchmedicalboardprep@gmail.com
Most burn resuscitation formulas estimate fluid Rationale: Burn (and/or inhalational injury) drives
requirements using the burn size as %Total Body inflammatory response that leads to capillary leak
Surface Area (TBSA) As the plasma leaks into the extravascular space,
Rule of nines is a crude but quick and effective crystalloid administration maintains the
method of estimating burn size (Refer to Figure 24 ) intravascular volume
Thorough cleaning of soot and debris is mandatory to Therefore, if a patient receives large fluid bolus in a
avoid confusing areas of soiling with burns prehospital setting or ER, that fluid has likely
Superficial (first degree) burns SHOULD NOT be leaked into the interstitium and the patient will still
included when calculating the %TBSA require ongoing burn resuscitation
Several formulas are available to compute for the
total fluid requirement but among the most widely
used one is the Parkland formula(Refer to Table
36 )
Table 36. Parkland formula
Parkland formula
Total fluid requirement* = 4 mg/kg per %TBSA burn
volume during first 8 hours during next 16 hours post-
post-injury injury
*Use of lactated ringers solution
Figure 24. Rule of nines to estimate burn size
Continuation of fluid volumes should depend on
the time since injury, UO, and MAP
As the leak closes, patient will require less volume
to maintain the UO and BP
o Target MAP: 60 mmHg to ensure optimal
end-organ perfusion
o Target UO:30 cc/h in adults and 1 to 1.5
cc/kg/hr in pediatric patients
Maintenance IV fluid with glucose
supplementation in addition to the calculated
resuscitation fluid with LR is given in children
under 20 kg
o They do not have sufficient glycogen
stores to maintain an adequate glucose
level in response to the inflammation.
Blood transfusions be used only when there is an
apparent physiologic need

3. Treatment of burn wound


4. Diagnosis of Carbon Monoxide and Cyanide poisoning Patients with acute burn injuries should NEVER receive
Unexpected neurologic symptoms should raise the level prophylactic oral/IV antibiotics
of suspicion for CO poisoning This intervention has been clearly demonstrated to
o Affinity of CO for hemoglobin is 200-250x promote development of fungal infections and resistant
more than that of O2, which decreases the organisms
levels of normal oxygenated hemoglobin and Silver sulfadiazine: most widely used
can quickly lead to anoxia and death o Wide range of anti-microbial activity,
o Treatment ||Administration of 100% oxygen primarily as topical prophylaxis against burn
is the gold standard, and reduces the half-life wound infections rather than treatment of
of CO from 250 mins in room air to 40-60 mins existing infection
Cyanide poisoning is seen in smoke inhalation injury o Not significantly absorbed systemically
o May have lactic acidosis or ST elevation o Side effects ||Neutropenia as a result of
o Cyanide inhibits cytochrome oxidase, which in neutrophil margination due to the
turn inhibit cellular oxygenation inflammatory response to burn injury
o Treatment ||Consists of sodium thiosulfate, o Destroy skin grafts and is contraindicated on
hydroxocobalamin, and 100% oxygen burns in proximity to newly grafted areas
Others: Mafenide acetate, Silver nitrate, Bacitracin,
D. MANAGEMENT OF BURNS Neomycin, and Polymyxin B
1. Referral to a burn center (Refer to Table 35) Pain management
o Important to administer an anxiolytic such as
Table 35. Guidelines for referral to a burn center benzodiazepine with the initial narcotics
Guidelines for referral to a burn center
1. Partial thickness burns greater than 10% TBSA 4.
Complications of burn
2. Burns involving the face, hands, feet, genitalia, perineum, or Hypothermia is one of the common pre-hospital
major joints complications that contributes to resuscitation failure
3. Third degree burns in any age group o Patients should be kept wrapped with clean
4. Electric burns (including lightning injury)
blankets
5. Chemical burns
6. Inhalational injury
Ventilator-associated pneumonia, like all critically ill
7. Patients with complicated preexisting medical disorders patients, is a significant problem in burn patients
8. Patients with burns and concomitant trauma in which the o Simple measures such as elevating the head of
burn is the greatest risk the bed and maintaining excellent oral hygiene
9. Burned children in hospitals without qualified personnel for and pulmonary toilet are recommended to
the care of children help decrease the risk of postinjury
10. Burn injury in patients who will require special social, pneumonia
emotional, or rehabilitative intervention 4. Complications of burn(continuation)
Massive resuscitation of burn patients may lead to an
2. Resuscitation abdominal compartment syndrome
o Characterized by increased airway pressures
TOPNOTCH MEDICAL BOARD PREP SURGERY SUPPLEMENT HANDOUT Page 28 of 85
For inquiries visit www.topnotchboardprep.com.ph or email us at topnotchmedicalboardprep@gmail.com
TOPNOTCH MEDICAL BOARD PREP SURGERY SUPPLEMENT HANDOUT - Jules Lopez,MD-MBA,Teddy
Carpio,MD-MBA
For inquiries visit www.topnotchboardprep.com.ph or email us at topnotchmedicalboardprep@gmail.com
with hypoventilation, and decreased urine eventually obstruction of the lower
output and hemodynamic compromise airways
o Treatment ||Decompressive laparotomy is Physiologic effects include decrease lung compliance,
the standard of care for refractory abdominal increase airway resistance work of breathing, increase
compartment syndrome but carries an overall metabolic demands, and an increase in fluid
especially lethal prognosis in burn patients requirements during resuscitation of patients with burn
o Adjunctive measures such as minimizing fluid, injuries
performing truncal escharotomies, decreasing Treatment ||Supportive care including aggressive
tidal volumes, and chemical paralysis should pulmonary toilet, routine use of nebulized agents (e.g.
be initiated before resorting to decompressive Salbutamol) and ventilation for ARDS
laparotomy
Burn patients may be at higher risk for catheter- REVIEW QUESTIONS a
related bloodstream infections
Full thickness burns with a rigid eschar can form a 1. Which of the following patients should be
tourniquet effect as the edema progresses, leading to immediately referred to a burn center?
compromised venous outflow and eventually arterial a. 20 y/o with a 12% partial thickness burn
inflow, leading to compartment syndrome b. 30 y/o with a major liver injury and a 15%
o Common in circumferential extremity burns partial thickness burn
o Warning signs include paresthesia, pain, c. 2% TBSA partial thickness burn to the
decreased capillary refill, and progression to anterior leg, crossing the knee
loss of distal pulses d. 10 y/o with a 7% partial thickness burn

5. Nutrition of burn patients Answer: A


Burn injury causes a hypermetabolic response raising All patients with a partial thickness burn >10% TBSA
baseline metabolic rates by as much as 200%, leading to should be transferred to a burn center. A patient with
catabolism of muscle proteins and decreased lean a burn and other major trauma can be treated in the
body mass that delay functional recovery trauma center first. Burns that involve the entire joint
Early enteral feeding for patients help prevent loss of should be transferred to a burn center, but a small
lean body mass, slow the hypermetabolic response, and burn to the anterior surface of the knee would not
result in a more efficient protein metabolism necessarily mandate transfer. Children should be
If enteral feeds are started within the first few hours transferred if there are no personnel able to care for
after admission, gastric ileus can often be avoided them, but for a child with a 7% TBSA burn, this would
not be mandatory
6. Surgery (Refer to Table 35)
Escharotomies are rarely needed within the first 8
hours following injury and SHOULD NOT be performed 2. Which of the following is indicated in a 46 y/o patient
unless indicated because of the aesthetic sequelae with a 22% TBSA partial thickness burn?
Burn excision and wound coverage should ideally a. Prophylactic 1st generation cephalosporin
start within the first several days, and in larger burns, b. Prophylactic clindamycin
serial excisions can be performed as the patients c. Tetanus booster
condition allows d. Tetanus toxoid
Excision is performed with repeated tangential slices
until only non burned tissue remains Answer: C
It is appropriate to leave healthy dermis, which will Patients with acute burn injuries should never receive
appear white with punctate areas of bleeding prophylactic antibiotics. This intervention promote
development of fungal infections and resistant
7. Wound coverage/ Grafts organisms and was abandoned in the mid-1980s. A
Split thickness sheet autografts make the most durable tetanus booster should be administered in the ER.
wound coverings
In larger burns, meshing of autografted skin provides a 3. Formic acid burns are associated with?
larger area of wound coverage, allowing drainage of a. Hemoglobinuria
blood and serous fluid to prevent accumulation under b. Rhabdomyolosis
the skin graft with subsequent graft loss c. Hypocalcemia
d. Hypokalemia
8. Rehabilitation
Should be initiated on admission Answer: A
Immediate and ongoing physical and occupational The offending agents in chemical burns can be
therapy is mandatory to prevent loss of physical systematically absorbed and may causes specific
function metabolic derangements. Formic acid has been
known to cause hemolysis and hemoglobinuria.
E. INHALATIONAL INJURY
Commonly seen in tandem with burn injuries 4. The major improvement in burn survival in the 20th
Drastically increase mortality in burn patients century can be attributed to the introduction of which
Causes injury in 2 ways: of the following therapies?
o Direct heat injury to the upper airways a. Antibiotics
Leads to maximal edema in the first b. Central venous fluid resuscitation
24 to 48 hours after injury c. Nutritional support
Will require short course of d. Early excision of the burn wound
endotracheal intubation for airway
protection Answer: D
o Inhalation of combustion products into the
lower airways
Irritants (combustion products) WOUND HEALING
cause direct mucosal injury leading
to mucosal sloughing, edema, A. Classification of Wound Healing
reactive bronchoconstriction, and B. Normal Phases of Wound Healing
C. Classification of Wounds
TOPNOTCH MEDICAL BOARD PREP SURGERY SUPPLEMENT HANDOUT Page 29 of 85
For inquiries visit www.topnotchboardprep.com.ph or email us at topnotchmedicalboardprep@gmail.com
TOPNOTCH MEDICAL BOARD PREP SURGERY SUPPLEMENT HANDOUT - Jules Lopez,MD-MBA,Teddy
Carpio,MD-MBA
For inquiries visit www.topnotchboardprep.com.ph or email us at topnotchmedicalboardprep@gmail.com
interaction and intercellular adhesion
molecules
A. CLASSIFICATION OF WOUND HEALING
Surgical wounds can heal in several ways T lymphocytes
(Refer to Figure 25 ) o Less numerous than macrophages
o Primary intention: an incised wound that is o Peak at about 1 week post injury and truly
clean and closed by sutures bridge the transition from the inflammatory to
o Secondary intention: Because of bacterial the proliferative stage of wound healing
contamination or tissue loss, a wound will be o Role is not fully defined
left open to heal by granulation tissue o Theory is that they play an active role in
formation and contraction modulation of the wound environment
o Tertiary intention or delayed primary o Exert a downregulating effect on fibroblast
closure: represents a combination of the first collagen synthesis by cell-associated
two, consisting of the placement of sutures, interferon-gamma, TNF alpha, and IL1
allowing the wound to stay open for a few
days, and the subsequent closure of the
sutures

Figure 25. Different clinical approaches to the closure and healing of


acute wounds

Figure 26. Phases of wound healing

2. Proliferation
B. NORMAL PHASES OF WOUND HEALING Roughly spans day 4 through 12
Normal wound healing follows a predictable pattern Phase where tissue continuity is re-established
that can be divided into three overlapping phases: Fibroblasts and endothelial cells are the last cell
1. Hemostasis and inflammation populations to infiltrate the healing wound
2. Proliferation Strongest chemotactic factor for fibroblasts is PDGF
3. Maturation and remodeling Upon entering the wound environment, recruited
fibroblasts first need to proliferate, and then become
1. Hemostasis and Inflammation activated, to carry out their primary function of matrix
Hemostasis precedes and initiates inflammation with synthesis remodeling
the ensuing release of chemotactic factors from wound Fibroblasts from wounds synthesize more collagen,
site proliferate less, and actively carry out matrix
Cellular infiltration after injury follows a characteristic, contraction
predetermined sequence o Type I collagen is the major component of
o PMNs are the first infiltrating cells to enter extracellular matrix in skin
the wound site, peaking at 24 to 48 hours, o Type III, which is also normally present in
stimulated by increased vascular permeability, skin, becomes more prominent and important
local prostaglandin release, and the presence during the repair process
of chemotactic substances Endothelial cells also proliferate extensively during this
o These cells DO NOT play a role in collagen phase of healing, participating in angiogenesis, under
deposition and collagen synthesis the influence of cytokines and growth factors such as
TNF-alpha, TGF-beta, and VEGF
Macrophages (Refer to Figure 26) Macrophages represent a major source of VEGF
o Recognized to be essential in successful
wound healing 3. Maturation and Remodeling
o Achieve significant numbers by 48 to 96 Begins during the fibroplastic phase
hours post injury and remain present until Characterized by a reorganization of previously
wound healing is complete synthesized collagen
o Participate in wound debridement via Collagen is broken down by matrix metalloproteases,
phagocytosis and the net wound collagen content is the result of a
o Contribute to microbial stasis via oxygen balance between collagenolysis and collagen synthesis
radical and nitric oxide synthesis There is a net shift toward collagen synthesis and
o Activation and recruitment of other cells via eventually the re-establishment of extracellular matrix
mediators as well as directly by cell-cell composed of a relatively acellular collagen-rich scar

TOPNOTCH MEDICAL BOARD PREP SURGERY SUPPLEMENT HANDOUT Page 30 of 85


For inquiries visit www.topnotchboardprep.com.ph or email us at topnotchmedicalboardprep@gmail.com
TOPNOTCH MEDICAL BOARD PREP SURGERY SUPPLEMENT HANDOUT - Jules Lopez,MD-MBA,Teddy
Carpio,MD-MBA
For inquiries visit www.topnotchboardprep.com.ph or email us at topnotchmedicalboardprep@gmail.com
Wound strength and mechanical integrity in the fresh
wound are determined by both the quantity and
quality of the newly deposited collagen
The deposition of matrix at the wound site follows a
characteristic pattern: fibronectin and collagen type III
constitute the early matrix scaffolding,
Glycosaminoglycans and proteoglycans represent the
next significant matrix components, and collagen type I
is the final matrix
By several weeks post injury, the amount of collagen in
the wound reaches a plateau, but the tensile strength
continues to increase for several more months
Scar remodeling continues for 6 to 12 months post
injury, gradually resulting in a mature, avascular, and
acellular scar
Mechanical strength of the scar never achieves that
of the uninjured tissue

4. Epithelialization
While tissue integrity and strength are being re-
established, the external barrier must also be restored
Characterized primarily by proliferation and
migration of epithelial cells adjacent to the wound 2. Chronic
Process begin within day 1 of injury and is seen as Defined as wounds that have failed to proceed through
thickening of epidermis at the wound edge the orderly process that produces satisfactory anatomic
Re-epithelialization is complete in less than 48 hours and functional integrity or that have proceeded through
in the case of approximated incised wounds, but may the repair process without producing an adequate
take longer in case of larger wounds, in which there is a anatomic and functional result
significant epidermal/dermal defect Wounds that have NOT healed in 3 months
Mediated by a combination of a loss of contact
inhibition, exposure to constituents of the extracellular Table 37. Factors affecting wound healing
matrix, particularly fibronectin, and cytokines produced Factors affecting wound healing
by immune mononuclear cells Systemic
Age
5. Wound Contraction Nutrition
All wounds undergo some degree of contraction Trauma
Starts almost immediately after injury despite the Metabolic diseases
Immunosuppression
absence of myofibroblasts
Connective tissue disorders
For wounds that do not have surgically approximated
Smoking
edges, the area of the wound will be decreased by this Local
action (healing by secondary intention), the shortening Mechanical injury
of the scar itself results in contracture Infection
Myofibroblast has been postulated as being the major Edema
cell responsible for contraction, and it differs from the Ischemic/necrotic tissue
normal fibroblast in that it possesses a cytoskeletal Topical agents
structure Ionizing radiation
Low oxygen tension
C. CLASSIFICATION OF WOUNDS Foreign bodies
1. Acute
Heal in a predictable manner and time frame
Process occurs with few complications and the end QUICK REVIEW a
result is a well-healed wound
Normal process of wound healing is characterized by a Normal wound healing follows a predictable pattern
constant and continual increase that reaches a plateau that can be divided into three overlapping
at some point post injury phases:Hemostasis and inflammation, Proliferation,
Wounds with delayed healing are characterized by and Maturation and remodeling
decreased wound breaking strength in comparison to PMNs are the first infiltrating cells to enter the wound
wounds that heal at a normal rate, however, they site, peaking at 24 to 48 hours
eventually achieve the same integrity and strength as Myofibroblast has been postulated as being the major
wounds that heal normally cell responsible for contraction
Delayed healing is caused by conditions such as
nutritional deficiencies, infections, or severe trauma
which reverts to normal with correction of the REVIEW QUESTIONS a
underlying pathophysiology (Refer to Figure 27)
Impaired healing is characterized by a failure to 1. The peak number of fibroblasts in a healing wound
achieve mechanical strength equivalent to normally occurs?
healed wounds a. 2 days post injury
Patients with compromised immune system (diabetics, b. 6 days post injury
chronic steroid usage, tissues damaged by c. 15 days post injury
radiotherapy) are prone to impaired healing d. 60 days post injury
(Refer to Table 37)
Answer: B
Figure 27. The acquisition of wound mechanical strength over time in See Figure 26
normal, delayed, and impaired healing
2. The first cells to migrate into a wound are:
a. Macrophages

TOPNOTCH MEDICAL BOARD PREP SURGERY SUPPLEMENT HANDOUT Page 31 of 85


For inquiries visit www.topnotchboardprep.com.ph or email us at topnotchmedicalboardprep@gmail.com
TOPNOTCH MEDICAL BOARD PREP SURGERY SUPPLEMENT HANDOUT - Jules Lopez,MD-MBA,Teddy
Carpio,MD-MBA
For inquiries visit www.topnotchboardprep.com.ph or email us at topnotchmedicalboardprep@gmail.com
b. T Lymphocytes with increased risk of wound infection, especially
c. PMNs when placed in fat. Drains may be placed in areas
d. Fibroblasts at risk of forming fluid collections.

Answer: C
PMNs are the 1st infiltrating cells to enter the wound
site, peaking at 24-48 hours. Increased vascular ORGAN SYSTEM PATHOLOGIES
permeability, local prostaglandin release and the
presence of chemotactic substances, such as SKIN AND SOFT TISSUES
complement factors, IL-1, TNF-alpha, TGF beta,
platelet factor 4, or bacterial products, all stimulate
A. Anatomy and Physiology of the Skin
neutrophil migration.
B. Injuries to the Skin and subcutaneous
C. Infections of the skin and the subcutaneous
3. The tensile strength of a completely healed wound
D. Inflammatory diseases of the skin and
approaches the strength of uninjured tissue?
subcutaneous
a. 2 weeks after injury
E. Benign tumors of the skin and subcutaneous
b. 3 months after injury
F. Malignant tumors of the skin
c. 12 months after injury
G. Syndromic skin malignancies
d. NEVER

Answer: D
By several weeks postinjury, the amount of collagen in
the wound reaches a plateau, but the tensile strength
continues to increase for several more months. Fibril
formation and fibril cross-linking result in decreased
collagen solubility, increased strength and increased
resistance to enzymatic degradation of the collagen
matrix. Scar remodeling continues for many months
(6-12) post-injury, gradually resulting in a mature,
avascular and acellular scar. The mechanical strength A. ANATOMY AND PHYSIOLOGY OF THE SKIN
of the scar never achieves that of the uninjured tissue.
QUICK REVIEW a
4. Which layer of the intestine has the greatest tensile Layers of the skin p. 26
strength (ability to hold sutures)? Phases of wound healing p. 29

a. serosa
b. muscularis Epidermis
c. submucosa o Keratinocyte transit time (basal layer to
d. mucosa shedding) is approximately 40 to 56 days.
o Melanocytes
Answer: C Derived from precursor cells of the neural
The submucosa is the layer that imparts the crest/neuroectodermal in origin
greatest tensile strength and gretest suture- Produce melanin from tyrosine and cysteine
holding capacity, a characteristic that should be Despite differences in skin tone, the density
kept in mind during surgical repair of GI tract. of melanocytes is constant among
Additionally, serosal healing is essential for individuals. It is the rate of melanin
quickly achieving a watertight seal from the production, transfer to keratinocytes, and
luminal side of the bowel. The importance of the melanosome degradation that determine
serosa is underscored by the significantly higher the degree of skin pigmentation
rates of anastomotic failure observed clinically in Cutaneous melanocytes play a critical role in
segments of bowel that are extraperitoneal and neutralizing the sun's harmful rays.
lack serosa (ex. Esophagus and rectum) UV-induced damage affects the function of
tumor suppressor genes, directly causes cell
5. A 20 year old male presents to the ER with large death, and facilitates neoplastic
contaminated laceration received during a touch transformation.
football game. It has been irrigated with normal saline
and subsequently debrided. Which suture should be QUICK REVIEW a
used to close the subcutaneous layer? What factors increase melanin production?
1. UV radiation
a. biologic absorbable monofilament (plain 2. Estrogen
gut) 3. Adrenocorticotropic hormone
b. synthetic absorbable monofilament 4. Melanocyte-stimulating hormone
c. absorbable braided
d. none of the above
o Keratinocytes
Answer: C Primarily found in the spindle layer
In general, the smallest suture required to hold Contains intermediate filaments (keratin)
the various layers of the wound in approximation provides flexible scaffolding resist external
should be selected in order to minimize suture- stress
related inflammation. Nonabsorbable or slowly Point mutations cause blistering diseases,
absorbing monofilament sutures are most such as epidermolysis bullosa, associated
suitable for approximating deep fascial layers, with spontaneous release of dermal-epidermal
particularly in the abdominal wall. Subcutaneous attachments.
tissues should be closed with braided absorbable o Langerhans cells (not Langhans cells!)
sutures, with care to avoid placement of sutures skin's macrophages; from the bone marrow
in fat. Although traditional teaching in wound expresses class II major histocompatibility
closure emphasized multiple-layer closures, antigens antigen-presenting capabilities.
additional layers of suture closure are associated
TOPNOTCH MEDICAL BOARD PREP SURGERY SUPPLEMENT HANDOUT Page 32 of 85
For inquiries visit www.topnotchboardprep.com.ph or email us at topnotchmedicalboardprep@gmail.com
TOPNOTCH MEDICAL BOARD PREP SURGERY SUPPLEMENT HANDOUT - Jules Lopez,MD-MBA,Teddy
Carpio,MD-MBA
For inquiries visit www.topnotchboardprep.com.ph or email us at topnotchmedicalboardprep@gmail.com
Functions: o Severe hypothermia primarily exerts its damaging
1. rejection of foreign bodies effect by causing direct cellular injury to bv walls
2. immunosurveillance against viral and microvascular thrombosis.
infections o skin's tensile strength decreases by 20% in a cold
3. immunosurveillance against neoplasms of environment [12C, (53.6F)].
the skin o Trench foot: reactive hyperthermia with blistering
Dermis as a result of prolonged exposure to ice-cold water
o Collagen (main functional component of the after rapidly bringing it back to normal
dermis) comprises 70% of its dry weight temperature
o Skin is primarily comprised of type I collagen
o Fetal dermis is primarily comprised of type III c. Pressure injury
collagen (reticulin fibers) provides tensile - 1 hour of 60 mmHg pressure can lead to
strength (property of the skin that resists histologically identifiable venous thrombosis, muscle
stretching) to both dermis and epidermis degeneration, and tissue necrosis
Cutaneous Adnexal Structures - Pressures:
1. Eccrine glands: sweat-producing glands located Normal arteriole: 32 mmHg
over the entire body but are concentrated on the Normal capillary: 20 mmHg
palms, soles, axillae, and forehead Normal venule: 12 mmHg
2. Apocrine glands: Pheromone producing glands Sitting: 300 mmHg
primarily found in the axillae and anogenital Sacral pressure at hospital mattress bed: 150
region. It is these structures that predispose both mmHg
regions to suppurative hidradenitis - Muscle tissue is more sensitive to ischemia than
3. hair follicles: contains a reservoir of pluripotential skin. Implication: necrosis usually extends to a deeper
stem cells critical in epidermal reproductivity area than that apparent on superficial inspection
- Treatment: relief of pressure, wound care, systemic
enhancement (nutritional optimization) and surgical
management (debridement of all necrotic tissue
followed by irrigation; if shallow ulcer close by
secondary intention; if deeper ulcer require surgical
debridement and coverage)
B. INJURIES TO THE SKIN AND SUBCUTANEOUS
d. Radiation exposure
a. Exposure to Caustic substances - Solar or UV radiation: most common form of radiation
exposure
Table 38. Difference between acidic and alkali injury - Melanin: most important protective factor from UV
Acidic Alkali related damage
Coagulative necrosis can Liquefactive necrosis - UV spectrum:
damage nerves, blood causes fat saponification
UVA (400 to 315 nm): majority of solar radiation
vessels and tendons but is that facilitates tissue
less damaging compared to penetration and increases that reaches the Earth
alkali injury tissue damage producing UVB (315 to 290 nm): less than 5% of all solar UV
a longer more sustained radiation; responsible for acute sunburn and
injury compared to acidic chronic skin damage leading to malignant
burns degeneration (known risk factor in the
Tx: copious irrigation with development of melanoma.)
either saline or water for 30 Tx: continuous irrigation
UVC (290 to 200 nm): absorbed by the ozone layer
minutes with water for 2 hours or
until symptomatic relief is
achieved C. INFECTIONS OF THE SKIN AND THE SUBCUTANEOUS
a. Cellulitis, Folliculitis, furuncles & carbuncles
- Intravenous fluid (IVF) extravasation: leakage of Table 39: Comparison of skin infections
Cellulitis Folliculitis Furuncles Carbuncles
injectable fluids into the interstitial space
- Superficial, -infection of -begins as folliculitis - deep seated
o Is considered a chemical burn spreading the hair but progresses as a infections
o Produces chemical toxicity, osmotic toxicity and infection of the follicle fluctuant nodule that result in
pressure effects in a closed environment. skin and subQ -usual cause: (boil/furuncle) multiple
o Culprits: -usual cause: Staphylococc -tx: warm water draining
Cationic substances: K, Ca and bicarbonate Grp. A strep & us, followed hastens liquefaction sinuses
Osmotically active agents: TPN, hypertonic S. aureus by G(-) & spontaneous -tx: incision
dextrose solution -tx for organisms rupture; incision and drainage
Antibiotics uncomplicated -tx: adequate and drainage if
cellulitis with hygiene necessary
Cytotoxic drugs / chemotherapeutic drugs no morbidities:
most common cause of extravasation in adults outpatient oral
o Most common site of extravasation in adults: antibiotics
dorsum of the hand
o Most common cause of extravasation in infants b. Necrotizing soft tissue infections
causing necrosis: high concentration dextrose, Ca, - Basis of classification:
bicarbonate and TPN the tissue plane affected and extent of invasion
necrotizing fasciitis: rapid, extensive
b. Thermal injuries hypothermic vs hyperthermic infection of the fascia deep to the adipose
injuries tissue
necrotizing myositis: primarily involves the
QUICK REVIEW a muscles but typically spreads to adjacent soft
Jacksons 3 zones of tissue injury for hyperthermic tissues
injuries p. 26 the anatomic site
Most common sites: the external genitalia,
perineum, or abdominal wall (Fournier
Hypothermic injuries gangrene)
the causative pathogen

TOPNOTCH MEDICAL BOARD PREP SURGERY SUPPLEMENT HANDOUT Page 33 of 85


For inquiries visit www.topnotchboardprep.com.ph or email us at topnotchmedicalboardprep@gmail.com
TOPNOTCH MEDICAL BOARD PREP SURGERY SUPPLEMENT HANDOUT - Jules Lopez,MD-MBA,Teddy
Carpio,MD-MBA
For inquiries visit www.topnotchboardprep.com.ph or email us at topnotchmedicalboardprep@gmail.com
polymicrobial more common than single - Main characteristic: rapidly enlarging, destructive,
organism infections cutaneous necrotic lesion with undermined border and
most common causative organisms: group A surrounding erythema
streptococci, enterococci, coagulase-negative - (+) associated with a systemic disease 50% of the time
staphylococci, S. aureus, S. epidermidis, and (inflammatory bowel disease, rheumatoid arthritis,
Clostridium species hematologic malignancy, and monoclonal
others (Gram negatives): Escherichia coli, immunoglobulin A gammapathy)
Enterobacter, Pseudomonas species, Proteus - Tx: Recognition of the underlying disease, systemic
species, Serratia species, and bacteroides steroids or cyclosporine & chemotherapy with
- risk factors: diabetes mellitus, malnutrition, obesity, aggressive wound care and skin graft coverage
chronic alcoholism, peripheral vascular disease, CLL,
steroid use, renal failure, cirrhosis, and autoimmune b. SSS vs TEN
deficiency syndrome Table 41: comparison between SSSS & TEN
- tx: prompt recognition, broad-spectrum IV antibiotics, SSSS TEN
aggressive surgical debridement (should be extensive- Difference:
including all skin, subcutaneous tissue, and muscle, until -caused by an exotoxin (TSS -caused by an immune response to
there is no further evidence of infected tissue followed toxin-1)produced during staph certain drugs (sulfonamides,
infection of the nasopharynx or phenytoin, barbiturates,
by as needed debridement), and aggressive fluid
middle ear cytokine release tetracycline)
replacement (needed to offset acute renal failure from throughout the body causing -more than 30% TBSA involved
ongoing sepsis) diffuse injury and systemic (if less than 10% TBSA SJS)
symptoms
c. Hidradenitis suppuritiva
- is a defect of the terminal follicular epithelium -histopath: cleavage plane in the - histopath: structural defect at
leading to apocrine gland blockage gives rise to granular layer of epidermis dermoepidermal jxn; similar to a
abscess formation in the axillary, inguinal, and 2nd degree burn
perianal regions Similarity:
- Tx: warm compresses, antibiotics, and open drainage if - appearance: skin erythema, bullae formation, wide area of tissue
acute; wide excision with closure using skin graft or loss
local flap placement if chronic -diagnosis: skin biopsy
treatment: fluid and electrolyte replacement, as well as wound care
similar to burn therapy
d. Actinomycosis
- appearance: skin erythema, bullae formation, wide area of tissue
- is a granulomatous suppurative bacterial disease & loss
deep cutaneous infections that present as nodules and -diagnosis: skin biopsy
spread to form draining tracts caused by Actinomyces treatment: fluid and electrolyte replacement, as well as wound care
(pathognomonic: (+) sulfur granules within similar to burn therapy
purulent specimen).
- Usual site: face or head (60%) E. BENIGN TUMORS OF THE SKIN AND SUBCUTANEOUS
- Risk factors: tooth extraction, odontogenic infection, or
facial trauma. a. Cutaneous cysts: Epidermal, dermoid or
- Tx: Penicillin and sulfonamides; surgery for deep seated trichelemmal
infections. Table 42: Comparison between epidermal, dermoid & trichilemmal cyst:

e. Viral infections HPV Epidermal cyst Dermoid cyst Trichilemmal (pilar)


- Warts are epidermal growths resulting from human cyst
papillomavirus (HPV) infection. Difference:
- most common - congenital lesions that -2nd most common
Table 40: Comparison of HPV infections - single, firm result when epithelium - when ruptured:
nodule is trapped during fetal produce an intense
Common Plantar Flat Venereal warts
midline closure characteristic odor
wart warts warts (condylomata
-location: - most common
(verruca (verruca (verruca acuminata)
anywhere in the location: eyebrow -location: scalp (of
vulgaris) plantaris) plana)
body females)
-fingers -soles and - the face, - the vulva, anus, and
and toes palms legs, and scrotum (relatively
-histopath: -histopath:
-described -resemble hands moist areas)
mature demonstrates -histopath: no
as rough a common - slightly - STD
epidermis squamous epithelium, granular layer;
and callus raised - HPV 6 & 11 complete with eccrine glands, and
bulbous and flat. -buschke
granular layer pilosebaceous units. In
Lowenstein tumor:
addition, these
Extensive growths,
particular cysts may
facilitated by
develop bone, tooth, or
concomitant HIV
nerve tissue on
infection
occasion

- histopathology: hyperkeratosis (hypertrophy of the Similiarity:


horny layer), acanthosis (hypertrophy of the -contain keratin (not sebum)
spinous layer), and papillomatosis -appear the same clinically (subcutaneous, thin-walled nodule
containing a white, creamy material)
- Tx: formalin, podophyllum, and phenol-nitric acid;
-treatment: excision; incision and drainage if infected; make sure to
Curettage with electrodesiccation also can be used for remove the cyst wall to prevent recurrence
scattered lesions
- HPV types 5, 8, and 10: (+) association with
squamous cell carcinoma: b. Keratosis seborrheic vs solar
lesions that grow rapidly, atypically, or ulcerate
should be biopsied Table 43. Comparison between seborrheic keratosis and
actinic keratosis:
D. INFLAMMATORY DISEASES OF THE SKIN AND Seborrheic (or solar) Actinic keratosis
SUBCUTANEOUS keratosis

a. Pyoderma gangrenosum

TOPNOTCH MEDICAL BOARD PREP SURGERY SUPPLEMENT HANDOUT Page 34 of 85


For inquiries visit www.topnotchboardprep.com.ph or email us at topnotchmedicalboardprep@gmail.com
TOPNOTCH MEDICAL BOARD PREP SURGERY SUPPLEMENT HANDOUT - Jules Lopez,MD-MBA,Teddy
Carpio,MD-MBA
For inquiries visit www.topnotchboardprep.com.ph or email us at topnotchmedicalboardprep@gmail.com
- considered as a -considered as a - with direct - histopath: tumor -histopath:
premalignant lesion of SCC premalignant lesion of SCC nerve contains Schwann granular cells
- appearance: light brown or (although at least 25% attachment cells with nuclei derived from
yellow with a velvety, greasy spontaneously regress) packed in Schwann cells
texture - histopath: , palisading row that often
-arise in sun exposed areas proliferation of - tx: resection infiltrate the
(face, forearms, back of hands) perineurial and surrounding
-common in old age groups endoneurial striated muscle.
-sudden eruptions are fibroblasts with -tx:resection
associated with internal Schwann cells
malignancies embedded in
-treatment: topical 5- collagen
fluorouracil, surgical excision,
electrodesiccation, and
dermabrasion
F. Malignant tumors of the skin
c. Nevi acquired vs congenital: both are histologically
similar.
Basal cell carcinoma
- most common type of skin cancer.
Acquired melanocytic nevi
- Arises from the pluripotential basal epithelial cells of
- Classification is based on different stages of maturation
epidermis and NOT DERMIS!
Junctional: epidermis
- Slow growing and metastasis is rare but are capable
Compound: extend partially into dermis
of extensive local tissue destruction
Dermal: dermis
- Subtypes:
Nodulocystic/noduloulcerative
Congenital nevi
70% of BCC tumors (most frequent form)
- Rare (less than 1% of neonates)
Waxy and frequently cream
- Giant congenital lesions (giant hairy nevi): appear in a
colored/translucent;over time, can present as
swim trunk distribution, chest, or back
a rolled, pearly borders surrounding a central
- may develop into malignant melanoma in 1 to 5% of
ulcer (rodent ulcer)
cases
Morpheaphorm
- tx: total excision of nevus
flat, plaque-like lesion
most aggressive clinically (due to presence of
d. Vascular tumors of the skin and subcutaneous type IV collagenase that facilitates local
spread) early excision
basosquamous type
combination of both BCC + SCC
aggressive treated right away!
- Tx:
Less than 2mm nodular lesions: curettage,
Table 44: Comparison between hemangioma, vascular malformation, electrodesiccation, or laser vaporization.
port wine stain and glomus tumors.
If located at cheek, nose, or lip: Mohs' surgery
Hemangioma Vascular Capillary Glomus
malformat malformat tumors
e. Soft tissue tumors ion ion (port
Table 45. Comparison of lipoma, dermatofibroma & wine
achrochordon stain)
Lipoma Dermatofibroma Achrochordo -most common - vascular -flat, dull- -benign
n (skin tags) cutaneous lesion of malformati red lesion -located at
- most common - solitary, soft-tissue - fleshy, infancy ons are a often the
subcutaneous nodules measuring 1 to pedunculated -benign lesion that result of located on extremities
neoplasm 2 cm masses present soon after structural the - arise from
- soft and fleshy -usual location: legs and -usual birth (not at birth!) abnormalit trigeminal dermal
on palpation flanks location: -histopath: mitotically ies formed (CN V) neuromyoa
--usual location: -histopath: preauricular active endothelial cells during distributio rterial
back unencapsulated areas, axillae, surrounding several, fetal n on the apparatus
-histopath: connective tissue whorls trunk, and confluent blood-filled developm face, trunk, (glomus
lobulated tumor containing fibroblasts eyelids spaces ent or bodies).
composed of - do biopsy for atypical -tx: tying-off -enlarge at 1st year of -histopath: extremities - usually
normal fat cells presentation or resection life 90% eventually enlarged ; presents
-tx: excision -tx: excision involute vascular associated with severe
-tx: if it interferes with spaces with pain, point
f. Neural tumors airway, vision, and lined by sturge- tenderness
- Benign feeding or results to nonprolifer weber and cold
- Arise from the nerve sheath systemic problems ating symdrome sensitivity
(thrombocytopenia or endotheliu s -tx: tumor
high-output cardiac m. (leptomeni excision
Table 46. Comparison of neurofibroma, neurilemoma &
failure) resection - grow in ngeal
granular cell tumor
and proportion angiomato
Neurofibroma Neurilemoma Granular cell
prednisone/interferon to the body sis,
tumor
alpha 2 (for rapidly and never epilepsy,
-sporadic, solitary - solitary tumors - solitary lesions enlarging lesions) involute and
-can be arising from cells of the skin or, glaucoma)
syndromic (von of the peripheral more
Recklingh - nerve sheath commonly, the
ausen's disease: tongue Large tumors, those that invade surrounding
caf au lait spots, structures, & aggressive histologic types
Lisch nodules, (morpheaform, infiltrative, and
and an basosquamous): surgical excision with 0.5-cm
autosomal to 1-cm margins.
dominant
- Syndromic skin malignancies associated with BCC:
inheritance)
basal cell nevus (Gorlin's) syndrome:

TOPNOTCH MEDICAL BOARD PREP SURGERY SUPPLEMENT HANDOUT Page 35 of 85


For inquiries visit www.topnotchboardprep.com.ph or email us at topnotchmedicalboardprep@gmail.com
TOPNOTCH MEDICAL BOARD PREP SURGERY SUPPLEMENT HANDOUT - Jules Lopez,MD-MBA,Teddy
Carpio,MD-MBA
For inquiries visit www.topnotchboardprep.com.ph or email us at topnotchmedicalboardprep@gmail.com
autosomal dominant disorder characterized Malignant Melanoma
by the growth of hundreds of BCCs during - Arise from melanocytes
young adulthood. - Premalignant lesion: dysplastic nevi (vs freckles -
Palmar and plantar pits: common physical benign melanocytic neoplasms found on the skin)
finding - Most common location: skin (>90%); other sites: anus,
Tx: excision of aggressive and symptomatic eyes
lesions - 4%: discovered as metastases without any identifiable
nevus sebaceus of Jadassohn: primary site.
lesion containing several cutaneous tissue - Suspicious features: pigmented lesion with an
elements that develops during childhood. irregular border, darkening coloration, ulceration,
raised surface and recent changes in nevus
Squamous cell carcinoma appearance
- Arise from epidermal keratinocytes - Risk factors:
- Less common than BCC increased sun exposure of fair skinned people to
- Highly invasive and tends to metastasize solar radiation
- Tend to occur in persons with blond hair, light, thin, dry Familial dysplastic nevus syndrome
and irritated skin. autosomal dominant disorder
- In situ lesions: Bowens disease; if in the penis - Subtypes:
erythroplasia of Queyrat
- Risk factors: Table 47. comparison of malignant melanoma subtypes:
Skin lesions: actinic keratosis, atrophic
Superficial Nodular Lentigo Acral lentiginous
dermatitis spreading maligna
Occupational exposure: arsenics, nitrates and - most common -15 to 30% -4 to -2 to 8% of
hydrocarbons type (70% of of 15% of melanomas (least
Syndromic malignancies associated with SCC: melanomas) melanoma melano common)
epidermolysis bullosus - location: s mas -occurs at palms,
lupus erythematosus anywhere on the - darker - occur soles, and subungual
Epidermodysplasia verruciformis skin except the coloration on neck, regions
rare autosomal recessive disease hands and feet and often face, - Hutchinson's sign:
- flat and raised and presence of
associated with infection with HPV measure 1 to 2 -lack hands of pigmentation in the
Xeroderma pigmentosum cm in diameter radial elderly proximal or lateral
autosomal recessive disease associated at diagnosis growth -best nail folds; diagnostic
with a defect in cellular repair of DNA - Before vertical - progno of subungual
damage. extension, a aggressive sis melanoma
- tumor thickness correlates well with malignant prolonged but same -dark skinned +
behavior. radial growth prognosis acral lentiginous
phase is with melanoma:
more than 4 mm: Tumor recurrence is more
characteristic of superficial increased risk of
prevalent these lesions spreading malignancy
if 10 mm or more: these lesions usually have
associated metastasis -
Prognostic indicators:
- Burn scars (Marjolin's ulcer), areas of chronic
Location: lesions of the extremities have a better
osteomyelitis, and areas of previous injury tend to
prognosis than patients with melanomas of the
metastasize early.
head, neck, or trunk
- Tx:
(+) ulceration (due to increased angiogenesis):
Excision with 1 cm margin + histologic
worse prognosis
confirmation of tumor free borders
Gender: females have higher survival rates than
If located at cheek, nose, or lip: Mohs' surgery
men
Regional LN excision is indicated for clinically
palpable nodes Tumor types:
Best: lentigo maligna
If SCC arises from chronic wounds,
Worse: acral lentiginous
lymphadenectomy before development of palpable
- Staging from AJCC: breslow and clark level
nodes (prophylactic LN dissection) is indicated
Breslow thickness: the vertical thickness of the
because it is more aggressive and lymph node
primary tumor (from the granular layer of the
metastases are observed more frequently
epidermis or base of ulcer to the greatest depth of
- Metastatic disease is a poor prognostic sign (13%
the tumor); most important prognostic variable
survival after 10 years). predicting survival in those with cutaneous
melanoma; considered to be more precise in
MUST KNOW a predicting biologic behavior
I: 0.75 mm or less
Keratoacanthoma, which is characterized by rapid II: 0.76 to 1.5 mm
growth, rolled edges and a crater filled with keratin, can III: 1.51 to 4.0 mm
be confused with SCC or BCC. It spontaneously involutes IV: 4.0 mm or more
over time. Biopsy lesion to rule out carcinoma. Clark level: anatomic depth of invasion
I: superficial to basement membrane (in situ)
II: papillary dermis
Mohs surgery for BCC and SCC (nice to know!) III: papillary/reticular dermal junction
- This precise, specialized surgical technique uses IV: reticular dermis
minimal tissue resection and immediate microscopic V: subcutaneous fat
analysis to confirm appropriate resection yet limit - LN status & Metastasis:
removal of valuable anatomy. Evidence of tumor in regional LNs is a poor
- Done for aesthetic purposes prognostic sign (automatic stage III)
- uses serial excision in small increments coupled with Identification of distant metastasis is the worst
immediate microscopic analysis to ensure tumor prognostic sign ( automatic stage IV)
removal all specimen margins are evaluated. - Dx: excisional biopsy
Recurrence and metastases rates are comparable to - Tx:
those of wide local excision. Melanoma in situ/lentigo maligna melanoma in
face: 0.5 cm margins
TOPNOTCH MEDICAL BOARD PREP SURGERY SUPPLEMENT HANDOUT Page 36 of 85
For inquiries visit www.topnotchboardprep.com.ph or email us at topnotchmedicalboardprep@gmail.com
TOPNOTCH MEDICAL BOARD PREP SURGERY SUPPLEMENT HANDOUT - Jules Lopez,MD-MBA,Teddy
Carpio,MD-MBA
For inquiries visit www.topnotchboardprep.com.ph or email us at topnotchmedicalboardprep@gmail.com
1mm or less: excision with 1 cm margin cold. Meissners corpuscles occur in the papillae of
1 to 4 mm: excision with 2 cm margin the corium of the hands, the feet, the skin of the lips
More than 4 mm or (+) satellosis: 3-5 cm margin and other areas concerned with tactile sensation.
High dose interferon has a role in high risk Autonomic fibers that synapse to sweat glands and
melanoma receptors in the vasculature govern thermoregulation.
LN dissection:
Sentinel LN biopsy: 1mm or thicker with 2. Select the treatment options(s) in the 1st set of choices
clinically negative nodes or 0.75 mm thick + (UPPER CASE) that is/are most appropriate for the
clark level IV or ulcerated melanoma case summaries outlined in the 2nd set of
Radical regional lymphadenectomy: choices (lower case)
clinically (+) nodes with no evidence of distant
disease on metastatic work up. A. Level III superficial spreading melanoma (0.4
mm thick with clinically negative regional lymph
QUICK REVIEW a nodes
Remember: B. Level IV nodular melanoma (2mm thick) with
Mohs surgery is not appropriate for any type of satellosis and clinically negative regional lymph
melanoma nodes
C. Level IV superficial spreading melanoma (1.5
If melanoma is 4mm or greater + clinically negative
mm thick) with palpable regional lymph nodes
nodes perform metastatic work up first
D. Level IV acral lentiginous melanoma (2 mm
thick) with clinically negative regional lymph
nodes
Merkel cell carcinoma
- Primary Neuroendocrine Carcinoma of the Skin E. Level II lentigo maligna melanoma (0.3 mm)
- associated with a synchronous or metasynchronous SCC
25% of the time.
f. Mohs micrographic surgery
- Tx: wide local resection with 3-cm margins +
g. Wide local excision with 0.5 cm margins
Prophylactic regional LN dissection + adjuvant radiation
h. Wise local excision with 1.0 cm margins
therapy are recommended.
i. Wide local excision with 2.0 cm margins
- Prognosis: worse than malignant melanoma
j. Wide local excision with 4.0 cm margins
k. Sentinel lymph node biopsy
Kaposis sarcoma
l. Regional lymph node biopsy
- rubbery bluish nodules that occur primarily on the
extremities (also skin and viscera) m. Radical regional lymphadenectomy
- usually multifocal rather than metastatic.
Answer: A c; B e,f; C d,h; D d,f; E b
- Histopath: capillaries lined by atypical endothelial cells.
Virtually all melanomas are best treated by wide
- seen in people of Eastern Europe or sub-Saharan Africa,
excision. The excision margin that minimizes the risk
AIDS or immunosuppression with chemotx
of local recurrence depends on the thickness of the
- locally aggressive but undergo periods of remission
tumor. Melanoma in situ and thin lentigo maligna
melanomas of the face are treated adequately by
margins of 0.5 cm. For melanomas less than 1.0 mm
Extramammary Pagets disease
thick, 1 cm excision margins are appropriate. For
- cutaneous lesion that appears as a pruritic red patch
intermediate thickness melanoma (1-4 mm), a 2 cm
that does not resolve
margin is sufficient. Margins of 3-5 cm are generally
- histologically similar to the mammary type.
employed for melanomas 4 mm in thickness and for
those with associated satellosis. Mohs chemosurgery
is not appropriate for the treatment of any
REVIEW QUESTIONS
melanomas. The indications for elective lymph node
dissection remain controversial. Sentinel lymph node
1. Match the item in the left hand column with the
biopsy is indicated for aptients with melanoma 1 mm
appropriate item in the right hand column
or thicker with clinically negative nodes. The
indication is extended to patients with 0.75 mm thick
a. modulate cold sensation a. Ruffinis
melanomas if they are Clarks level IV or ulcerated.
endings
Patients with clinically positive lymph nodes with no
b. modulate sensitivity to warmth b. Krause end-
bulb evidence of distant disease on metastatic workup (CT
c. modulate sensation of pressure c. Meissners of chest, abdomen and pelvis; MRI of brain; PET)
corpuscles should undergo radical regional lymphadenectomy.
d. modulate tactile sensation d. Pacinian Patients with primary tumors 4 mm or greater with
clinically negative nodes should undergo metastatic
corpuscles
workup before undergoing sentinel node biopsy and
e. modulate thermoregulation e. autonomic
wide local excision.
nerve endings
3. With regards to keloids and hypertrophic scars, which
Answer: A-b; B-a; C-d; D-c; E-e
of the following statements is/are true?
A variety of highly specialized structures are
responsible for modulating the skins various sensory
A. There are no histologic differences between the
functions. The numbers of these structures vary with
two
the region of the body. Pacinian corpuscles are found
B. The differences between hypertrophic scar and
in the subcutaneous tissue, in the nerves of the palm of
the hand and the sole of the foot, and in other areas. keloid are clinical, not pathologic
Each of these corpuscles is attached to and encloses C. Hypertrophic scars outgrow their original
the termination of a single nerve fiber. They are borders
involved in the sensation of pressure. Ruffinis D. Hypertrophic scars and keloids have been treated
successfully with intralesional injection of
endings are a variety of nerve endings in the
steroids
subcutaneous tissue of the fingers and modulate
E. Keloids are seen in dark-skinned individuals,
sensitivity to warmth. Krauses end-bulbs are formed
whereas hypertrophic scars are seen in fair-
by the expansion of the connective tissue sheath of
medullated fibers and are involved in the sensation of skinned individuals

TOPNOTCH MEDICAL BOARD PREP SURGERY SUPPLEMENT HANDOUT Page 37 of 85


For inquiries visit www.topnotchboardprep.com.ph or email us at topnotchmedicalboardprep@gmail.com
TOPNOTCH MEDICAL BOARD PREP SURGERY SUPPLEMENT HANDOUT - Jules Lopez,MD-MBA,Teddy
Carpio,MD-MBA
For inquiries visit www.topnotchboardprep.com.ph or email us at topnotchmedicalboardprep@gmail.com
Answer: A,B,D -branches from axillary artery -tributaries of the axillary
Histologically, keloids and hypertrophic scars appear (highest thoracic, lateral vein.
the same. Hypertrophic scars are thick, red, raised thoracic, and pectoral branches - Batson's vertebral venous
of the thoracoacromial artery plexus : possible route for
scars that do not outgrow their original borders,
breast cancer metastases to
whereas keloids do. Keloids are dense accumulations the vertebrae, skull, pelvic
of fibrous tissue that form at the surface of the skin. bones, and central nervous
The defect appears to result from a failure in collagen system.
breakdown rather than an increase in its production.
Keloids and hypertrophic scars have been successfully - Innervation:
treated with intralesional steroid injectin, radiation, Sensory innervation to breast & anterolateral chest
pressure and the use of silicone gel sheets. wall: Lateral cutaneous branches of the 3rd 6th
intercostal nerves (slips out in between serratus
anterior muscles)
BREAST Intercostobrachial nerve: lateral cutaneous
branch of the second intercostal nerve; injury
A. Embryology of the breast to this nerve results to loss of sensation over
B. functional anatomy of the breast the medial aspect of the upper arm.
C. Physiology of breast Cutaneous branches from cervical plexus (anterior
D. infectious and inflammatory disorders of the breast branches of the supraclavicular nerve): supply a
E. common benign disorders and diseases of the limited area of skin over the upper portion of the
breast breast.
F. breast cancer - Lymphatics:
G. special clinical situations 6 axillary lymph node groups:

Table 49. Location and drainage pattern of breast


Name location drainage
A. EMBRYOLOGY OF THE BREAST Lateral medial or upper extremity; receives
- 5th or 6th week of fetal development mammary ridges (axillary posterior to the 75% drainage of the
vein vein breast; most common site
(thickened ectoderm): precursors of breast group) of axillary LN metastasis
Extends from the base of the forelimb (future level I
axilla) to the hind limb (future inguinal region) Anterior or lower border of lateral aspect of the breast
- Witchs milk: (+) breast secretions in an infant pectoral the pectoralis
secondary to maternal hormones that crosses the (external minor muscle
placenta mammary contiguous with
- Anomalies in embryology: group) the lateral
Polymastia: accessory breast; can be seen in level I thoracic vessels
Posterior posterior wall of lower posterior neck, the
Turners syndrome (ovarian agenesis and
of the axilla at the posterior trunk, and the
dysgenesis)and Fleischers syndrome subscapula lateral border of posterior shoulder
(displacement of the nipples and bilateral renal r (scapular the scapula
hypoplasia); can enlarge during pregnancy & group) contiguous with
lactation level I the subscapular
Polyethelia: accessory nipples; maybe associated vessels
with CVS and urinary tract anomalies Central embedded in receive lymph drainage both
Amastia: congenital absence of breast due to group the fat of the from the axillary vein,
arrest in mammary ridge development during the level II axilla lying external mammary, and
immediately scapular groups of lymph
6th week
posterior to the nodes, and directly from the
Polands syndrome: hypoplasia or complete pectoralis minor breast
absence of the breast, costal cartilage and rib muscle
defects, hypoplasia of the subcutaneous Apical posterior and from all of the other groups
tissues of the chest wall, and brachysyndactyly (subclavicu superior to the of axillary lymph nodes
Symmastia: webbing between the breasts across lar group) upper border of
the midline (no cleavage) level III the pectoralis
Inverted nipple: occurs in 4% of infants minor muscle
Interpector interposed receive lymph drainage
al group between the directly from the breast. The
B. FUNCTIONAL ANATOMY OF THE BREAST (Rotters pectoralis major lymph fluid that passes
- It extends from the level of the 2nd or 3rd rib to the nodes)- and pectoralis through the interpectoral
inframammary fold at the 6th or 7th rib. level II minor muscles group of lymph nodes
- It extends transversely from the lateral border of the passes directly into the
sternum to the anterior axillary line. central and subclavicular
- The deep or posterior surface of the breast rests on the groups.
fascia of the pectoralis major, serratus anterior, external
oblique abdominal muscles, & the upper extent of the Figure 28. Axillary lymph node groups
rectus sheath.
- retromammary bursa: located at posterior aspect of
the breast between the investing fascia of the breast and
the fascia of the pectoralis major muscles.
- The axillary tail of Spence extends laterally across the
anterior axillary fold.
- upper outer quadrant: greatest volume; most
common site of breast cancer
- Blood supply:

Table 48. Blood supply of the breast


Arterial blood supply Venous blood supply
- perforating branches of the -perforating branches of the
internal mammary artery internal thoracic vein
- lateral branches of the -perforating branches of the
posterior intercostal arteries posterior intercostal veins
TOPNOTCH MEDICAL BOARD PREP SURGERY SUPPLEMENT HANDOUT Page 38 of 85
For inquiries visit www.topnotchboardprep.com.ph or email us at topnotchmedicalboardprep@gmail.com
TOPNOTCH MEDICAL BOARD PREP SURGERY SUPPLEMENT HANDOUT - Jules Lopez,MD-MBA,Teddy
Carpio,MD-MBA
For inquiries visit www.topnotchboardprep.com.ph or email us at topnotchmedicalboardprep@gmail.com
e. Hidradenitis suppurativa
Reference point: - can also occur in the nipple-areola complex
pectoralis minor - originates within the Montgomery glands or axillary
(not major!)
sebaceous glands.
Level I: lateral to
the pectoralis - Risk factor: chronic acne
minor muscle (PM) - may mimic Paget's disease of the nipple or invasive
Level II: deep to breast cancer.
PM - Tx: Antibiotic + I&D
Level III: medial to
the PM. f. Mondors disease
- a benign self limited condition which is a variant of
Arrows indicate the
thrombophlebitis that involves the superficial veins of
direction of lymph
flow. the anterior chest wall and breast.
Also seen: axillary - Involved veins: lateral thoracic vein, the
vein & its major thoracoepigastric vein, and, less commonly, the
tributaries, superficial epigastric vein.
supraclavicular LN - SSx: acute pain in the lateral aspect of the breast or the
anterior chest wall with palpation of a tender, firm cord
C. PHYSIOLOGY OF BREAST along the distribution of the major superficial veins.
- Breast development and function - Tx: anti-inflammatory medications + warm compresses
Hormonal stimuli: along the symptomatic vein + Restriction of motion of
Estrogen: ductal development the ipsilateral extremity and shoulder + brassiere
Progesterone: differentiation of epithelium & support of the breast are important (4 to 6 weeks) or
lobular development excision of vein (if not improving)
Prolactin: 1o hormonal stimulus for
lactogenesis in late pregnancy & the E. COMMON BENIGN DISORDERS AND DISEASES OF
postpartum period. THE BREAST
- Gynecomastia: enlarged breast in males measuring at a. Fibroadenoma
least 2 cm in diameter - seen predominantly in younger women aged 15 to 25
- Gynecomastia generally does not predispose the male years
breast to cancer unless syndromic - can be self limiting
- Physiologic gynecomastia occurs due to excess in - if greater than 3cm consider giant fibroadenoma
circulating estrogens (in relation to circulating - if multiple (more than 5 lesions in 1 breast)
testosterone): considered as abnormal
neonatal period: action of placental estrogens on - tx: cryoablation, surgical removal or observation
neonatal breast tissues
adolescence: excess of estradiol relative to b. Cyclical mastalgia and nodularity
testosterone; can be unilateral - associated with premenstrual enlargement of the breast
senescence: circulating testosterone level falls; - physiologic.
usually bilateral - If Painful nodularity persists for >1 week of the
menstrual cycle consider a disorder.
D. INFECTIOUS AND INFLAMMATORY DISORDERS OF - bilateral bloody nipple discharge can be seen in
THE BREAST epithelial hyperplasia of pregnancy
a. Breast abscess c. Breast cysts
- Staphylococcus aureus (more localized & deep) and - occurs when the stroma involutes too quickly, and
Streptococcus (diffuse superficial involvement) species: alveoli remain forming microcysts & macrocysts
causative organisms - characteristics of benign lesions: sharp, smooth
- SSx: point tenderness, erythema, and hyperthermia. margins, a homogenous interior and posterior
- Risk factor: lactation (because a lactating breast is an enhancement (vs malignancy which will show irregular
excellent culture medium) and jagged margins, heterogenous interior and posterior
- Tx: preoperative UTZ + incision & drainage (if already shawoding)
with suppuration) + local wound care (warm - management: needle biopsy ( 1st line investigation for
compresses &IV antibiotics - penicillins or palpable breast masses)
cephalosporins). if (+) fluid on aspiration aspirate to dryness, no
Remember: Biopsy of the abscess cavity wall is need to do cytologic examination
recommended at the time of I&D rule out breast If after aspiration, (+) residual mass do UTZ
cancer with necrotic tumor. guided needle biopsy
- Chronic breast abscesses: consider acid-fast bacilli, If blood stained fluid aspirate 2 mL for cytologic
anaerobic and aerobic bacteria, and fungi. examination, utz imaging and biopsy solid areas
If fungal. Consider blastomycosis or sporotrichosis - If complex cyst rule out malignancy.
(rare)
d. Calcium deposits
b. Epidemic puerperal mastitis - benign
- MRSA: causative organism - cause: cellular secretions , debris or by trauma and
- Transmission via suckling neonate inflammation.
- Tx: stop breastfeeding , antibiotics & I&D - (+) cancer if <0.5 mm in size, fine, linear calcifications,
may branch (microcalcifications).
c. Nonepidemic (sporadic) puerperal mastitis
- involvement of the interlobular CT of the breast e. Sclerosing adenosis
- tx: Emptying of the breast using breast suction pumps + - Common in childbearing and perimenopausal years
antibiotics - no malignant potential.
- characterized by distorted breast lobules + multiple
d. Zuska's disease (recurrent periductal mastitis) microcysts + benign calcifications
- recurrent retroareolar infections and abscesses.
- Risk factor: smoking f. Radial scars (1 cm or less) or Complex central
- Tx: antibiotics + I&D as necessary. sclerosis (more than 1 cm)

TOPNOTCH MEDICAL BOARD PREP SURGERY SUPPLEMENT HANDOUT Page 39 of 85


For inquiries visit www.topnotchboardprep.com.ph or email us at topnotchmedicalboardprep@gmail.com
TOPNOTCH MEDICAL BOARD PREP SURGERY SUPPLEMENT HANDOUT - Jules Lopez,MD-MBA,Teddy
Carpio,MD-MBA
For inquiries visit www.topnotchboardprep.com.ph or email us at topnotchmedicalboardprep@gmail.com
- characterized by central sclerosis, epithelial routine screening mammography starting 50
proliferation, apocrine metaplasia, and papilloma years old age reduces mortality from breast
formation cancer by 33%
- can mimic cancer hence an excisional biopsy is done to baseline mammography at age 35
to exclude diagnosis of cancer annual mammographic screening
beginning at age 40.
g. Ductal hyperplasia If (+) family history for breast cancer
- Severity: Baseline mammogram 10 years before
Mild: 3-4 cell layers above the basement the youngest age of diagnosis of breast ca
membrane. among 1st degree relatives. (this rule is
Moderate:5 or more cell layers above the basement modified if age of diagnosis is less than
membrane. 35)
Florid ductal epithelial hyperplasia: occupies at
least 70% of a minor duct lumen. Chemoprevention
- associated with an increased cancer risk Tamoxifen: selective estrogen receptor
modulator
h. Intraductal papillomas recommended only for women who have
- Seen in premenopausal women. a Gail relative risk of 1.70 or er.
- common symptom: serous or bloody nipple SE: deep vein thrombosis, pulmonary
discharge emboli, endometrial cancer
- Gross appearance: pinkish tan, friable, reduce the incidence of LCIS and ductal
- rarely undergo malignant transformation & no carcinoma in situ (DCIS)
increased risk of breast cancer, unless multiple Raloxifene: estrogen receptor modulator
Equivalent to tamoxifen
i. Atypical proliferative disease associated with a more favorable
- has some of the features of Ca in situ but lack a major adverse event profile
defining feature of Ca in situ or have the features in less no effect on LCIS or DCIS
than fully developed form
- Atypical ductal hyperplasia & lobar hyperplasia prophylactic mastectomy
Increases risk of breast cancer 4x; if with (+) greatly reduces risk for breast cancer
family hx, 10x only for high risk populations
+ 3 years if with 40% risk of having breast
F. BREAST CANCER Ca
- risk factors + 5 years if with 85% risk of having breast
increased exposure to estrogen: early menarche, cancer
nulliparity, late menopause, older age at first live
birth (after the age of 30 yo), HRT, obesity, (major - BRCA mutation
source of estrogen in postmenopausal women is Constitutes 5-10% of breast cancers
the conversion of androstenedione to estrone by Autosomal dominant inheritance
adipose tissue) tumor-suppressor genes
radiation exposure: patients with multiple prevalent in Ashkenazi Jews
fluoroscopies, mantle radiation for treatment of
hodgkins lymphoma Table 50. Comparison of BRCA 1 & 2
increased alcohol intake (leads to increased BRCA 1 BRCA 2
estradiol levels) - location: ch arm 17q -location: ch arm 13q
high fat diet (increased serum estrogen levels) -predisposing genetic - lifetime risk for carrier
factor: 45% of breast Ca & Ca: 85% for breast ca &
prolonged use of OCPs (particularly estrogen- 85% of ovarian Ca 20% for ovarian ca ; if male
plus-progesterone) and HRT -lifetime risk for carrier carrier: 6%
(+) family history of breast cancer: the greater Ca: 90% for breast ca & -usually develops invasive
the number of relatives affected, the closer the 40% for ovarian ca ductal carcinomas: well
genetic relationship, the younger the age at - usually develops invasive differentiated, hormone
diagnosis, and the presence of bilateral versus ductal carcinomas: poorly receptors (+)
unilateral disease all increased the likelihood of differentiated and hormone -early age of onset, bilateral
development of breast cancer in an individual. receptor (-) breast cancer, & other
-early age of onset, bilateral associated ca: ovarian,
breast cancer and other colon, prostate, pancreatic,
MUST KNOW associated ca: ovarian, gallbladder, bile duct &
Remember: colon and prostate stomach cancers,
melanoma.
Smoking is not considered a risk factor for breast
cancer!!!!!!!!!!! Please dont make the mistake of answering Risk mgt strategies for BRCA carriers:
this as part of the risk factors in breast cancer. Prophylactic mastectomy and reconstruction
Prophylactic oophorectomy (because of risk
of ovarian ca) at the completion of
- risk management childbreaing or manopause + HRT
Postmenopausal hormone replacement tx Intensive surveillance for breast and ovarian
Widely prescribed because it is effective in cancer
controlling symptoms of estrogen deficiency Chemoprevention
(vasomotor symptoms such as hot flashes, BRCA mutation carriers who do not undergo
night sweats and their associated sleep prophylactic mastectomy must do clinical breast
deprivation, osteoporosis, and cognitive examination every 6 months and mammography
changes) every 12 months beginning at age 25 years
Results of Womens health initiative study (because the risk of breast cancer in BRCA
(2002): breast Ca risk is increased to 3- mutation carriers increases after age 30 years)
4fold after > 4 years of use + no reduction
in CAD or CVD - HER-2 gene
Encodes transmembrane tyrosine kinase, a protein
screening mammogram with potent growth stimulating activity

TOPNOTCH MEDICAL BOARD PREP SURGERY SUPPLEMENT HANDOUT Page 40 of 85


For inquiries visit www.topnotchboardprep.com.ph or email us at topnotchmedicalboardprep@gmail.com
TOPNOTCH MEDICAL BOARD PREP SURGERY SUPPLEMENT HANDOUT - Jules Lopez,MD-MBA,Teddy
Carpio,MD-MBA
For inquiries visit www.topnotchboardprep.com.ph or email us at topnotchmedicalboardprep@gmail.com
In breast cancer, this gene is amplified (indicating LCIS DCIS
more rapid growth & aggressive behavior) Interval to 15-20 y 5-10 y
Treatment if there is a mutation: Herceptin diagnosis
Histo type Ductal; cytoplasmic ductal
- Breast cancer signs and symptoms: mucoid globules are
characteristic
mass (most common)
Specifics Only in breast; more Other name:
if size is 1 cm mass has been present for 5
common in whites; not intraductal
years considered an carcinoma (true
breast enlargement or asymmetry anatomic precursor anatomic
nipple changes - retraction, or discharge ( due to of breast ca; only a precursor); 5%
shortening of Cooper's suspensory ligament) risk marker of male cancers
skin dimpling (increases risk of
ulceration / erythema of the skin breast ca 9x)
axillary mass or mets
firm or hard with continued growth of the Tx:
metastatic cancer. LCIS: observation, chemoprevention
involved sequentially from the low (level I) to with tamoxifen, and bilateral total
the central (level II) to the apical (level III) mastectomy or may opt to do close follow
lymph node groups. up + periodic PE + bilateral mammograms
axillary lymph node status: most important for a more conservative approach
prognostic correlate of disease-free and DCIS:
overall survival o > 4 cm or disease in >1 quadrant:
peau d'orange (Localized edema): blocked mastectomy
drainage of lymph fluid o Low-grade DCIS of the solid,
musculoskeletal discomfort. cribriform, or papillary subtype that
Distant metastases: most common cause of death is <0.5 cm: lumpectomy (If margins
in breast cancer patients are free of disease)
Due to neovascularization (hematogenous o Adjuvant tamoxifen therapy has a
spread) cancer cells shed directly to role for DCIS pt.
axillary and intercostals veins or vertebral
column via batsons plexus of veins. MUST KNOW a
Metastatic foci occurs after the 1o ca exceeds Remember:
0.5 cm in diameter Mastectomy vs lumpectomy + adjuvant RT: same
Common sites of involvement (in order of mortality rate (<2%) but lumpectomy + adjuvant RT
frequency): bone, lung, pleura, soft tissues, and has a higher local recurrence rate (up to 9%, compared
liver to 2% for mastectomy)
Role of RT: markedly decreases the risk of in-breast
MUST KNOW a recurrence and significantly reduces the risk that any
Remember: recurrence will be invasive disease
Breast pain is usually associated with benign disease. High recurrence rate for DCIS comedo type

- In situ breast cancer - Invasive Breast Cancer


Multicentricity: occurrence of a second Paget's disease of the nipple (unrelated to Pagets
breast cancer outside the breast quadrant of disease of the bone)
the primary cancer (or at least 4 cm away) chronic, eryhthematous, eczemamatoid rash
Multifocality: the occurrence of a second or ulcer
cancer within the same breast quadrant as the associated with DCIS & invasive cancer.
primary cancer (or within 4 cm of it) Pathognomonic sign: large, pale,
Difficult to differentiate from atypical vacuolated cells (Paget cells) in the rete
hyperplasia or cancers with early invasion pegs of the epithelium.
Subtypes: Rule out superficial spreading melanoma
(+) s-100 antigen in immunostaining (vs
Table 51. Comparison of LCIS vs DCIS
LCIS DCIS
pagets disease which is (+) in
Age 44-47 54-58 carcinoembryonic antigen
Incidence 2-5 5-10 immunostaining)
Clinical sx None Mass, pain & Tx: lumpectomy, mastectomy, or MRM
nipple discharge (depending on the extent of involvement and
Mammographic None / Microcalcification the presence of invasive cancer)
sx mammographically s (usually in areas Invasive ductal carcinoma
featureless; may have of necrosis) Occurs in perimenopausal or postmenopausal
calcifications in (5th-6th decade)
adjacent tissues
Most common carcinoma presenting as a
occasionally
breast mass
Premenopausal 2/3 (more common) 1/3
poorly defined margin, central stellate
Incidence of 5% 2-46% configuration with chalky white or yellow
synchronous streaks extending into surrounding breast
invasive ca tissues
Multicentricity 60-90 % 40-80% (more macroscopic/microscopic axillary LN
common if metastases in 60% of cases
comedo type)
Bilaterality 50-70% (more 10-20% Medullary carcinoma
common)
Associated with BRCA1phenotype & DCIS
Axillary 1% 1-2%
metastasis Gross appearance: well circumscribed, soft &
Subsequent carcinoma hemorrhagic (when accompanied with a rapid
Laterality Bilateral Ipsilateral increase in size)

TOPNOTCH MEDICAL BOARD PREP SURGERY SUPPLEMENT HANDOUT Page 41 of 85


For inquiries visit www.topnotchboardprep.com.ph or email us at topnotchmedicalboardprep@gmail.com
TOPNOTCH MEDICAL BOARD PREP SURGERY SUPPLEMENT HANDOUT - Jules Lopez,MD-MBA,Teddy
Carpio,MD-MBA
For inquiries visit www.topnotchboardprep.com.ph or email us at topnotchmedicalboardprep@gmail.com
PE: bulky and mass is positioned deep within Intraductal papillomas are seen as small filling
the breast. defects surrounded by contrast media
Can mimic a benign condition on diagnostic Ca: may appear as irregular masses or as
imaging (looks like a fibroadenoma on multiple intraluminal filling defects
UTZ)
Occurs bilaterally in 20% of cases. Ultrasonography
Microscopically: dense lymphoreticular Ideal for younger patients (because of
infiltrate tendency to have denser breasts can affect
results if mammography is used)
Mucinous (colloid) carcinoma Useful for resolving equivocal mammographic
Occurs in the elderly findings, defining cystic masses, and
Characteristic lesion: extracellular pools of demonstrating the echogenic qualities of
mucin specific solid abnormalities.
Gross appearance: glistening & gelatinous with breast cysts: well circumscribed, with smooth
a firm consistency margins and an echo-free center
features of benign breast masses: smooth
Papillary carcinoma contours, round or oval shapes, weak internal
Usually occurs in the 7th decade of life echoes, and well-defined margins.
More common in nonwhite . Features of breast ca: irregular walls but may
defined by papillae with fibrovascular have smooth margins with acoustic
stalks and multilayered epithelium. enhancement.
does not reliably detect lesions that are 1cm.
Tubular carcinoma
Usually occurs during perimenopausal or early - Breast cancer staging
menopausal periods. Clinically based
Microscopically: haphazard array of small, tumor size correlates with the presence of axillary
randomly arranged tubular elements is seen. lymph node metastases
Well-differentiated type of infiltrating ductal The single most important predictor of 10- and
cancer 20-year survival rates in breast cancer is the
Favorable diagnosis number of axillary lymph nodes involved with
metastatic disease.
Invasive lobular carcinoma
Histopath: small cells with rounded nuclei, Table 52. TNM breast cancer staging
inconspicuous nucleoli, and scant cytoplasm; T N M
(+) intracytoplasmic mucin, which may T1: <2cm N1: M1: (+) lung,
T2: 2-5 cm suspicious liver or bone
displace the nucleus (signet-ring cell
T3: >5cm mobile involvement
carcinoma). T4: (+) chest axillary
frequently multifocal, multicentric, and wall & direct nodes
bilateral. skin N2: matted
Hard to detect mammographically involvement or fixed
axillary
- Diagnosis of breast cancer nodes
Mammography: N3:
ipsilateral
Views:
internal
Craniocaudal (CC) view: medial aspect of mammary
the breast; permits greater breast nodes
compression
mediolateral oblique (MLO) view: images (+) supraclavicular nodes: stage III disease (not
the greatest volume of breast tissue & stage IV as formerly classified)
upper outer quadrant and the axillary tail
of Spence - Treatment for breast cancer
Features suggestive of breast cancer: Treatment is dependent on the stage at diagnosis
solid mass +/- stellate features Early invasive breast cancer (stage I, IIa, IIb)
asymmetric thickening of breast tissues Lumpectomy +/- RT (breast conservation sx)
clustered microcalcifications is an acceptable tx option since survival rates
presence of fine, stippled calcium in & are comparable to total mastectomy.
around a suspicious lesion is suggestive of However, recurrence er in the
breast cancer; occurs in 50% of lumpectomy with no RT stage I and II
nonpalpable cancers. breast cancer.
Mimickers of breast ca mammographically: CI to breast conservation sx:
radial scars, fibromatosis, granular cell prior RT to the breast or chest wall
tumor and fat necrosis (surgical excision is involved surgical margins or unknown
indicated for these lesions, owing to their margin status after re-excision
resemblance to ca) multicentric disease
% reduction in mortality for women after scleroderma or lupus erythematosus.
screening mammography. If clinically negative nodes but with T1-T2
Recommendations: primary ca perform sentinel LN
normal-risk women at 20 yo breast dissection
examination every 3 years If (+): perform axillary lymph node
at age 40 yo annual breast examination dissection should be performed.
/ mammography Adjuvant chemotherapy is indicated for node-
false (-)/(+) rate: 10% positive cancers, >1 cm, and node-negative
cancers of >0.5 cm when adverse prognostic
features (blood vessel or lymph vessel
Ductography invasion, high nuclear grade, high histologic
Indication: is nipple discharge, (particularly grade, HER-2/neu overexpression, and
when bloody) negative hormone receptor status).
TOPNOTCH MEDICAL BOARD PREP SURGERY SUPPLEMENT HANDOUT Page 42 of 85
For inquiries visit www.topnotchboardprep.com.ph or email us at topnotchmedicalboardprep@gmail.com
TOPNOTCH MEDICAL BOARD PREP SURGERY SUPPLEMENT HANDOUT - Jules Lopez,MD-MBA,Teddy
Carpio,MD-MBA
For inquiries visit www.topnotchboardprep.com.ph or email us at topnotchmedicalboardprep@gmail.com
Tamoxifen therapy: women with hormone o injury to the axillary vein
receptor (+) cancer that are >1 cm. sudden painful early postoperative
HER-2/neu expression is determined for node swelling of the involved arm (due to
(-) breast ca acute thrombosis as the collateral
Trastuzumab: medication for HER-2/neu channels do not have the chance to
() develop acute and painful)
doxorubicin, cyclophosphamide, & o injury to the thoracodorsal
paclitaxel: medication for HER-2/neu (+) vascular pedicle ischemic loss of
& node (+) breast cancer. the entire latissimus dorsi flap
utilized for reconstruction
Advanced local regional breast cancer (stage o injury to the medial pectoral
IIIa or IIIb) pedicle progressive atrophy of
Surgery (MRM) + adjuvant RT + CT the pectoralis muscle
(neoadjuvant) o injury to the 2nd intercostals
Role of CT: maximize distant disease-free brachiocutaneous nerve
survival hypesthesia of the upper inner
Role of RT: maximize local-regional aspect of the ipsilateral arm
disease-free survival. Halsted radical mastectomy: removes all
If stage IIIA ca: neoadjuvant breast tissue and skin, the nipple-areola
(preoperative) CT reduce the size of complex, the pectoralis major and pectoralis
the primary ca & permit breast- minor muscles & the level I, II, and III LN.
conserving surgery.
- Non surgical breast cancer tx
Figure 29. Treatment pathyway for stage IIIa & IIIb cancer RT
Adjuvant RT after mastectomy decrease
local recurrence rates but will not prolong
survival
o Indicated for those with high risk for
local recurrence: large tumors, skin
involvement, > 4 axillary LN involved
Chemotx
Adjuvant chemotheraphy
Indicated if node (-) tumor >1cm that are
ER (-)
Neoadjuvant chemotherapy
Neoadjuvant endocrine therapy
Tamoxifen: indicated if node (-) tumor
>1cm that are ER (+)
Distant metastases (stage IV) Herceptin: (+) her2/neu
Not anymore curative but may prolong
survival - breast cancer in pregnancy
occurs in 1 of every 3000 pregnant
- Breast cancer prognosis TX:
5 year survival rate MRM: 1st & 2nd trimesters of pregnancy
Stage I: 94% lumpectomy with axillary node dissection: 3rd
stage IIA: 85% trimester
stage IIB: 70% adjuvant RT: after delivery.
stage IIIA: 52%
stage IIIB: 48% - male breast cancer
stage IV: 18% <1% of all breast cancers occur in men
preceded by gynecomastia in 20% of men.
- surgical techniques in breast cancer tx associated with radiation, estrogen tx, testicular
sentinel LN dissection: used to assess the regional feminizing syndromes, and Klinefelter's syndrome
LN in women with early breast ca who are (XXY)
clinically node negative by PE & imaging studies usual types of cancer: DCIS, infiltrating ductal ca
breast conservation therapy (BCT) Overall, men do worse because of the advanced
if stage 0, I & II, BCT is preferable to total stage of their ca (stage III or IV) at the time of
mastectomy ( with equivalent survival rates) diagnosis and poorer prognosis. But stage for stage,
mastectomy and axillary dissection the results of treatment are similar to those in
simple mastectomy: removes all breast women.
tissue, the nipple-areola complex, skin & level I
LN G. special clinical situations
Modified radical mastectomy: removes all a. nipple discharge
breast tissue, the nipple-areola complex, skin, suggestive of cancerous lesion: spontaneous,
& level I and level II LN. unilateral, localized to a single duct, present in
preserves pectoralis major, pectoralis women 40 years of age, bloody, clear, serous, or
minor, level III LN & medial (anterior associated with a mass.
thoracic) pectoral nerve Suggestive of a benign condition: bilateral,
complications multiductal in origin, occurs in women 39 years of
o most frequent: Seromas beneath the age, or is milky or blue-green.
skin flaps or in the axilla Consider prolactin-secreting pituitary
o injury to the long thoracic nerve adenomas ( serum prolactin levels, Optical
(affects serratus anterior) nerve compression, visual field loss, &
winging of scapula infertility)
o lymphatic fibrosis painless, slow
progressive swelling of the involved b. Cystosarcoma phylloides tumor
arm - Resembles a giant fibroadenoma

TOPNOTCH MEDICAL BOARD PREP SURGERY SUPPLEMENT HANDOUT Page 43 of 85


For inquiries visit www.topnotchboardprep.com.ph or email us at topnotchmedicalboardprep@gmail.com
TOPNOTCH MEDICAL BOARD PREP SURGERY SUPPLEMENT HANDOUT - Jules Lopez,MD-MBA,Teddy
Carpio,MD-MBA
For inquiries visit www.topnotchboardprep.com.ph or email us at topnotchmedicalboardprep@gmail.com
- Can occur in benign and malignant forms e. Aromatase inhibitor
- gross appearance: classical leaf-like (phyllodes)
appearance; greater cellular activity than fibroadenoma Answer: A,B,E
- metastasis is usually vascular and no axillary LN
involvement is expected. Patients with high ER & PR levels (based on
- tx: immunohistomchemical stains) have better prognosis
if benign total excision with 2-3 cm margin compared to those with zero or low levels. The most
if malignant total mastectomy w/o axillary LN common hormonal manipulation is estrogen
dissection; if small wide excision with 2cm withdrawal, usually with a receptor-blocking agent
margin is acceptable (tamoxifen). However, bilateral oophorectomy in
if large mastectomy. premenopausal women is still considered a reasonable
Follow up is important due to high local recurrence option. Surgical hypophysectomy & adrenalectomy
rate were at one point considered forms of hormonal
manipulation, but are now being replaced by medical
c. Inflammatory breast cancer adrenalectomy in the form of anastrazole, which
- variant of infiltrating ductal ca inhibit the production of adrenal steroids and
- characterized by the skin changes of brawny conversion of androgens to estrogens in the adrenal
induration, erythema with a raised edge, and gland and peripherally. The aromatase inhibitors are
edema or peau d'orange (hence the name beneficial only in postmenopausal women.
inflammatory) + breast mass
- appearance is due to a dermal lymphatic 3. a 39 year old woman presents with an ill-defined 2 cm
invasion mass in the outer quadrant of her breast.
Mammography shows very dense tissue but no
discrete lesion. Ultrasound examination shows a solid
SURGICAL TECHNIQUES FOR BREAST SURGERY lesion. An ultrasound-guided fine needle aspiration
(FNA) is performed, and the aspirate is plated, fixed,
1. Skin-Sparing Mastectomy: removes ALL breast tissue, and sent to the laboratory for cytologic study. A highly
the nipple-areola complex and scars from any prior cellular monomorphic pattern is seen, with poorly
biopsy procedure cohesive intact cells, nuclear crowding with a
2. Total (Simple) Mastectomy: removes ALL breast tissue, variation in nuclear size, radial dispersion and
the nipple-areola complex and skin clumping of the chromatin, and prominent nucleoli.
3. Extended Simple Mastectomy: removes ALL breast Which of the following management choices is/are
tissue, the nipple-areola complex, skin and level I appropriate?
axillary lymph nodes
4. Modified Radical Mastectomy: removes ALL breast a. MRM
tissue, nipple-areola complex, skin and the level I and II b. Reassuring the patient that the process is
axillary lymph nodes benign
5. Halsted Radical Mastectomy: removes ALL breast c. Lumpectomy, sentinel lymph node biopsy
tissue and skin, the nipple-areola complex, the and irradiation
pectoralis major and pectoralis minor muscles and the d. Excision of a fibroadenoma with narrow
level I, II and III axillary lymph nodes margins
e. Lumpectomy and sentinel lymph node
biopsy without irradiation

Answer: A,C
REVIEW QUESTIONS a Aspiration biopsy with a 22 gauge needle is an effective
and safe way of assessing palpable breast lesions.
1. a 58 yo woman presents with chronic, erythematous, Performing the aspiration under ultrasound guidance
oozing, eczematoid rash involving the left nipple and ensures that the lesion has been sampled thoroughly
areola. There are no breast masses palpable, and her while under direct vision. Although a smaller volume
mammogram is normal. Which of the following of tissue is obtained than the core needle biopsy, FNA
recommendations is appropriate? frequently yields results that may be equal to core
biopsy if read by an experienced cytopathologist. A
a. Referral to a dermatologist fibroadenoma would show broad sheets of cohesive
b. Oral vitamin E and topical aloe and lanolin cells with nuclei that are unfirm in size and shape. The
c. Biopsy chromatin pattern would be finely granular and large
d. Non allergenic brassiere numbers of bare nuclei would be present. The cytologic
e. Standard treatment that includes breast findings described in this question is diagnostic of
conservation carcinoma. Appropriate management, therefore,
includes either a modified radical mastectomy or
Answer: C lumpectomy, axillary evaluation by either a sentinel
This is a case of Pagets disease of the breast. It is a lymph node biopsy or an axillary nodal dissection, and
case of primary ductal carcinoma that secondarily whole-breast irradiation.
invades the epithelium of the nipple and areola. Biopsy
of any chronic nipple rash is mandatory and will show
the distinctive pagetoid cells. Because of the possible
invasion of the tumor on the underlying rich lymphatics HEAD and NECK: BENIGN CONDITIONS & TUMORS
of the nipple areolar complex, mastectomy is usually
indicated. In selected cases, breast conservation A. Risk factors for tumors of head and neck
therapies can also be employed. B. Anatomy of Oral cavity
C. Cancer of the Lip
2. If patient with metastatic breast ca is ER (+), which of D. Cancer of the Tongue
the following statements are appropriate? E. Tumors of Alveolus/gingiva
F. Anatomy of pharynx
a. Bilateral oophorectomy G. Tumors of Nasopharynx
b. Antiestrogen drugs (tamoxifen) H. Tumors of Oropharynx
c. Hypophysectomy I. Tumors of Hypopharynx/cervical esophagus
d. Adrenalectomy
TOPNOTCH MEDICAL BOARD PREP SURGERY SUPPLEMENT HANDOUT Page 44 of 85
For inquiries visit www.topnotchboardprep.com.ph or email us at topnotchmedicalboardprep@gmail.com
TOPNOTCH MEDICAL BOARD PREP SURGERY SUPPLEMENT HANDOUT - Jules Lopez,MD-MBA,Teddy
Carpio,MD-MBA
For inquiries visit www.topnotchboardprep.com.ph or email us at topnotchmedicalboardprep@gmail.com
J. Anatomy Larynx Prophylactic supraomohyoid neck dissection
K. Benign conditions of the Larynx should be considered for patients with tumors
L. Laryngeal Carcinoma greater than 4 cm, desmoplastic tumor & (+)
M. Neck and associated conditions perineural invasion
N. Salivary gland tumors Realignment of the vermilion border during the
O. Thyroid and associated conditions reconstruction and preservation of the oral
commissure (when possible) are important
principles in attempting to attain an acceptable
cosmetic result.
- Prognosis is most favorable for all H&N CA
A. RISK FACTORS FOR TUMORS OF HEAD AND NECK
D. CANCER OF THE TONGUE
- tobacco & alcohol: most common preventable risk - muscular structure with overlying nonkeratinizing
factors associated with head and neck CA. squamous epithelium.
- betel nut chewing - Posterior border: circumvallate papillae
- reverse smoking - Tongue cancer
- HPV 16 and 18. Same risk factors with other H&N CA
- UV light exposure (for lip CA) Associated with plummer-vinson syndrome
- Patients with H&N CA are predisposed to the (cervical dysphagia, IDA, atrophic oral mucosa,
development of a 2nd tumor within the aerodigestive brittle spoon finger nails)
tract. Clinical findings: ulcerations or as exophytic
presentation of a new-onset dysphagia, masses
unexplained weight loss, or chronic The regional lymphatics of the oral cavity are to the
cough/hemoptysis must be assessed submandibular space and the upper cervical
thoroughly in patients with a history of prior lymph nodes
treatment for a head and neck cancer Involvement of lingual nerve ipsilateral
ex. If (+) primary malignancy of oral cavity paresthesias
orpharynx secondary malignancy at cervical Involvement of hypoglossal nerve deviation of
esophagus; (+) primary malignancy at larynx tongue on protusion + fasciculations atrophy
secondary malignancy at lungs most common location: lateral and ventral surfaces
- Synchronous neoplasm: a 2nd 1o tumor detected within if base of the tongue advanced stage and poorer
6 months of the diagnosis of the initial primary lesion prognosis
- Metachronous tumor: detection of a 2nd 1o lesion more tx:
than 6 months after the initial. Surgical treatment of small (T1T2) primary
- Initial evaluation of patients with primary CA of H&N: tumors is wide local excision with either
"panendoscopy." primary closure or healing by secondary
intention.
B. ANATOMY OF ORAL CAVITY If base of tonguePartial glossectomy with
- Borders: supraomohyoid dissection if N0 or MRND if
Anterior: vermilion border of the lip N(+)
Superior: hard-palate/soft-palate junction
Inferior: circumvallate papillae E. TUMORS OF ALVEOLUS/GINGIVAL
Lateral: anterior tonsillar pillars - Because of the tight attachment of the alveolar mucosa
- The oral cavity includes lips, alveolar ridges, oral to the mandibular and maxillary periosteum, treatment
tongue, retromolar trigone, floor of mouth, buccal of lesions of the alveolar mucosa frequently requires
mucosa, and hard palate. resection of the underlying bone.
- Regional metastatic spread of lesions of the oral cavity - Diagnosis for alveolar or gingival cancer
is to the lymphatics of the submandibular and the Panorex: demonstrate gross cortical invasion
upper jugular region (levels I, II, and III) CT: imaging subtle cortical invasion
- Majority of tumors in the oral cavity are squamous MRI: demonstrates invasion of the medullary
cell carcinoma (>90%) cavity
- Tx for alveolar or gingival cancer
C. CANCER OF THE LIP If minimal bone invasion: mandibular resection
- most commonly seen old people (50-70 years old) with If (+) medullary cavity invasion: segmental
fair complexion mandibulectomy
- Risk factors: prolonged exposure to sunlight, fair
complexion, immunosuppression, and tobacco use. F. ANATOMY OF PHARYNX
- Most common location: lower lip (88 to 98%), upper - three regions:
lip (2 to 7%) & oral commissure (1%). nasopharynx
- Predominantly squamous cell CA extends from the posterior nasal septum and
- Basal cell carcinoma presents more frequently on the choana to the skull base
upper lip than lower. includes fossa of rossenmuller, Eustachian
- Clinical findings: tube orifices (torus tuberous) and adenoid pad
ulcerated lesion on the vermilion or cutaneous bilateral regional metastatic spread in this
surface. area is common
(+) paresthesia in the area of lesion: mental Lymphadenopathy of the posterior triangle
nerve involvement. (level V) of the neck should provoke
- unfavorable prognosticating factors: perineural consideration for a nasopharyngeal primary
invasion, involvement of maxilla/mandible, upper lip or Oropharynx:
commissure involvement, regional lymphatic Includes tonsillar region, base of tongue, soft
metastasis, and age younger than 40 years at onset. palate, and posterolateral pharyngeal walls
- primary echelon of nodes at risk is in the Regional lymphatic drainage for
submandibular and submental regions oropharyngeal lesions frequently occurs to the
- Tx: upper and lower cervical lymphatics
T1 & T2 (4cm): Surgery = RT (levels II, III, IV) +Retropharyngeal
T3 & T4: surgical excision with histologic metastatic spread
confirmation of tumor-free margins + postop RT hypopharynx.
TOPNOTCH MEDICAL BOARD PREP SURGERY SUPPLEMENT HANDOUT Page 45 of 85
For inquiries visit www.topnotchboardprep.com.ph or email us at topnotchmedicalboardprep@gmail.com
TOPNOTCH MEDICAL BOARD PREP SURGERY SUPPLEMENT HANDOUT - Jules Lopez,MD-MBA,Teddy
Carpio,MD-MBA
For inquiries visit www.topnotchboardprep.com.ph or email us at topnotchmedicalboardprep@gmail.com
extends from the vallecula to the lower border CT and/or MRI imaging: check for regional
of the cricoid posterior and lateral to the metastases (paratracheal and upper mediastinal
larynx. lymph nodes)
includes pyriform fossa, the postcricoid space, - Tx:
and posterior pharyngeal wall. T1: RT
Regional lymphatic spread is frequently T2 & T3: chemoradiation
bilateral and to the mid- and lower cervical Larynx-preserving surgical procedures: only if the
lymph nodes (levels III, IV) tumor must not involve the apex of the pyriform
sinus, vocal cord mobility must be unimpaired, and
G. TUMORS OF THE NASOPHARYNX the patient must have adequate pulmonary
- Tumors arising in the nasopharynx are usually of reserve.
squamous cell origin Bilateral neck dissection is frequently indicated
- Most common nasopharyngeal malignancy in the given the elevated risk of nodal metastases found
pediatric age group: lymphoma with these lesions
- Risk factors for nasopharyngeal carcinoma: area of
habitation & ethnicity (southern China, Africa, J. ANATOMY OF LARYNX:
Alaska, and in Greenland Eskimos.), EBV infection, & - divided into 3 regions:
tobacco use. supraglottis: epiglottis (lined by stratified,
- Symptoms: nonkeratinizing squamous epithelium), false vocal
nasal obstruction, posterior (level V) neck cords (lined by pseudostratified, ciliated
mass, epistaxis, headache, serous otitis media respiratory epithelium), medial surface of the
with hearing loss, and otalgia. aryepiglottic folds, and the roof of the laryngeal
Cranial nerve involvement is indicative of skull ventricles
base extension and advanced disease. has a rich lymphatic network, which
- Lymphatic spread occurs to the posterior cervical, accounts for the high rate of bilateral spread
upper jugular, and retropharyngeal nodes. of metastatic disease
- Bilateral regional metastatic spread is common. glottis: the true vocal cords, anterior and posterior
- Diagnosis for nasopharyngeal CA: commissure, and the floor of the laryngeal
flexible or rigid fiber-optic endoscope ventricle.
CT with contrast: determining bone destruction Subglottis: extends from below the true vocal
MRI: assess for intracranial and soft-tissue cords to the cephalic border of the cricoid within
extension. the airway
- Tx: chemoradiation pseudostratified, ciliated respiratory
epithelium
H. TUMORS OF THE OROPHARYNX Glottic and subglottic lesions: spread to the
- Direct extension of tumors from the oropharynx into cervical chain, paralaryngeal and paratracheal
these lateral tissues may involve spread into the LN
parapharyngeal space
- histology of the majority of tumors in this region is K. BENIGN CONDITIONS OF THE LARYNX
squamous cell carcinoma
- (+) asymmetrical enlargement of the tonsils and tongue Recurrent respiratory papillomatosis (RRP)
base think lymphoma - (+)HPV 6 & 11
- Clinical findings: ulcerative lesion, exophytic mass, - larynx is the most frequently involved site
tumor fetor, muffled or "hot potato" voice (large tongue - presents in early childhood, secondary to viral
base tumors), Dysphagia, weight loss, Referred otalgia, acquisition during vaginal delivery.
(tympanic branches of CN IX & CN X), Trismus - Sx: hoarseness, airway compromise
(involvement of the pterygoid musculature), ipsilateral - Diagnosis: endoscopy
or bilateral nontender cervical lymphadenopathy - Tx: operative microlaryngoscopy with excision or laser
- LN metastasis from oropharyngeal cancer most ablation
commonly occurs in the subdigastric area of level II. - High tendency to recur
Others - levels III, IV, & V, retropharyngeal &
parapharyngeal LN. Laryngeal granulomas
Bilateral metastases: seen in tumors originating - typically occur in the posterior larynx on the arytenoid
from the tongue base and soft palate; if found in mucosa
these areas associated with poor survival - risk factors: reflux, voice abuse, chronic throat clearing,
- Tx: endotracheal intubation, and vocal fold paralysis
Options: surgery, primary radiation alone, surgery - Sx: pain often with swallowing (less commonly: vocal
with postoperative radiation, & combined changes)
chemotherapy with radiation therapy. - Dx: fiber-optic laryngoscopy, voice analysis, laryngeal
If tongue base crossing middling: do total electromyography (EMG), and pH probe testing.
glossectomy with possible total laryngectomy - Tx: voice rest, voice retraining therapy, and antireflux
Tumors of the oropharynx tend to be therapy.
radiosensitive.
Reinke's edema
I. TUMORS OF THE HYPOPHARYNX/CERVICAL - located at the superficial lamina propria due to injury to
ESOPHAGUS the capillaries that exist in this layer, with subsequent
- Squamous cancers of the hypopharynx frequently extravasation of fluid.
present at an advanced stage, hence are associated with - Sx: rough, low-pitched voice.
poorer survival rates - Risk factors: smoking, laryngopharyngeal reflux,
- Clinical findings: neck mass, muffled or hoarse voice, hypothyroidism, and vocal hyperfunction.
referred otalgia, progressive dysphagia to solids
liquids, weight loss. Vocal cord cyst
- Invasion of the larynx by direct extension vocal - may occur under the laryngeal mucosa (in regions
cord paralysis (if unilaterally affected) airway containing mucous-secreting glands)
compromise (if bilaterally affected) - Cysts of the vocal cord may be difficult to distinguish
- Diagnosis: from vocal polyps
flexible fiber-optic laryngoscopy - Diagnosis: video stroboscopic laryngoscopy

TOPNOTCH MEDICAL BOARD PREP SURGERY SUPPLEMENT HANDOUT Page 46 of 85


For inquiries visit www.topnotchboardprep.com.ph or email us at topnotchmedicalboardprep@gmail.com
TOPNOTCH MEDICAL BOARD PREP SURGERY SUPPLEMENT HANDOUT - Jules Lopez,MD-MBA,Teddy
Carpio,MD-MBA
For inquiries visit www.topnotchboardprep.com.ph or email us at topnotchmedicalboardprep@gmail.com
- Tx: Large cysts of the supraglottic larynx are treated by
marsupialization with cold steel or a CO2 laser.

Vocal cord paralysis


- most commonly is iatrogenic (s/p thyroid,
parathyroid, carotid, or cardiothoracic surgeries)
- can be secondary to malignant processes in the lungs,
thyroid, esophagus, thoracic cavity, skull base, or neck.
- Sx: presents with hoarseness and breathy voice
If superior laryngeal nerve is affected
demonstrate aspiration secondary to diminished
supraglottic sensation
left vocal cord is more commonly involved
secondary to its longer course of the recurrent
laryngeal nerve (RLN) on that side 1. Level I: submental & submandibular nodes
if anterior surgical approaches to the cervical spine - Level Ia: the submental nodes; medial to the anterior
are performed right RLN is at an increased risk belly of the digastric muscle bilaterally, symphysis of
(courses more laterally to the tracheoesophageal mandible superiorly, and hyoid inferiorly
complex) - Level Ib: the submandibular nodes and gland; posterior
to the anterior belly of digastric, anterior to the
L. LARYNGEAL CARCINOMA posterior belly of digastric, and inferior to the body of
- Suspect if with (+) Hx of smoking & complaint of a the mandible
change in vocal quality
- are primarily squamous cell carcinoma 2. Level II: upper jugular chain nodes
- sx: - Level IIa: jugulodigastric nodes; deep to
supraglottic larynx: chronic sore throat, sternocleidomastoid (SCM) muscle, anterior to the
dysphonia ("hot potato" voice), dysphagia, or a posterior border of the muscle, posterior to the
neck mass secondary to regional metastasis, posterior aspect of the posterior belly of digastric,
Referred otalgia or odynophagia is encountered superior to the level of the hyoid, inferior to spinal
with advanced supraglottic cancers. accessory nerve (CN XI)
Glottic larynx: hoarseness (early; because only a - Level IIb: submuscular recess; superior to spinal
small degree of change is required to produce accessory nerve to the level of the skull base
hoarseness), Airway obstruction (late), Decreased
vocal cord mobility may be caused by direct muscle 3. Level III: middle jugular chain nodes
invasion or involvement of the RLN. - inferior to the hyoid, superior to the level of the cricoid,
Subglottic larynx: vocal cord paralysis (usually deep to SCM muscle from posterior border of the
unilateral) and/or airway compromise (are muscle to the strap muscles medially
relatively uncommon).
- Lymphatic drainage: 4. Level IV: lower jugular chain nodes
Supraglottic larynx: subdigastric and superior - inferior to the level of the cricoid, superior to the
jugular nodes clavicle, deep to SCM muscle from posterior border of
glottic and subglottic larynx: prelaryngeal node the muscle to the strap muscles medially
(the Delphian node), the paratracheal nodes, and jaime is the best!!
the deep cervical nodes
glottic cancers have limited lymphatic access 5. Level V: posterior triangle nodes
regional nodal metastases is low - Level Va: lateral to the posterior aspect of the SCM
- treatment muscle, inferior and medial to splenius capitis and
early stage glottis & supraglottic cancer: RT trapezius, superior to the spinal accessory nerve
small glottic cancers: Partial laryngectomy - Level Vb: lateral to the posterior aspect of SCM muscle,
supraglottic cancers w/o arytenoid or vocal cord medial to trapezius, inferior to the spinal accessory
extension: supraglottic laryngectomy nerve, superior to the clavicle
advanced tumors with extension : total
laryngectomy + postop RT 6. Level VI: anterior compartment nodes
Subglottic cancers: total laryngectomy. - inferior to the hyoid, superior to suprasternal notch,
medial to the lateral extent of the strap muscles
bilaterally
M. NECK AND ASSOCIATED CONDITIONS
- differential diagnosis of neck masses is dependent on its 7. Level VII: paratracheal nodes
location and patients age - inferior to the suprasternal notch in the upper
pediatric age: think congenital or inflammatory mediastinum
conditions
adult + risk factors: rule out malignancy Patterns of spread from primary tumor sites:
in terms of location, think about patterns of - oral cavity and lip: levels I, II, and III
drainage Skip metastases may occur with oral tongue
cancers such that involvement of nodes in level III
Lymphatic drainage of the neck is divided into 7 levels. or IV may occur without involvement of higher
echelon nodes (levels I & II).
Figure 30. levels of the neck bearing LN bearing regions
- oropharynx, hypopharynx, and larynx: levels II, III, and
IV.
- nasopharynx and thyroid: level V nodes in addition to
the jugular chain nodes.
- nasopharynx, soft palate, and lateral and posterior walls
of the oropharynx and hypopharynx: Retropharyngeal
lymph nodes
- hypopharynx, cervical esophagus, and thyroid:
paratracheal nodal compartment + upper mediastinum
nodes (level VII).

TOPNOTCH MEDICAL BOARD PREP SURGERY SUPPLEMENT HANDOUT Page 47 of 85


For inquiries visit www.topnotchboardprep.com.ph or email us at topnotchmedicalboardprep@gmail.com
TOPNOTCH MEDICAL BOARD PREP SURGERY SUPPLEMENT HANDOUT - Jules Lopez,MD-MBA,Teddy
Carpio,MD-MBA
For inquiries visit www.topnotchboardprep.com.ph or email us at topnotchmedicalboardprep@gmail.com
- advanced tumors of the glottis with subglottic spread: - present as a midline or paramedian cystic mass
Delphian node adjacent to the hyoid bone.
- After an upper respiratory infection, the cyst may
Neck dissections: enlarge or become infected.
- Tx: removal of the cyst, the tract, and the central portion
- Radical neck dissection (RND or CRILE method): of the hyoid bone (Sistrunk procedure) + portion of
removes levels I to V of the cervical lymphatics + SCM + the tongue base up to the foramen cecum.
internal jugular vein + CN XI - Check 1st for normal thyroid tissue in the lower neck
area & if ensure that patient is euthyroid
- Modified radical neck dissection (MRND) or - 1% of thyroglossal duct cysts contain cancer (85% is
functional neck dissection: Any modification of the usually papillary)
RND that preserves nonlymphatic structures (i.e., CN XI,
SCM muscle, or internal jugular vein) Congenital branchial cleft anomalies:
Comparable to RND in controlling regional - remnants are derived from the branchial cleft apparatus
metastasis with superior functional results that persists after fetal development.
- Selective neck dissection (SND): any modification of 1st branchial cleft: EAC & parotid gland.
the RND that preserves lymphatic compartments 2nd branchial cleft: courses between the internal
normally removed in RND and external carotid arteries and proceeds into the
Also comparable to RND in controlling regional tonsillar fossa
metastasis with superior functional results 3rd branchial cleft: courses posterior to the
Types: common carotid artery, ending in the pyriform
1. supraomohyoid neck dissection sinus region.
used with oral cavity malignancies
removes lymph nodes in levels I to III Dermoid cysts
2. lateral neck dissection - midline masses and represent trapped epithelium
used for laryngeal malignancies originating from the embryonic closure of the midline.
removes lymph nodes in levels II through IV
3. posterolateral neck dissection N. SALIVARY GLAND TUMORS
Used for thyroid cancer - Majority of neoplasms are benign
removes lymph nodes in levels II to V - Most common gland involved: parotid gland (85% of
- if clinically N(+) necks: do MRND or RND or SND ( only all salivary gland neoplasms)
if limited N1 disease) - Most common benign tumor of the salivary gland:
- if (+) extracapsular spread, perineural invasion, pleomorphic adenoma
vascular invasion, and the presence of multiple involved - Most common malignant epithelial neoplasm of
lymph nodes are noted neck dissection of choice + salivary gland: mucoepidermoid carcinoma
Adjuvant RT +/- chemoRT - 2nd most common malignant epithelial neoplasm of
salivary gland:Adenoid cystic carcinoma, which has a
Parapharyngeal space masses propensity for neural invasion,
- Is a potential space, shaped like an inverted pyramid - Risk of malignancy depending on location: minor
spanning the skull base to the hyoid. salivary gland > submandibular, sublingual >
- Contents of the prestyloid space: parotid, fat, and lymph parotid gland
nodes. - Symptoms suggestive of malignancy: pain, paresthesias,
- Contents of poststyloid compartment: CNs IX to XII, the facial nerve weakness, skin invasion, fixation to the
carotid space contents, cervical sympathetic chain, fat, mastoid tip and trismus (invasion of the masseter or
and lymph nodes. pterygoid muscles
- Tumors in this space can produce displacement of the - Tx:
lateral pharyngeal wall medially into the oropharynx If benign neoplasm: do surgical excision
, dysphagia, cranial nerve dysfunction, Horner's If parotid: minimal surgical procedure for
syndrome, or vascular compression. neoplasms of the parotid is superficial
- Tumors found in the parapharyngeal space: parotidectomy with preservation of the facial
40 to 50% of the tumors are of salivary gland nerve.
origin Most frequently injured nerve in
usually arising anterior to the styloid parotid surgery: greater auricular
process nerve (not facial nerve!); if transected,
20 to 25% of tumors are of neurogenic origin will produce numbness of the lower
such as paragangliomas (glomus vagale, carotid portion of the auricle & periauricular skin
body tumor), schwannomas, and neurofibroma If the auriculotemporal nerve is
usually arising posterior to the styloid injured Freys syndrome
process (postoperative gustatory sweating)
angiography has a role if the tumor in question
is located posterior to the styloid process If malignant: do en bloc removal of the involved
if a paraganglioma is suspected request for gland with preservation of all nerves unless
a 24-hour urinary catecholamine directly invaded by tumor.
15% represent LN metastases & 1o lymphoma if parotid tumor arising from the lateral
lobe: superficial parotidectomy with
Benign neck masses ecpreservation of CN VII is indicated.
If the tumor extends into the deep lobe of the
Thyroglossal duct cyst parotid: a total parotidectomy with nerve
- most commonly encountered congenital cervical preservation is performed
anomalies If submandibular involvement: en bloc
- represents the vestigial remainder of the tract of the resection of the gland and submental and
descending thyroid gland from the foramen cecum, at submandibular lymph nodes.
the tongue base, into the lower anterior neck during Nerves at risk for a submandibular
fetal development. gland removal: lingual and hypoglossal
- An embryological anomaly wherein there is failure of nerve
obliteration of the midline pharyngeal diverticulum Postoperative radiation treatment plays an
during thyroid descent important role in the treatment of salivary
TOPNOTCH MEDICAL BOARD PREP SURGERY SUPPLEMENT HANDOUT Page 48 of 85
For inquiries visit www.topnotchboardprep.com.ph or email us at topnotchmedicalboardprep@gmail.com
TOPNOTCH MEDICAL BOARD PREP SURGERY SUPPLEMENT HANDOUT - Jules Lopez,MD-MBA,Teddy
Carpio,MD-MBA
For inquiries visit www.topnotchboardprep.com.ph or email us at topnotchmedicalboardprep@gmail.com
malignancies. The presence of - Increased levels seen in hyperthyroid patients, elevated
extraglandular disease, perineural Tg levels secondary to pregnancy,
invasion, direct invasion of regional estrogen/progesterone use or congenital diseases
structures, regional metastasis, and high- - Decreased levels seen in hypothyroid patients,
grade histology are all indications for decreased Tg levels secondary to anabolic steroid use
radiation treatment and protein losing disorders (i.e. nephrotic syndrome)
These individuals maybe euthyroid if their free
O. THYROID AND ASSOCIATED CONDITIONS T4levels are normal

Important facts about Thyroid anatomy: 3. Total T3


- Normal: 1.5 3.5 nmol/L
- Weight of a normal thyroid gland: 20 g - Total T3 levels reflect peripheral thyroid hormone
- pyramidal lobe is present in about 50% of patients metabolism
in disorders resulting in thyroid hypertrophy (e.g., - Not suitable as a general screening test
Graves' disease, diffuse nodular goiter, or - Measurement of total T3 levels is important for
lymphocytic thyroiditis), the pyramidal lobe clinically hyperthyroid patients with normal T4 levels
usually is enlarged and palpable think T3 thyrotoxicosis
- enveloped by a loosely connecting fascia
- thyroidea ima artery: arises directly from the aorta or 4. Free T4
innominate in 1-4% of individuals - Normal: 12 28 pmol/L
- ligament of berry: posteromedial suspensory - Measures the biologically active hormone
ligament; has a close relationship with the recurrent - Not performed as a routine screening test in thyroid
laryngeal nerve disease
- inferior thyroid artery crosses recurrent laryngeal - Its utility is in detecting early hyperthyroidism in which
total T4 levels maybe normal but free T4 levels are
nerve (RLN), necessitating identification of the RLN
raised
before ligation - Refetoff syndrome: end-organ resistance to T4 wherein
- RLNs innervate all the intrinsic muscles of the free T4 are increased and TSH levels are normal
larynx, except the cricothyroid muscles, which are
innervated by the external laryngeal nerves 5. Free T3
Injury to one RLN: paralysis of the ipsilateral - normal: 3 9 pmol/L
vocal cord (lie in the paramedian or the abducted - most useful in the diagnosis of early hyperthyroidism in
position) which levels of free T3 and T4 rise before total T3 and T4
Injury to Bilateral RLN: airway obstruction,
necessitating emergency tracheostomy, or loss of 6. Serum TRH
voice. - used for the evaluation of pituitary TASH secretory
Most common position of right RLN: posterior function
to the inferior thyroid artery
- Injury to the internal branch of the superior 7. Thyroid antibodies
laryngeal nerve aspiration. - include anti-Tg, antimicrosomal, or anti-TPO and TSI
- Injury to the external branch of the superior - anti-Tg & anti-TPO antibody levels: elevated if with
laryngeal nerve inability to tense the ipsilateral autoimmune thyroiditis
vocal cord and hence difficulty "hitting high notes - can be elevated in Hashimotos, Graves, multinodular
- Loop of galen: where the pharyngeal branches of the goiter & thyroid neoplasms
recurrent laryngeal nerve communicate with the
branches of the superior laryngeal nerve. Maybe 8. Serum Thyroglobulin
injured when dissecting or ligating the superior - amount is increased in destructive processes of the
thyroid artery thyroid gland (thyroiditis) or overactive states (graves
- Regional lymph nodes include pretracheal, or toxic multinodular goiter)
paratracheal, perithyroidal, RLN, superior mediastinal, - most important use is for the monitoring of
retropharyngeal, esophageal, and upper, middle, and differentiated thyroid cancer recurrence, after total
lower jugular chain nodes. thyroidectomy and RAI ablation
- Histology: elevated anti-Tg antibodies can interfere with the
the thyroid is divided into lobules that contain 20 accuracy of Tg levels and should always be
to 40 follicles measured when interpreting Tg levels.
Each follicle is lined by cuboidal epithelial cells
C cells or parafollicular cells: secrete the 9. Serum Calcitonin
hormone calcitonin. - normal: 0-4 pg/mL basal
- secreted by C cells
Evaluation of patients with thyroid disease: - function: lower serum calcium
- sensitive marker for medullary thyroid cancer
Tests of thyroid function:
Thyroid Imaging:
1. Serum TSH
- normal: 0.5 5U/mL 1. Radionuclide imaging
- only test necessary in most patients with thyroid
nodules that clinically appear euthyroid Types:
- serum TSH levels reflect the ability of the anterior 1. iodine 123 (123I)
pituitary to detect free T4 levels - emits low dose radiation
- ultrasensitive TSH assay: most sensitive and most - t 1/2 : 12 14 hours
specific test for the diagnosis of hyperthyroidism and - used to image lingual thyroids or goiter
hypothyroidism
2. iodine 131 (131I)
2. Total T4 - higher dose radiation exposure because of longer t
- normal: T4: 55 150 nmol/L - t : 8 to 10 days
- Total T4 levels reflect the output from the thyroid gland - used to screen and treat patients with differentiated
- Not suitable as a general screening test thyroid cancers for metastatic disease

TOPNOTCH MEDICAL BOARD PREP SURGERY SUPPLEMENT HANDOUT Page 49 of 85


For inquiries visit www.topnotchboardprep.com.ph or email us at topnotchmedicalboardprep@gmail.com
TOPNOTCH MEDICAL BOARD PREP SURGERY SUPPLEMENT HANDOUT - Jules Lopez,MD-MBA,Teddy
Carpio,MD-MBA
For inquiries visit www.topnotchboardprep.com.ph or email us at topnotchmedicalboardprep@gmail.com
** Cold: trap less radioactivity compared to the -
Macroscopic appearance: diffusely and smoothly
surrounding gland, risk of malignancy is higher in cold enlarged, increase in vascularity
lesions (20%) compared to hot lesions (<5%) - Microscopic appearance: hyperplastic gland, minimal
** Hot: trap more radioactivity, therefore, with colloid present
increased acitivity - Clinical features:
Hyperthyroid Sx: heat intolerance, sweating,
3. technetium Tc 99m pertechnetate (99mTc) thirst, weight loss despite adequate caloric
- this isotope is taken up by the mitochondria intake
- shorter t , therefore, less radiation exposure adrenergic excess: palpitations, nervousness,
- sensitive for nodal metastases fatigue, emotional lability, hyperkinesis, and
tremors
4. F-fluorodeoxyglucose PET scan most common GI symptom: diarrhea
- used to screen for metastases in patients with can also develop amenorrhea, decreased fertility,
thyroid cancer in whom other imaging studies are and an increased incidence of miscarriages
negative. - PE: facial flushing, warm & moist skin, Tachycardia,
- May show clinically occult lesions atrial fibrillation, fine tremor, muscle wasting, and
proximal muscle group weakness with hyperactive
2. Ultrasound tendon reflexes
- excellent noninvasive imaging study of thyroid gland 50% of patients ophthalmopathy
- no radiation exposure lid lag (von Graefe's sign)
- useful for the evaluation of thyroid nodules, spasm of the upper eyelid
distinguinshing cystic from solid ones, size, revealing the sclera above the corneoscleral
multicentricity and cervical lymphadenopathy limbus (Dalrymple's sign)
prominent stare
3. CT/MRI
1 to 2% of patients dermopathy (deposition of
- useful for the evaluation of extent of large, fixed or
glycosaminoglycans leading to thickened skin in
substernal goiters and their relationship to the
the pretibial region and dorsum of the foot)
airway and vascular structures
- Diagnostic: suppressed TSH with or without an
Developmental abnormalities: elevated free T4 or T3 level.
If eye signs are present, other tests are generally
Thyroglossal duct cyst (see Neck) not needed.
123I uptake and scan: elevated uptake, with a
Lingual thyroid diffusely enlarged gland, confirms the diagnosis
- Treatment:
- failure of the median thyroid anlage to descend
Antihyroid drugs:
normally
propylthiouracil (PTU, 100 to 300 mg three
- may appear as reddish brown mass at the base of the
tongue times daily) or methimazole (10 to 30 mg
- may be the only thyroid tissue present (hence, if three times daily, then once daily because it
surgical tx is warranted, evaluation of normal has a longer half t )
thyroid tissue in the neck must be carried out 1st) MOA: inhibits the organic binding of
- Intervention becomes necessary for obstructive iodine and the coupling of iodotyrosines
symptoms such as choking, dysphagia, airway (mediated by TPO).
obstruction, or hemorrhage or if suspicious for PTU also inhibits the peripheral
malignancy conversion of T4 to T3
- Tx: administration of exogenous ORAL thyroid Most patients have improved
hormone to suppress thyroid-stimulating hormone symptoms in 2 weeks and become
(TSH) and radioactive iodine (RAI) ablation followed euthyroid in about 6 weeks.
by hormone replacement. IMPORTANT SIDE EFFECT OF PTU:
AGRANULOCYTOSIS
Ectopic thyroid Propranolol is the most commonly
- Normal thyroid tissue in aberrant locations (esophagus, prescribed medication in doses of about 20 to
trachea and anterior mediastinum) 40 mg four times daily for control of
adrenergic symptoms
Pyramidal lobe RAI: most often used in older patients with small
- The distal end of the atrophied thyroglossal duct that or moderate-sized goiters, those who have
connects to the thyroid, projecting up to the isthmus, relapsed after medical or surgical therapy, and
lying just to the left or right of the midline. those in whom antithyroid drugs or surgery are
contraindicated.
Benign thyroid disorders Absolute CI: women who are pregnant or
breastfeeding
Hyperthyroidism Relative contraindications:
young patients (i.e., especially children
Graves disease and adolescents)
- autoimmune disease with a strong familial those with thyroid nodules
predisposition, female preponderance (5:1), and those with ophthalmopathy
peak incidence between the ages of 40 to 60 years. Surgery:
- Most common cause of hyperthyroidism in North Patients should be rendered euthyroid before
America operation
- characterized by thyrotoxicosis, diffuse goiter, and Lugol's iodide solution or saturated
extrathyroidal conditions including ophthalmopathy, potassium iodide generally is administered
dermopathy (pretibial myxedema), thyroid acropachy beginning 7 to 10 days preoperatively (three
- hallmark: thyroid-stimulating antibodies stimulate drops twice daily) to reduce vascularity of
the thyrocytes to grow and synthesize excess the gland and decrease the risk of
thyroid hormone precipitating thyroid storm.
- associated with other autoimmune conditions (ex. type I Indications for Total or near-total
DM, Addison's disease, pernicious anemia, and thyroidectomy: Patients with coexistent
myasthenia gravis) thyroid cancer, and those who refuse RAI
TOPNOTCH MEDICAL BOARD PREP SURGERY SUPPLEMENT HANDOUT Page 50 of 85
For inquiries visit www.topnotchboardprep.com.ph or email us at topnotchmedicalboardprep@gmail.com
TOPNOTCH MEDICAL BOARD PREP SURGERY SUPPLEMENT HANDOUT - Jules Lopez,MD-MBA,Teddy
Carpio,MD-MBA
For inquiries visit www.topnotchboardprep.com.ph or email us at topnotchmedicalboardprep@gmail.com
therapy or have severe ophthalmopathy or - autoimmune process leads to destruction of thyrocytes
have life-threatening reactions to antithyroid by autoantibodies, which lead to complement fixation
medications (vasculitis, agranulocytosis, or and killing by natural killer cells
liver failure) - Antibodies directed against three main antigensTg
(60%), TPO (95%), the TSH-R (60%), and, less
commonly, to the sodium/iodine symporter (25%)
REMEMBER: a - more common in women (male:female ratio 1:10 to 20 )
between the ages of 30 and 50 years old.
How would you know if there is an undiagnosed - The most common presentation is that of a
hyperthyroid problem intraoperatively? minimally or moderately enlarged firm granular
gland discovered on routine PE or the awareness of
Increased vascularity increased bleeding in a sedated a painless anterior neck mass
patient - Gross appearance: mildly enlarged, pale, gray-tan cut
surface that is granular, nodular, and firm.
- microscopic examination: the gland is diffusely
Toxic multinodular goiter infiltrated by small lymphocytes and plasma cells and
- Symptoms and signs of hyperthyroidism are similar to occasionally shows well-developed germinal centers,
Graves' disease, but extrathyroidal manifestations follicles are lined by Hrthle or Askanazy cells
are absent - Dx: elevated TSH and the presence of thyroid
- Possible presence of cervical compressive symptoms autoantibodies usually confirm the diagnosis.
- Diagnosis: - Tx: Thyroid hormone replacement therapy or surgery
suppressed TSH level and elevated free T4 or T3 (if with compressive symptoms)
levels.
RAI uptake also is increased, showing multiple Reidels thyroiditis
nodules with increased uptake - characterized by the replacement of all or part of the
Treatment: subtotal thyroidectomy thyroid parenchyma by fibrous tissue
- primary autoimmune etiology
Toxic adenoma (Plummers disease) - occurs predominantly in women between the ages of 30
- Hyperthyroidism from a single hyperfunctioning nodule to 60 years old.
- typically occurs in younger patients - presents as a painless, hard anterior neck mass, which
- PE: solitary thyroid nodule without palpable thyroid progresses over weeks to years to produce symptoms
tissue on the contralateral side of compression, including dysphagia, dyspnea,
- RAI: "hot" nodule choking, and hoarseness.
- rarely malignant. - Can result to hypothyroidism
- Tx: Surgery (lobectomy and isthmusectomy on the - Associated with retroperitoneal fibrosis and sclerosing
affected side) is preferred to treat young patients and mediastinitis
those with larger nodules. - PE: hard, "woody" thyroid gland with fixation to
surrounding tissues.
Thyroid storm - Tx: surgery
- hyperthyroidism + fever, central nervous system Goal of surgery: to decompress the trachea by
agitation or depression, cardiovascular dysfunction due wedge excision of the thyroid isthmus and to make
to infection, surgery, trauma or amiodarone a tissue diagnosis
administration.
- Tx: ICU, Beta blockers, Oxygen supplementation, Fever Solitary thyroid nodule
reduction, fluids, hemodynamic support, PTU, - History: time of onset ( usually slow and indolent),
Corticosteroids (to prevent adrenal exhaustion and change in size, and associated symptoms such as pain,
block hepatic thyroid hormone conversion) dysphagia, dyspnea, choking, hoarseness (secondary to
malignant involvement of the RLNs)
Acute (suppurative) thyroiditis - Risk factors for malignancy:
- more common in children and often is preceded by an 1. exposure to ionizing radiation
upper respiratory tract infection or otitis media. 2. (+) FH of thyroid and other malignancies
- It is characterized by severe neck pain radiating to the associated with thyroid cancer
jaws or ear, fever, chills, odynophagia, and dysphonia. 3. Men > women
- Complications: systemic sepsis, tracheal or esophageal 4. Children > adults
rupture, jugular vein thrombosis, laryngeal chondritis, - PE: nodules that are hard, gritty, or fixed to surrounding
and perichondritis or sympathetic trunk paralysis structures (if malignant)
- Streptococcus and anaerobes account for about 70% - Diagnosis:
of cases
- Diagnosis: leukocytosis on blood tests and FNAB for Figure 31. Management of a solitary thyroid nodule
Gram's stain, culture, and cytology.
- Tx: parenteral antibiotics & drainage of abscesses.

Subacute (de quervains) thyroiditis


- strong association with the HLA-B35 haplotype.
- Self-limiting painful thyroiditis most commonly occurs
in 30- to 40-year-old women
- characterized by the sudden or gradual onset of neck
pain, which may radiate toward the mandible or ear.
- History of a preceding upper respiratory tract
infection often can be elicited.
- The gland is enlarged, exquisitely tender, and firm.
- Diagnosis: TSH is decreased, and Tg, T4 , and T3 levels
are elevated (during the early phase)
- tx: symptomatic; NSAIDs are used for pain relief
(steroids may be indicated in more severe cases)

Hashimotos thyroiditis (chronic thyroiditis)

TOPNOTCH MEDICAL BOARD PREP SURGERY SUPPLEMENT HANDOUT Page 51 of 85


For inquiries visit www.topnotchboardprep.com.ph or email us at topnotchmedicalboardprep@gmail.com
TOPNOTCH MEDICAL BOARD PREP SURGERY SUPPLEMENT HANDOUT - Jules Lopez,MD-MBA,Teddy
Carpio,MD-MBA
For inquiries visit www.topnotchboardprep.com.ph or email us at topnotchmedicalboardprep@gmail.com
FNAB: single most important test in the "grooving," and intranuclear cytoplasmic
evaluation of thyroid masses; 1st diagnostic test inclusions (Orphan Annie nuclei)
ordered in a patient with a solitary thyroid
Psammoma bodies: microscopic, calcified
nodule
deposits representing clumps of sloughed cells,
Results: benign cysts & colloid nodules
also may be present
(65%), suspicious follicular or hurthle cell
Multifocality: associated with an increased risk of
neoplasms (20%), malignant (5%), and
cervical nodal metastases
nondiagnostic (10%)
Other variants: tall cell, insular, columnar, diffuse
false-positive results is about 1%
sclerosing, clear cell, trabecular, and poorly
false-negative results occur in approximately
differentiated types.
3%
are generally associated with a worse
a negative FNAB does not rule out CA
prognosis.
if suspicious result, the diagnosis of
- Tx:
malignancy relies on demonstrating
If less than 1.5 cm: lobectomy + isthmusectomy
capsular or vascular invasion, features
If multicentric: near total or total thyroidectomy
that cannot be determined via FNAB.
(+) cervical node mets: MDRD
RAI scan:
patients with papillary thyroid CA have an
Single, cold, solid nodule malignant
excellent prognosis with a >95% 10-year
Multiply, Hot, cystic benign
survival rate.
Labs:
Age is the most important prognostic factor
TSH: expect euthyroid
in determining long term survival
Tg levels: useful for patients who have
undergone total thyroidectomy for thyroid
Follicular CA
cancer & for serial evaluation of patients
- account for 10% of thyroid cancers
undergoing nonoperative management of
- occur more commonly in iodine-deficient areas.
thyroid nodules.
- Women have a higher incidence of follicular cancer,
Serum calcitonin: obtained in patients with
with a female-to-male ratio of 3:1
MTC or a family history of MTC or MEN2
- usually present as solitary thyroid nodules, occasionally
RET oncogene mutations: All patients with
with a history of rapid size increase, and long-standing
MTC should be tested for RET oncogene
goiter.
mutations and have a 24-hour urine collection
- In <1% of cases, follicular cancers may be
with measurement of levels of
hyperfunctioning, leading patients to present with signs
vanillylmandelic acid (VMA), metanephrine,
and symptoms of thyrotoxicosis.
and catecholamine to rule out a coexisting
- Spread via hematogenous route, hence their spread
pheochromocytoma.
is more distant, than regional
Ultrasound: helpful for detecting nonpalpable
- Most common site of distant metastasis: lung &
thyroid nodules, differentiating solid from cystic
bone
nodules, and identifying adjacent
- Diagnosis:
lymphadenopathy
FNAB shows follicular type must do lobectomy
- Tx:
to demonstrate capsular or vascular invasion
Malignant tumors are treated by thyroidectomy
(criteria for malignancy)
Simple thyroid cysts resolve with aspiration; if
- Microscopically: follicles are present, but the lumen may
persists after 3 attempts at aspiration unilateral
be devoid of colloid.
thyroid lobectomy is recommended.
- Tx:
Lobectomy is recommended for cysts >4 cm in
If follicular lesion thyroid lobectomy +
diameter or complex cysts with solid and cystic
isthmusectomy because at least 80% of these
components
patients will have benign adenomas.
Colloid nodule observe with serial ultrasound
older patients with follicular lesions >4 cm: total
and Tg measurements.
thyroidectomy
if (+) thyroid CA: do Total thyroidectomy
Thyroid cancer (+) cervical node mets: MDRD
mortality from follicular thyroid cancer is
Papillary CA approximately 15% at 10 years and 30% at 20
- 80% of all thyroid malignancies in iodine-sufficient years.
areas Poor long-term prognosis: age over 50 years
- predominant thyroid cancer in children and old at presentation, tumor size >4 cm, higher
individuals exposed to external radiation. tumor grade, marked vascular invasion,
- occurs more often in women, 2:1 extrathyroidal invasion, and distant
- symptoms of locally advanced disease: Dysphagia, metastases at the time of diagnosis.
dyspnea, and dysphonia
- Diagnosis: Hurtle cell CA
FNAB of the thyroid mass or lymph node. - account for approximately 3% of all thyroid
Complete neck UTZ: to evaluate the contralateral malignancies
lobe and for LN metastases in the central and - considered to be a subtype of follicular thyroid
lateral neck compartments. cancer.
- The most common sites are lungs, followed by bone, - Cannot be diagnosed by FNAB.
liver, and brain. - Microscopically: hurthle cells (variable enlargement,
- Spread via lymphatic route hyperchromatic nuclei and granular cytoplasm)
- Gross appearance: hard and whitish and remain flat on - Difference from follicular CA:
sectioning with a blade, macroscopic calcification, multifocal
necrosis, or cystic change may be apparent. bilateral (about 30%)
- Microscopically: usually do not take up RAI (about 5%)
papillary projections -a mixed pattern of papillary more likely to metastasize to local nodes (25%)
and follicular structures associated with a higher mortality rate (about 20%
pure follicular pattern (follicular variant). at 10 years)
Cells are cuboidal with pale, abundant higher recurrence rate
cytoplasm, large nuclei that may demonstrate - Same management with follicular neoplasms
TOPNOTCH MEDICAL BOARD PREP SURGERY SUPPLEMENT HANDOUT Page 52 of 85
For inquiries visit www.topnotchboardprep.com.ph or email us at topnotchmedicalboardprep@gmail.com
TOPNOTCH MEDICAL BOARD PREP SURGERY SUPPLEMENT HANDOUT - Jules Lopez,MD-MBA,Teddy
Carpio,MD-MBA
For inquiries visit www.topnotchboardprep.com.ph or email us at topnotchmedicalboardprep@gmail.com
- If (+) for hurthle malignancy: perform total 10-year survival rate is approximately 80% but
thyroidectomy + routine central neck (level 6) node decreases to 45% in patients with lymph node
removal or MDRD when lateral neck nodes are palpable. involvement.
best in patients with non-MEN familial MTC,
Medullary thyroid CA (MTC) followed by those with MEN2A
- accounts for about 5% of thyroid malignancies Prognosis is the worst (survival 35% at 10
- arises from the parafollicular or C cells of the thyroid years) in patients with MEN2B.
usually located superolaterally in the thyroid lobes
(usual site of MTC) Anaplastic CA
- female-to-male ratio is 1.5:1 - approximately 1% of all thyroid malignancies
- Most patients present between 50 and 60 years old - the most aggressive of thyroid malignancies
- Most MTCs occur sporadically. - Women are more commonly affected
Occur singly - present in the 7th & 8th decade of life
unilateral (80%) - Clinical features:
no familial predisposition Presents as a long-standing neck mass, which
- approximately 25% occur within the spectrum familial rapidly enlarges and may be painful. Associated
MTC - MEN2A (pheochromocytoma + parathyroid symptoms such as dysphonia, dysphagia, and
hyperplasia), and MEN2B (pheochromocytoma + dyspnea are common.
neuromas) Lymph nodes usually are palpable at presentation.
due to germline mutations in the RET proto- - Gross appearance: firm and whitish in appearance.
oncogene - Microscopically: characteristic giant and
encodes for tyrosine-kinase receptor in the multinucleated cells. with marked heterogeneity are
cell membrane seen (spindle shaped, polygonal, or large,
RET protein is expressed in tissues derived multinucleated cells)
from embryonic nervous and excretory - Tx: if resectable mass surgery will only give small
systems improvement in survival
present at a younger age - Prognosis: 6 months
multicentric
(+) C cell hyperplasia: premalignant lesion Thyroid Lymphoma
- clinical features: - <1% of thyroid malignancies
present with a neck mass that may be associated - Most common: non-Hodgkin's B-cell type.
with palpable cervical lymphadenopathy (15 to - develop in patients with chronic lymphocytic
20%). thyroiditis.
Pain or aching is common - present with a rapidly enlarging neck mass that is often
dysphagia, dyspnea, or dysphonia already painless.
invasive - may present with acute respiratory distress.
diarrhea indicates metastatic disease (due to - Tx:
increased intestinal motility and impaired CT(CHOPcyclophosphamide, doxorubicin,
intestinal water and electrolyte flux) vincristine, and prednisone) + RT
2 to 4% of patients develop Cushing's syndrome as Thyroidectomy and nodal resection: for alleviation
a result of ectopic production of of airway obstructive symptoms who do not
adrenocorticotropic hormone (ACTH) respond quickly to the above regimens or who
- tumor markers: calcitonin (diagnostic, most sensitive have completed the regimen before diagnosis.
tumor marker), CEA (better predictor of prognosis), - The overall 5-year survival rate is about 50%; patients
calcitonin generelated peptide, histaminadases, with extrathyroidal disease have markedly lower
prostaglandins E2 and F2 & serotonin. survival rates.
- Microscopically:
Heterogenous Thyroid surgeries:
Cells are polygonal or spindle shaped
infiltrating neoplastic cells separated by collagen Total thyroidectomy: dissection and remonal of all visible
and amyloid thyroid tissue bilaterally, which usually reveals the entrance of
presence of amyloid is a diagnostic finding the recurrent laryngeal nerve as they enter the ligament of berry
- Diagnosis:
history, physical examination, raised serum Near total thyroidectomy: complete hemithyroidectomy and
calcitonin, or CEA levels, and FNAB cytology of the isthmusectomy; most of the contralateral side is removed but a
thyroid mass remnant is left to prevent damage to parathyroid glands
all new patients with MTC should be screened
for RET point mutations, pheochromocytoma, Subtotal thyroidectomy: removes all visible thyroid tissue
and HPT. If (+) carrier, perform total except for a rim of thyroid tissue bilaterally to ensure
thyroidectomy parathyroid viability and avoids damage to the recurrent
- Tx: laryngeal nerve
Total thyroidectomy + bilateral central node
dissection (level 6) because of high incidence of REVIEW QUESTIONS
multicentricity
If (+) pheochromocytoma manage this 1st 1. Regarding salivary gland tumors, which one of
If with palpable cervical nodes or involved central the following statements is true?
neck nodes: ipsilateral or bilateral MDRD
If tumors >1 cm, ipsilateral prophylactic modified a. The majority of malignant salivary gland
radical neck dissection is recommended because tumors arise in the parotid gland
>60% of these patients have nodal metastases. b. Most parotid neoplasms are malignant
c. Fine needle aspiration biopsy is
if ipsilateral nodes are positive do contralateral
recommended for all suspected salivary
node dissection
gland malignancies
- Postoperative Follow-Up: annual measurements of
d. Minor salivary gland tumors occur most
calcitonin and CEA levels
commonly in the floor of the mouth
- Prognosis:
Answer: A

TOPNOTCH MEDICAL BOARD PREP SURGERY SUPPLEMENT HANDOUT Page 53 of 85


For inquiries visit www.topnotchboardprep.com.ph or email us at topnotchmedicalboardprep@gmail.com
TOPNOTCH MEDICAL BOARD PREP SURGERY SUPPLEMENT HANDOUT - Jules Lopez,MD-MBA,Teddy
Carpio,MD-MBA
For inquiries visit www.topnotchboardprep.com.ph or email us at topnotchmedicalboardprep@gmail.com
The likelihood of a given tumors being malignant is an operating microscope to visualize the field,
lowest in the parotid gland (approximately 20%), and 8.0 or 9.0 monofilament sutures to
followed by the submandibular salivary gland anstamose the cut ends of the nerves. There is no
(approximately 50%) and sublingual glands (nearly role for flexible bronchoscopy either
100%). However, because more than 75% of all intraoperatively or postoperatively unless there
salivary gland tumors occur in the parotid gland, the is uncertainty about the injury or the function of
parotid gland accounts for the majority of the the contralateral nerve.
malignant salivary gland tumors. The diagnostic
evaluation of a salivary gland mass depends on the
location and clinical scenario. FNAB is not indicated ESOPHAGUS
for all parotid tumors, since a tissue diagnosis does
not change the treatment plan for a patient with a A. Diagnostic tests for esophageal function
small, mobile mass clearly within the gland. When the B. GERD
location is uncertain, the history suggests the C. Diaphragmatic hernia
possibility of metastatic disease, or the tumor size or D. Schatzkis ring
location indicates a difficult facial nerve dissection, E. Scleroderma of esophagus
FNAB may be helpful. Biopsy, usually a punch or F. Zenkers diverticulum
excisional biopsy, should be performed for suspected G. Achalasia
minor salivary gland tumors, the most common site of H. Diffuse and segmental esophageal spasm
which is the palate, usually at the junction of the hard I. Nutcracker esophagus
and aoft palate. Like FNAB, imaging studies (CT or J. Hypertensive LES
MRI) should be used when they are likely to augment K. Esophageal diverticulum
the clinical assessment of staging and affect treatment L. Esophageal perforation
planning. M. Mallory weiss syndrome
N. Caustic injury
2. A 40 year-old woman comes to the clinicians O. Esophageal carcinoma
office with a thyroid mass, which is confirmed on
FNA and UTZ to be unilateral, 3.2 cm follicular
neoplasm. She has been completely
A. DIAGNOSTIC TESTS FOR ESOPHAGEAL FUNCTION
asymptomatic. What will the next intervention
be?
Tests to detect structural abnormalities
a. Total thyroidectomy
1. Barium swallow
b. Hemithyroidectomy or isthmusectomy
- 1st diagnostic test in patients with suspected esophageal
c. Excisional biopsy
disease (with full assessment of stomach and
d. Core-needle biopsy
duodenum)
e. Thyroid suppression via T3 or T4 analogues
- can reveal anatomic problems
- if patient complains of dysphagia and no obstructing
lesion seen in barium swallow use a barium-
Answer: A, B
impregnanted marshmallow, barium-soaked bread or
The presence of a follicular neoplasm as
barium hamburger
confirmed by FNA manadtes further evaluation,
will bring out the functional disturbance in the
since FNA does not provide enough information
esophageal transport that can be missed when
about tissie architecture to differentiate between
liquid barium is used.
a benign follicular adenoma and a follicular
carcinoma. Vascular or capsular invasion
2. endoscopic evaluation
confirms the presence of carcinoma. The
- endoscopy is indicated in patients complaining of
management of small, unilateral follicular lesions
dysphagia even with a normal radiographic study
is controversial (total versus hemi-
thyroidectomy with frozen section). Hwoever,
Tests to detect functional abnormalities
lesions larger than 4 cm should be treated with
1. manometry
total thyroidectomy, since multicentricity
- indicated when a motor abnormality of the esophagus is
becomes more common as tumor size increases.
considered on the basis of complaints (dysphagia,
Total thyroidectomy also facilitates the
odonyphagia, or noncardiac chest pain) and barium
effectiveness of postoperative radioactive iodine,
swallow and endoscopy does not show a structural
since no residual thyroid tissue remains to serve
abnormality
as a sink for the radioisotope.
- essential tool in preoperative evaluation of patients
before antireflux surgery
3. During a total thyroidectomy for papillary cancer,
the clinician observes an intact recurrent
Tests to detect increased exposure to gastric juice
laryngeal nerve on the right side and a
1. 24 hour ambulatory pH monitoring
completely transected nerve on the left, with
- most direct method of measuring increased esophageal
both ends in view. What should management of
exposure to gastric juice (not reflux)
this patient at this point entail?
- sensitivity and specificity of 96%
- gold standard for the dioagnosis of GERD
a. Complete the operation and evaluate the
vocal cords postoperatively via flexible
2. radiographic exposure of gastroesophageal reflux
bronchoscopy
- radiographic demonstration of spontaneous
b. Perform intraoperative flexible
regurgitation of barium into the esophagus in the
bronchoscopy to evaluate vocal cords
upright position is a reliable indicator that reflux is
c. Repair the nerve using 8.0 monofilament
present
sutures
- note: failure to see this does not indicate absence of
d. None of the above
disease
Answer: D
B. GERD
If the recurrent laryngeal nerve is injured or
- Clinical features:
transected during an otherwise uncomplicated
operation, it should be repaired using loupes or
TOPNOTCH MEDICAL BOARD PREP SURGERY SUPPLEMENT HANDOUT Page 54 of 85
For inquiries visit www.topnotchboardprep.com.ph or email us at topnotchmedicalboardprep@gmail.com
TOPNOTCH MEDICAL BOARD PREP SURGERY SUPPLEMENT HANDOUT - Jules Lopez,MD-MBA,Teddy
Carpio,MD-MBA
For inquiries visit www.topnotchboardprep.com.ph or email us at topnotchmedicalboardprep@gmail.com
1. Heartburn: substernal burning-type discomfort, LERD
beginning in the epigastrium and radiating upward. Adult-onset asthma
-It is often aggravated by meals, spicy or fatty Idiopathic pulmonary fibrosis
foods, chocolate, alcohol, and coffee - Treatment:
-worse in the supine position Medical:
1. Regurgitation: effortless return of acid or bitter Uncomplicated GERD: 12 weeks of empiric
gastric contents into the chest, pharynx, or mouth; treatment of antacid
highly suggestive of foregut pathology Persistent sx: PPIs or H2 antagonists
-severe at night when supine or when bending over A structurally defective LES is the most
-secondary to either an incompetent GEJ important factor predicting failure of
-explains the associated pulmonary symptoms, medical therapy
including cough, hoarseness, asthma, and recurrent They dont respond to medical therapy
pneumonia. well; candidates for anti-reflux surgery
2. Dysphagia: most specific symptom of foregut Lifestyle changes: elevate the head of the bed
disease; sensation of difficulty in the passage of during sleep; avoid tight-fitting clothing; eat small,
food from the mouth to the stomach frequent meals; avoid eating the nighttime meal
3. Chest pain immediately prior to bedtime; and avoid alcohol,
- primary cause of GERD: permanent attenuation of the coffee, chocolate, and peppermint (which are
collar sling musculature, with a resultant opening of known to reduce resting LES pressure)
the gastric cardia and loss of the high-pressure zone Surgical
as measured with esophageal manometry Nissen fundiplication: a abdominal or
characteristics of a defective sphincter thoracic approach using a 360 degree
1. LES with a mean resting pressure of less than 6 circumferential wrap of the gastric fundus
mmHg Belsey operation: difficult to learn, performed
2. overall sphincter length of <2 cm through the chestm, involves placement of 2
3. intra-abdominal sphincter length of <1 cm layers of placating structures between the
(most important consideration affecting the gastric fundus and lower esophagus with
competence of the GE jxn) subsequent creation of 280 degree anterior
- diagnosis: gastric wrap and posterior approximation of
24 hour pH monitoring (gold standard): most the crura
sensitive for the detection of reflux Hill operation: approach is through the
Endoscopic examination: assessing anatomic abdomen, posterior approximation of the
damage produced by reflux (esophagitis, ulceration crura followed by anchoring of the posterior
and strictures) & for ruling out CA and anterior aspects of the GEJ to the median
Grading of esophagitis arcuate ligament adjacent to the aorta,
Grade I: small circular nonconfluent erosions creating a 180 degree gastric wrap
Grade II: presence of linear erosions lined with Collis gastroplasty: esophageal lengthening
granulation tissue that bleeds easily when procedure
touched Angelchik prosthesis: horshe shoe shape
Grade II: linear erosions coaslesce into a silastic device placed around the distal
circumferential loss of the epithelium; esophagus, keeping this segment in the
cobblestone mucosa abdomen
Grade IV: (+) stricture
MUST KNOW a
**absence of esophagitis above a stricture Principles of surgical therapy in reflux disease
suggest chemical induced injury or neoplasm
as a cause. 1. the operation should restore the pressure of the
i. distal esophageal sphincter to a level twice the
Manometric studies: rule out motility DO resting gastric pressure
- Complications: 2. the operation should place an adequate length of the
1. Metaplastic (Barrett's Esophagus) distal esophageal sphincter in the positive-pressure
condition whereby the tubular esophagus is environment of the abdomen by a method that
lined with columnar epithelium rather than ensures its response to changes in intra-abdominal
squamous epithelium pressure
occurs in 10 to 15% of patients with GERD 3. the operation should allow the reconstructed cardia
end stage of natural Hx of GERD to relax on deglutition
hallmark: presence of intestinal goblet 4. the fundoplication should not increase the resistance
cells in esophageal epithelium (intestinal of the relaxed sphincter to a level that exceeds the
metaplasia) peristaltic power of the body of the esophagus
endoscopically: difficulty visualizing the 5. the operation should ensure that the fundoplication
squamocolumnar junction at its normal can be placed in the abdomen without undue
location & appearance of redder mucosa than tension, and maintained there by approximating the
normally seen in lower esophagus crura of the diaphragm above the repair
earliest sign for malignant degeneration:
severe dysplasia or intramucosal
adenocarcinoma C. DIAPHRAGMATIC HERNIA
antireflux surgery is an excellent means of - Types:
long-term control for most patients 1. type I (sliding hernia)
one third of all patients with BE present with upward dislocation of the cardia in the
malignancy posterior mediastinum
o should undergo surveillance with the phrenoesophageal ligament is
biopsy every 2 years stretched but intact
o if (+) low grade dysplasia, increase most common
frequency to 6 months can evolve into a type III hernia
2. Esophageal Adenocarcinoma 2. type II (rolling or paraesophageal or giant
Most important etiologic factor in its hiatal hernia)
development is barretts esophagus upward dislocation of the gastric fundus
3. Respiratory symptoms alongside a normally positioned cardia
TOPNOTCH MEDICAL BOARD PREP SURGERY SUPPLEMENT HANDOUT Page 55 of 85
For inquiries visit www.topnotchboardprep.com.ph or email us at topnotchmedicalboardprep@gmail.com
TOPNOTCH MEDICAL BOARD PREP SURGERY SUPPLEMENT HANDOUT - Jules Lopez,MD-MBA,Teddy
Carpio,MD-MBA
For inquiries visit www.topnotchboardprep.com.ph or email us at topnotchmedicalboardprep@gmail.com
defect in the phrenoesophageal membrane - most common esophageal diverticulum
rare - classified as false diverticulum
more likely to occur in women (4:1) - clinical features: dysphagia associated with the
3. type III (the combined sliding-rolling or mixed spontaneous regurgitation of undigested food,
hernia) halitosis, weight loss, chronic aspiration and repetitive
upward dislocation of both the cardia and the respiratory infection
gastric fundus; therefore - due to weakness of the cricopharyngeal muscle
the esophagogastric junction is in the weakness at the Killians area
mediastinum - Diagnosis: Barium swallow (to exclude neoplasia or
4. type IV: colon, herniates as well (in some ulceration)
classifications) - Treatment:
5. intrathoracic abdomen Pharyngomyotomy: 2 cm or less
the end stage of type I and type II hernias Diverticulectomy/diverticuopexy: >2cm
occurs when the whole stomach migrates up
into the chest by rotating 180 around its G. ACHALASIA
longitudinal axis, with the cardia and pylorus - Characterized by complete absence of peristalsis in the
as fixed points esophageal body and failure of LES relaxation
- most common complications: - Classic triad of symptoms: dysphagia, regurgitation
occult GI bleeding from gastritis and weight loss; also associated with nocturnal asthma
ulceration in the herniated portion of the stomach and foul smelling esophageal contents
gastric volvulus (surgical emergency): or - pathogenesis of achalasia is presumed to be a
Borchardts triad of pain, nausea with inability to neurogenic degeneration, which is either idiopathic or
vomit and inability to pass NGT due to infection.
- Diagnosis: - Can be caused by T. cruzi which demonstrates
Barium esophagogram: for diagnosis of destruction of smooth muscle myenteric auerbachs
paraesophageal hiatal hernia plexus
Fiber-optic esophagoscopy - Diagnosis:
Detection of pouch lined with gastric rugal Barium Esophagogram: dilated esophagus with a
folds lying 2 cm or more above the margins of tapering or other wise known as "bird's beak"
the diaphragmatic crura (identified by having Manometric studies: failure of the LES to relax,
the patient sniff) progressive peristalsis in proximal esophagus (if
- Treatment: surgical late disease)
Important principles Has a 10% chance of developing carcinoma due to
Reduce the hernia contents prolonged mucosal irritation
After reduction, excise the sac - Tx:
The use of mesh can reduce recurrence rates hellers myotomy (surgical myotomy of the LES)
of hernia is > than 8 cm goal of surgery: relieve functional obstruction at
the LES
D. SCHATZKIS RING
- thin submucosal circumferential ring in the lower H. DIFFUSE AND SEGMENTAL ESOPHAGEAL SPASM
esophagus at the squamocolumnar junction, often - characterized by substernal chest pain and/or
associated with a hiatal hernia. dysphagia.
- probably an acquired lesion that can lead to stenosis - The LES in patients with DES usually shows a normal
from chemical-induced injury by pill lodgment in the resting pressure and relaxation on swallowing
distal esophagus, or from reflux-induced injury to the - Diagnosis:
lower esophageal mucosa Manometric studies: frequent occurrence of
- Symptoms: brief episodes of dysphagia during hurried simultaneous waveforms and multipeaked
ingestion of solid foods. esophageal contractions, which may be of
- Treatment options: dilation alone, dilation with abnormally high amplitude or long duration.
antireflux measures, antireflux procedure alone, Esophagogram: corkscrew esophagus or
incision, and excision of the ring pseudodiverticulosis

E. SCLERODERMA OF ESOPHAGUS I. NUTCRACKER ESOPHAGUS


- Scleroderma is a systemic disease accompanied by - Other name: supersqueezer esophagus
esophageal abnormalities in approximately 80% of - most common of the primary esophageal motility
patients. disorders
- onset of the disease is usually in the third or fourth - characterized by peristaltic esophageal contractions
decade of life, occurring twice as frequently in with peak amplitudes greater than two SDs above the
women as in men. normal values (up to 400 mmHg)
- Small vessel inflammation appears to be an - Treatment in these patients should be aimed at the
initiating event, with subsequent perivascular treatment of GERD
deposition of normal collagen, which may lead to
vascular compromise. J. HYPERTENSIVE LES
- Muscle ischemia due to perivascular compression has - This disorder is characterized by an elevated basal
been suggested as a possible mechanism for the motility pressure of the LES with normal relaxation and normal
abnormality in scleroderma. propulsion in the esophageal body.
- predominant feature at GI tract: smooth muscle - Treatment: Myotomy of the LES may be indicated in
atrophy. patients not responding to medical therapy or dilation.
- Diagnosis:
Manometrically: observation of normal peristalsis K. ESOPHAGEAL DIVERTICULUM
in the proximal striated esophagus, with absent - Classification:
peristalsis in the distal smooth muscle portion Location: proximal, mid, distal
Barium swallow: dilated, barium-filled esophagus, Pathology
stomach, and duodenum, or a hiatal hernia with Pulsion: motor DO
distal esophageal stricture and proximal dilatation Traction: inflammatory DO
- Epiphrenic diverticula:
F. ZENKERS DIVERTICULUM

TOPNOTCH MEDICAL BOARD PREP SURGERY SUPPLEMENT HANDOUT Page 56 of 85


For inquiries visit www.topnotchboardprep.com.ph or email us at topnotchmedicalboardprep@gmail.com
TOPNOTCH MEDICAL BOARD PREP SURGERY SUPPLEMENT HANDOUT - Jules Lopez,MD-MBA,Teddy
Carpio,MD-MBA
For inquiries visit www.topnotchboardprep.com.ph or email us at topnotchmedicalboardprep@gmail.com
terminal 3rd of the thoracic esophagus & are usually antiemetics
found adjacent to the diaphragm
associated with distal esophageal muscular N. CAUSTIC INJURY
hypertrophy, esophageal motility abnormalities, - Alkalies vs acids
and increased luminal pressure Alkalies dissolve tissue, and therefore
considered as "pulsion" diverticula penetrate more deeply (more serious)
classified as false diverticulum (pouch of mucosa acids cause a coagulative necrosis that limits
that is protruding in the wall of the esophagus) their penetration
- Midesophageal or traction diverticula - The strength of esophageal contractions varies
Classified as true diverticulum (composed of all according to the level of the esophagus
layers of the esophageal wall) - weakest at the striated musclesmooth muscle interface
noted in patients who had mediastinal LN slower clearance allowing caustic substances to
involvement with tuberculosis, mediastinal
remain in contact with the mucosa longer explains
lymphadenopathy, such as pulmonary fungal
why the esophagus is preferentially and more severely
infections (e.g., aspergillosis), lymphoma, or
affected at this level than in the lower portions.
sarcoid
- Phases of injury:
acute necrotic phase:
L. ESOPHAGEAL PERFORATION
lasting 1 to 4 days after injury
- true emergency.
coagulation of intracellular proteins results in
- It most commonly occurs following diagnostic or
cell necrosis
therapeutic procedures (endoscopy).
living tissue surrounding the area of necrosis
- Boerhaave's syndrome: spontaneous perforation,
develops an intense inflammatory reaction.
- Clinical features: chest pain (very striking and
ulceration and granulation phase:
consistent symptom), fever, tachycardia, subcutaneous
3 to 5 days after injury
emphysema, dysphagia, dysnea
Considered a quiescent period because
- Diagnosis:
symptoms seem to disappear
contrast esophagogram with water soluble (like
the superficial necrotic tissue sloughs, leaving
gastrografin) medium: (+) extravasation
an ulcerated, acutely inflamed base, and
(diagnostic)
granulation tissue fills the defect left by the
chest xray: air or effusion in pleural space,
sloughed mucosa.
mediastinal or cervical emphysema
This phase lasts 10 to 12 days
- treatment:
period that the esophagus is the weakest
the incidence of mortality is related to the time
cicatrization and scarring
interval between perforation and treatment; hence
begins third week following injury.
the key to optimum management is early
previously formed connective tissue begins to
diagnosis.
contract, resulting in narrowing of the
The most favorable outcome is obtained following
esophagus
primary closure of the perforation within 24
characterized by dysphagia
hours, resulting in 80 to 90% survival.
It is during this period that efforts must be
The most common location for the injury is the left
made to reduce stricture formation.
lateral wall of the esophagus, just above the GEJ
- Clinical features: pain in the mouth and substernal
- non-operative management
region, hypersalivation, pain on swallowing, and
usually follows an injury occruing during dilation
dysphagia, fever (strongly correlated with the
of esophageal strictures or pneumatic dilations of
presence of an esophageal lesion)
achalasia
- Diagnosis: early esophagoscopy is advocated to
indications
establish the presence of an esophageal injury
1. barium swallow must show the perforation to
To lessen the chance of perforation, the scope
be contained within the mediastinum and
should not be introduced beyond the proximal
drain well back into the esophagus
esophageal lesion.
2. mild symptoms
- Treatment:
3. minimal evidence of clinical sepsis
- approach: Figure 32. algorithm for acute caustic injury
hyperalimentation
antibiotics
cimetidine: to decreased acid secretion, diminish
pepsin activity

M. MALLORY WEISS SYNDROME


- Longitudinal tear in the mucosa of the GE junction
- Characterized by acute upper GI bleeding caused by
forceful vomiting and/or retching
- Commonly seen in alcoholics
- arterial massive
- mechanism: an acute increase in intra-abdominal
pressure against a closed glottis in a patient with a
hiatal hernia.
- Diagnosis:
requires a high index of suspicion (the pattern of
sudden upper GI bleeding following prolonged
vomiting or retching is indicative).
Upper endoscopy: longitudinal fissures in the
mucosa of the herniated stomach as the source of
bleeding.
O. ESOPHAGEAL CARCINOMA
- Treatment:
- Squamous carcinoma accounts for the majority of
bleeding will stop 90% of the time
esophageal carcinomas worldwide.
spontaneously with nonoperative management.
- Risk factors:
Decompression

TOPNOTCH MEDICAL BOARD PREP SURGERY SUPPLEMENT HANDOUT Page 57 of 85


For inquiries visit www.topnotchboardprep.com.ph or email us at topnotchmedicalboardprep@gmail.com
TOPNOTCH MEDICAL BOARD PREP SURGERY SUPPLEMENT HANDOUT - Jules Lopez,MD-MBA,Teddy
Carpio,MD-MBA
For inquiries visit www.topnotchboardprep.com.ph or email us at topnotchmedicalboardprep@gmail.com
nitroso compounds in pickled vegetables and - Coexisting abnormalities include cardiac defects in
smoked meats 38%, skeletal defects in 19%, neurologic defects in 15%,
zinc & molybdenum deficiency renal defects in 15%, and anorectal defects in 8%
smoking (more squamous CA) - Initial management includes attention to respiratory
alcohol consumption (more squamous CA) status, decompression of the upper pouch and
achalasia appropriate timing of surgery
lye strictures Place the neonate in an infant warmer with the
tylosis (an autosomal dominant disorder head elevated at least 30 degrees
characterized by hyperkeratosis of the palms and - Definitive repair is achieved through a primary
soles) esophagoesophagostomy
human papillomavirus. Anastomotic leakage occurs in 10-15% of patients
Barretts esophagus (more adenocarcinoma) Early leakage: pleural effusion, pneumothorax
- Most common presenting symptom: dysphagia and sepsis; revision of anastomosis, cervical
(already a late symptom) esophagostomy and gastrostomy placement is
- Diagnosis: required
barium esophagogram (if with lesion) upper Late leakage: usually heals without
endoscopy intervention; administration of broad
CT scan of chest and abdomen: delineate the tumor spectrum antibiotics, pulmonary toiler and
and detect distant pulmonary or hepatic metastasis optimization of nutrition are important
- Characteristics based on tumor location and treatment:
Cervical esophagus (proximal 1/3)
Almost always squamous carcinoma Survival prediction in patients with Esophageal Atresia
Frequently unresectable because of early Group Survival Waterston Classification
invasion of larynx, great vessels or trachea Rate
Tx: stereotactic radiation with concomitant A 100% Birth wt > 2500 gms, otheriwse healthy
chemotherapy B 85% Birth wt 2000-2500 gms, otherwise well
thoracic esophagus (middle 1/3) OR higher wt with moderate anomaly
almost always squamous carcinoma with LN (non cardiac anomaly plus PDA, VSD,
metastasis ASD)
tx: video assisted thoracic surgery (VATS) C 65% Birth wt <2000 gm, otherwise well OR
thoracotomy hight wt with severe associated cardiac
distal 1/3 or near/at cardia anomaly
almost always adenocarcinoma
tx: curative resection requires cervical division
of esophagus + >50% proximal gastrectomy
- Staging: REVIEW QUESTIONS a
Stage 0: in situ, high-grade dysplasia, no LN mets
Stage I: invaded lamina propria 1. a 4 year old child is brought to the ER 15 minutes
Stage IIA: invaded esophageal wall but not surrounding after accidentally ingesting a drain cleaner. The
structures child exhibits a hoarse voice and is stridorous.
Stage IIB: LN (+); primary tumor has only invaded Which of the following apply?
submucosa or muscularis propria
Stage III: invaded the adventitia and surrounding a. Laryngeal ulceration
structures (pericardium, pleura and aorta) b. Instillation of vinegar into the stomach
Stage IV: (+) metastasis c. Immediate fiberoptic endoscopy
- Clinical factors that indicate advanced stage (and d. tracheostomy
therefore exclude surgery for curative intent):
Horners syndrome Answer: A & D
Persistent spinal pain This is a case of caustic ingestion. Since the child
Paralysis of diaphragm already exhibits laryngeal and epiglottic edema,
Fistula formation preservation of the airway must be the priority.
Malignant pleural effusion Therefore, endoscopy is deferred.
- Treatment:
Surgery: 2. A 50 year old healthy man is brought to the ER
Ivor lewis procedure: with retching followed by hematemesis.
primarily for middle esophageal lesion; all
LNs are removed en bloc with the lesser a. Treatment is by balloon tamponade
curvature of the stomach b. Bleeding often stops spontaneously
most radical highest number of c. It is not caused by forceful vomiting
complication rate d. There is air in the mediastinum
Left thoracoabdominal approach e. Diagnosis is not made by endoscopic
excellent exposure of distal esophagus examination
Transhiatal blunt resection: resection of the
thoracic esophagus from abdomen with Answer: B
subsequent pull-up of stomach and This is a case of Mallory-weis tear. The
esophagogastric anastomosis in the neck mechanism is similar to boerhave syndrome
Goes against the principle of en-bloc (postemetic esophageal rupture) in which there
resection of cancer surgery is associated perforation and vomiting against a
Minimized morbidity and mortality closed cardia. It is diagnosed by endoscopic
compared to the other procedures examination, and the bleeding usually stops
spontaneously. Because the bleeding is arterial, a
pressure tamponade (i.e. Sengstaken-blakemore
P. Esophageal Atresia tube) does not help and may lead to esophageal
- Diagnosis is confirmed by the inability to pass an disruption. If bleeding does not stop, gastrotomy
orogastric tube into the stomach and oversewing of the bleeding point is the
proper therapy, although nonsurgical

TOPNOTCH MEDICAL BOARD PREP SURGERY SUPPLEMENT HANDOUT Page 58 of 85


For inquiries visit www.topnotchboardprep.com.ph or email us at topnotchmedicalboardprep@gmail.com
TOPNOTCH MEDICAL BOARD PREP SURGERY SUPPLEMENT HANDOUT - Jules Lopez,MD-MBA,Teddy
Carpio,MD-MBA
For inquiries visit www.topnotchboardprep.com.ph or email us at topnotchmedicalboardprep@gmail.com
alternatives, such as endoscopic injection of 3. Posterior branch: Criminal nerve of Grassi
epinephrine and cautery have been attempted. (posterior fundus)
easily missed during truncal or highly
3. After diagnostic esophagoscopy, a patient selective vagotomy (HSV).
complains of odynophagia and chest pain, but - Gastric relaxation is due to CCK, distention of
results of water-soluble contrast swallow are duodenum and presence of glucose in the duodenum
negative. Which of the following apply?
MUST KNOW a
a. Discharge the patient if ECG is normal.
b. Use of barium in the chest is devastating Atonic gastritis and abnormal distention and failure to
c. Esophageal manometry should be empty of the stomach can occur in the postoperative
performed immediately patient due to electrolyte disturbances, hyperglycemia
d. Repeat swallow with barium and uremia.
Answer: D
Chest pain, fever, tachycardia, subcutaneous Gastric ulcers located in the PYLORUS are associated with
emphysema, dysphagia and dyspnea are typical increased gastric production (see below Type II & III
of esophageal perforation. Perforation may ulcers)
result from iatrogenic operations, external
trauma, primary esophageal disease or
postemetic (spontaneous) esophageal
hypertension. The incidence of mortality from B. DIAGNOSTIC TEST FOR STOMACH
esophageal perforation is clearly related to the
time interval between perforation and definitive 1. EGD
treatment. Whenever perforation is suspected, a - patients with one or more of the alarm symptoms must
contrast study should be performed with water- undergo immediate upper endoscopy
soluble contrast material. However, if this study
does not demonstrate the perforation, it should Table 53. Alarm symptoms
be repeated with barium. Although barium is Alarm symptoms that indicate the need for upper endoscopy
contraindicated in the presence of colonic Weight loss
injuries because of the harmful effects of feces Recurrent vomiting
Dysphagia
and barium, it does not cause a problem in the
Bleeding
chest. Barium is more accurate than water in Anemia
detecting esophageal leakage. Contrast studies
are important not just for verifying esophageal - requires an 8 hour fasting
rupture but also for documenting the level of - more sensitive than double contrast upper GI series
injury, which has important implications for - most serious complication: esophageal perforation
treatment.
2. Radiologic tests

Plain abdominal xray


- helpful in the diagnosis of gastric perforation
(pneumoperitoneum) or delayed gastric emptying
(large air-fluid level)
STOMACH
Double contrast upper GI series
A. Anatomy - better then EGD in detecting the ff: diverticula, fistula,
B. Diagnostic tests for stomach tortuisity or stricture location, and size of hiatal hernia
C. Peptic ulcer disease
D. Zollinger-Ellison syndrome 3. CT and MRI
E. Gastritis - is part of routine staging work-up for most patients
F. stress ulcer with a malignant gastric tumor
G. Malignant neoplasms of the stomach
H. Benign gastric neoplasms: polyps 4. Gastric secretory analysis
I. Gastric volvulus - maybe useful in the evaluation of patients with
J. Postgastrectomy problems hypergastrinemia, including Zollinger-Ellison
syndrome, patients with refractory ulcer or GERD or
recurrent ulcer after operation

A. ANATOMY 5. Tests for Helicobacter pylori


- Stomach is composed of 3 smooth muscle layers:
1. Outler longitudinal layer greater and lesser Serologic test for H. pylori
curvatures of the stomach - a positive test is a presumptive evidence of active
2. Middle circular pylorus infection if the patient has never been treated for H.
3. Inner oblique pylori infection
- Majority of parietal cells are in the Body of the
stomach Histologic examination of antral mucosal biopsy (with
- Largest artery to the stomach is the left gastric artery special stains)
(from the celiac trunk) - gold standard for H. pylori
- Gastric contraction is via the vagus nerve (primarily Urease breath test
due to parasympathetic fibers) - standard test for to confirm eradication of H. pylori
1. The vagus nerves forms LARP (left:anterior & post-treatment
right:posterior) at the esophageal hiatus as it - basis: the patient ingests urea labeled with
descends from the mediastinum nonradioactive 13C labeled urea is acted upon by the
2. Anterior branch of vagus: nerves of Laterjet urease present in H. pylori converts urea into
they send segmental branches to the body of ammonia and carbon dioxide radiolabeled carbon
the stomach before they terminate near the
angularis incisura as the "crow's foot
TOPNOTCH MEDICAL BOARD PREP SURGERY SUPPLEMENT HANDOUT Page 59 of 85
For inquiries visit www.topnotchboardprep.com.ph or email us at topnotchmedicalboardprep@gmail.com
TOPNOTCH MEDICAL BOARD PREP SURGERY SUPPLEMENT HANDOUT - Jules Lopez,MD-MBA,Teddy
Carpio,MD-MBA
For inquiries visit www.topnotchboardprep.com.ph or email us at topnotchmedicalboardprep@gmail.com
dioxide is excreted from lungs and is detected in Type III: prepyloric ulcer disease; associated with normal or
expired air. increased gastric acid secretion
Type IV: occur near the GE junction, and acid secretion is normal
or below normal
MUST KNOW a Type V: NSAID induced, can occur anywhere in the stomch

H.pylori has the enzyme urease, which converts urea into MUST KNOW a
ammonia and bicarbonate, thus creating an environment
around the bacteria that buffers the acid secreted by the Curling ulcers: peptic ulcers formed after severe burn
stomach. injury
Cushings ulcers: peptic ulcers formed after severe brain
damage

H. pylori fecal antigen test


- sensitive and specific for active H. pylori infection - Pathophysiology, Clinical manifestations, diagnosis
- can also be used to confirm cure and treatment

C. PEPTIC ULCER DISEASE Table 54: Comparison between gastric vs duodenal ulcer
- focal defects in the gastric or duodenal mucosa that Gastric ulcer Duodenal ulcer
extend into the submucosa or deeper Pathophysiology H.pylori, overuse of acid production &
- caused by an imbalance between mucosal defenses and NSAIDS & steroids H.pylori
Clinical Sharp burning pain Severe epigastric
acid/peptic injury.
manifestation in epigastrium pain 2-3 hours after
- Etiology shortly after eating; eating; epigastric
H. Pylori: associated with both gastric and nausea, vomiting pain can also
duodenal ulcer but is a higher predictor of and anorexia awaken them from
duonal ulcer formation sleep
NSAID patients taking NSAID and/or aspirin need Diagnosis Endoscopy and Endoscopy, history,
acid suppressing medication if any of the ff are biopsy (must for all PE, test for H pylori
present: age over 60 yo, hx of PUD, concomitant gastric ulcers to rule
steroid/anticogualant/high dose NSAIDs intake out cancer; test for
H.pylori)
Smoking largest positive predictor of risk (also
Best test to confirm eradication of H.
with alcoholic drinking) pylori: negative urea breath test
Stress both physiologic and psychologic stress treatment Triple therapy; PPI, Triple therapy; stop
Others antacids and H2 smoking, alcohol
More common in Type A personality blockers consumption
Sex: duodenal ulcer is twice more common in
males; same incidence between sexes for - More than 90% of patients with PUD complain of
gastric ulcer abdominal pain (non-radiating, burning in quality &
Blood type: epigastriac in location)
Type O: duodenal ulcer - Indication for endoscopy in PUD:
Type A: gastric ulcer Any symptomatic patient 45 yo and up
- Types of ulcer based on location and Any symptomatic patient regardless of age with
pathophysiology alarm symptoms (see table 54)
Duodenal ulcers patients have daytime and - Medical treatment for PUD: PPIs are the mainstay of
nocturnal acid secretion, BAO and MAO, gastric therapy for PUD.
emptying compared to gastric ulcer patients
Gastric ulcers patients have variable patterns of table 50. treatment regimens for H. pylori
secretion PPI + clarithromycin 500 mg BID + amoxicillin 1000 mg 10-14 d
BID
Figure 33. Modified Johnson classification of gastric ulcer PPI + clarithromycin 500 mg BID + metronidazole 500 mg 10-14 d
BID
PPI + + amoxicillin 1000 mg BID, then 5d
PPI + clarithromycin 500 mg BID + tinidazole mg BID
Salvage regimens for patients who fail one of the above initial
regimens
Bismuth subsalicylate 525 mg qid + metronidazole 250 mg 10-14 d
qid + tetracycline 500 mg qid + PPI
PPI + amoxicillin 1000 mg bid + levofloxacin 500 mg daily 10 d

- Indications for surgical treatment for PUD


bleeding
perforation
obstruction
intractability or nonhealing ulcers (with discretion)
- For nonhealing PUD
Rare indication for surgery
Consider possible differentials for nonhealing PUD
first
Surgical treatment is considered in patients with
nonhealing or intractable PUD who have multiple
recurrences, large ulcers (>2 cm), complications
(obstruction, perforation, or hemorrhage), or
suspected malignancy
Type I: located near the angularis incisura on the lesser
curvature; usually have normal or decreased acid secretion; most - Complications of PUD:
common
Type II: same with type I but with an associated active or
quiescent duodenal ulcer; associated with normal or increased
gastric acid secretion
TOPNOTCH MEDICAL BOARD PREP SURGERY SUPPLEMENT HANDOUT Page 60 of 85
For inquiries visit www.topnotchboardprep.com.ph or email us at topnotchmedicalboardprep@gmail.com
TOPNOTCH MEDICAL BOARD PREP SURGERY SUPPLEMENT HANDOUT - Jules Lopez,MD-MBA,Teddy
Carpio,MD-MBA
For inquiries visit www.topnotchboardprep.com.ph or email us at topnotchmedicalboardprep@gmail.com
- The inherited or familial form of gastrinoma is
Table 55: Comparison of complications of PUD associated with multiple endocrine neoplasia type 1
Bleeding PUD Perforation Gastric outlet or MEN1 (parathyroid, pituitary, and pancreatic or
Obstruction duodenal tumors)
-most common -2nd most common -rare (5% of all PUD
cause of ulcer complication of PUD complications) PHYSIOLOGY a
related death -classic symptom: -usually due to Gastrin
-most common patient can duodenal or
- produced by antral G cells
cause of UGIB in remember the exact prepyloric disease
admitted patients time of onset of -presents with - major hormonal stimulant of acid secretion during
-presents with abdominal pain bilous vomiting, the gastric phase.
melena, -presents acute profound - The biologically active pentapeptide sequence at the
hematemesis, shock abdomen with hypochloremic, C-terminal end of gastrin is identical to that of CCK
-abdominal pain is peritoneal signs metabolic alkalosis - Luminal peptides and amino acids are the most
uncommon (+)pneumoperitoneum -tx: nasogastric potent stimulants of gastrin release
-tx: acid on upright chest xray suction, IV - luminal acid is the most potent inhibitor of gastrin
suppression and (80% of patients) hydration and
secretion.
NPO, transfusion -Tx: analgesia, electrolyte
and endocopic tx antibiotics, isotonic repletion, and - principal mediator of gastrin-stimulated acid
(electrocautery + fluid resuscitation, antisecretory production is histamine from mucosal ECL cells
epi) for high risk immediate OR medication, OR
group - rule out
pancreatic, gastric - Gastrinoma triangle (or Pasaros triangle) : where
& duodenal CA as a
cause of 90% of ZES tumors are found
obstruction boundaries: jxn of cystic & CBD, confluence of 2nd &
3rd segments of the duodenum and jxn of body and
MUST KNOW a neck of pancreas
- most common symptoms of ZES are epigastric pain,
High risk lesions for massive bleeding (based on location): GERD & diarrhea. Can also be associated with
-posterior duodenal ulcer with erosion of gastroduodenal steatorrhea and other symptoms of malabsorption.
artery - Diagnosis:
-lesser curvature gastric ulcer with erosion of left gastric Fasting gastrin of 1mg/L, BAO >15 mEq/h or >5
artery or branch mEq/h (if with previous procedure for peptic
ulcer) are suggestive of ZES
Confirmatory test: secretin stimulation test
(+) secretin stimulation test: paradoxical rise
- Indications for surgery in PUD are bleeding, perforation,
in gastrin levels (200 pg/mL or greater) upon
obstruction and intractability or nonhealing.
administration of IV bolus of secretin (an
- Most patient undergoing operation for PUD have simple
inhibitor of gastrin)
oversewing of a bleeding ulcer, or simple patch of a
Should also check for serum calcium and PTH
perforated ulcer or distal gastrectomy levels to rule out MEN1.
- Surgical options for PUD Preoperative imaging of choice for gastrinoma:
1. HSV or parietal cell vagotomy or proximal gastric somatostatin receptor scintigraphy (octreotide
vagotomy scan)
- safe (mortality risk <0.5%) with minimal side effects Basis: Gastrinoma cells contain type 2
- done by severing the vagal nerve supply to the proximal somatostatin receptors that bind the indium-
2/3 of the stomach (where essentially all parietal cells labeled somatostatin analogue (octreotide)
are located) & preserves the vagal innervation to the with high affinity, making imaging with a
antrum and pylorus and remaining abdominal viscera. gamma camera possible
2. Taylor procedure
- posterior truncal vagotomy and anterior seromyotomy
- attractive to HSV with similar results
PHYSIOLOGY a
Somatostatin
3. Vagotomy + drainage (V+D) procedures
- produced by D cells located throughout the gastric
- Truncal vagotomy dennervates the antrapyloric
mucosa.
mechanism, therefore, some sort of procedure is needed
- major stimulus for somatostatin release is antral
to bypass or ablate the pylorus
acidification
- Types:
- acetylcholine inhibits its release
Truncal vagotomy and pyroplasty
- Somatostatin effects: inhibits acid secretion from
Pyroplasty useful in patients who require
parietal cells, inhibits gastrin release from G cells
pyloroduodenotomy to deal with the ulcer
& decreases histamine release from ECL cells.
complication (i.e. posterior bleeding duodenal
- Octreotide is a somatostatin analogue
ulcer), limited focal a scarring in the pyloric
region
Truncal vagotomy and gastrojejunostomy
Good choice in patients with gastric outlet - Treatment:
obstruction Surgical resection of gastrinoma
If (+) MEN1, perform parathyroidectomy 1st
- disadvantage: 10% of significant dumping / diarrhea before resection of gastrinoma
PPI for symptomatic relief
4. Vagotomy and distal gastrectomy
E. GASTRITIS
D. ZOLLINGER-ELLISON SYNDROME - Definition: Mucosal inflammation
- uncontrolled secretion of abnormal amounts of - Most common cause: H. pylori
gastrin by a duodenal or pancreatic neuroendocrine Other causes: alcohol, NSAIDs, Crohn's disease,
tumor (i.e., gastrinoma) leading to excessive tuberculosis, and bile reflux
production of HCl by the parietal cells, further - Pathophysiology:
excacerbating PUD. infectious and inflammatory causes: result in
- Most common symptoms are epigastric pain, GERD and immune cell infiltration and cytokine production
diarrhea. More than 90% of patients have peptic ulcer, which damage mucosal cells.
most are in the typical location (proximal duodenum)
TOPNOTCH MEDICAL BOARD PREP SURGERY SUPPLEMENT HANDOUT Page 61 of 85
For inquiries visit www.topnotchboardprep.com.ph or email us at topnotchmedicalboardprep@gmail.com
TOPNOTCH MEDICAL BOARD PREP SURGERY SUPPLEMENT HANDOUT - Jules Lopez,MD-MBA,Teddy
Carpio,MD-MBA
For inquiries visit www.topnotchboardprep.com.ph or email us at topnotchmedicalboardprep@gmail.com
chemical agents (alcohol, aspirin, and bile): disrupt chronic occult blood loss (iron deficiency
the mucosal barrier, allowing mucosal damage by anemia and heme+ stool)
back diffusion of luminal hydrogen ions. Dysphagia: if the tumor involves the cardia of
the stomach.
F. STRESS ULCER Paraneoplastic syndromes - Trousseau's
- Pathophysiology: due to inadequate gastric mucosal syndrome (thrombophlebitis), acanthosis
blood flow during periods of intense physiologic nigricans (hyperpigmentation of the axilla and
stress. groin), or peripheral neuropathy can be
Adequate mucosal blood flow is important to present.
maintain the mucosal barrier, and to buffer any Physical examination:
back-diffused hydrogen ions. When blood flow is Enlarged Cervical, supraclavicular (on the left
inadequate, these processes fail and mucosal referred to as Virchow's node), and axillary
breakdown occurs lymph nodes
Sister Josephs nodule: palpable umbilical
G. MALIGNANT NEOPLASMS OF THE STOMACH nodue; pathognomonic for advanced
- The three most common primary malignant gastric disease
neoplasms are adenocarcinoma (95%), lymphoma Blumer nodes: palpable nodularity in the
(4%), and malignant GIST (1%) pouch of douglas; evidence of drop
metastasis
GASTRIC ADENOCARCINOMA
- Epidemiology & etiology - Diagnosis
Gastric adenoCA is a disease of the elderly Do endoscopy and biopsy
Risk factors: Pre-operative staging: abdominal/pelvic CT
Black race: twice more common in blacks scanning with IV and oral contrast
compared to whites
Pernicious anemia - Treatment
Blood group A Surgery is the only curative treatment for gastric
FH of gastric CA cancer (radical subtotal gastrectomy)
Diet: starchy diet high in pickled, salted, or Goal in resecting gastric adenocarcinoma: grossly
smoked food, nitrates increases risk negative margin of at least 5 cm to achieve R0
H. pylori resection
Smoking
EBV infections
Remember: Alcohol has no role in gastric CA GASTRIC LYMPHOMA
protective factors: aspirin (Yes! Schwartz says so. - stomach is the most common site of primary GI
You dont believe me? Check p. 927, 9th edition), lymphoma
vitamin C and diet high in fruits and vegetables - over 95% are non-Hodgkin's type.
premalignant conditions: - Most are B-cell type, thought to arise in MALT
polyps - MALT lymphomas is a form of NHL arising from the B
hyperplastic and adenomas are the cells in the marginal zone of MALT
types associated with carcinoma - Is associated with chronic inflammation due to H. pylori
- Diligent search for extragstric disease should be done
inflammatory, hamartomatous and
heterotropic polyps are considered before giving a diagnosis of primary gastric lymphoma
benign lesions - Treatment: chemotx is equivalent to surgery
atrophic gastritis: most common
GASTROINTESTINAL STROMAL TUMOR (GIST)
precancerous lesion / precursor of gastric
- Are submucosal solitary slow growing tumors arising
cancer
from interstitial cells of Cajal (ICC)
intestinal metaplasia: can be caused by H.
- 2/3 of all GISTs occur in the stomach, occurring
pylori
commonly in the body
- Pathology
- defining feature of GISTS is their gain of function
Gastric Dysplasia: universal precursor to
mutation of protooncogene KIT, a receptor tyrosine
gastric adenocarcinoma
kinase (majority of GISTS have activated mutation in
Early gastric cancer: adenocarcinoma limited to
the c-kit protooncogene, which causes KIT to be
the mucosa and submucosa of the stomach,
constitutively activated, presumably leading to
regardless of lymph node status.
persistence of cellular growth or survival signals)
4 forms of gastric cancer (Gross morphology):
- Epithelial cell stromal GIST: most common cell type
1. Polypoid: bulk of tumor is intraluminal, not ulcerated
arising in the stomach; cellular spindle type is the next
2. Fungating: bulk of tumor is intraluminal, ulcerated
most common; glomus tumor type is seen only in the
3. Ulcerative: bulk of tumor is within the stomach wall
stomach.
4. Scirrhous (linitis plastic): bulk of tumor is within the
- Markers: (+) c-KIT, a protooncogene; a characteristic
stomach wall; infiltrate the entire thickness of stomch
shared with ICC
and cover a large surface area, poor prognosis
- Diagnosis: endoscopy and biopsy,
Location of primary tumor: 40% distal stomach,
- Mode of metastasis: hematogenous route; most
30% middle stomach and 30% proximal stomach
common sites: liver and lung
Most important prognosticating factors: lymph
- Treatment:
node involvement and depth of tumor invasion
Wedge resection with clear margins is adequate
surgical treatment
- Clinical manifestations:
Imatinib (Gleevec): a chemotherapeutic agent
Most patients diagnosed with gastric CA have
that blocks the activity of the tyrosine kinase
advanced stage III or IV disease
product of c-kit, is reserved for metastatic or
S/Sx:
unresectable GIST. benign gastric neoplasms
weight loss and decreased food intake due
to anorexia and early satiety (most
KRUKENBERG TUMOR
common)
- also known as Carcinoma mucocellulare
Abdominal pain (usually not severe and often
- refers to the signet ring subtype of metastatic tumors of
ignored)
the ovary
nausea, vomiting, & bloating.
Acute GI bleeding (unusual)
TOPNOTCH MEDICAL BOARD PREP SURGERY SUPPLEMENT HANDOUT Page 62 of 85
For inquiries visit www.topnotchboardprep.com.ph or email us at topnotchmedicalboardprep@gmail.com
TOPNOTCH MEDICAL BOARD PREP SURGERY SUPPLEMENT HANDOUT - Jules Lopez,MD-MBA,Teddy
Carpio,MD-MBA
For inquiries visit www.topnotchboardprep.com.ph or email us at topnotchmedicalboardprep@gmail.com
- colon and stomach are the most common primary GASTRIC OUTLET OBSTRUCTION (see complications
tumors to result in ovarian metastases (followed by the of PUD as well)
breast, lung, and contralateral ovary - presents with hypochloremic, hypokalemic
metabolic alkalosis dehydration
- as a compensatory response due to worsening
H. BENIGN GASTRIC NEOPLASMS: POLYP (see also dehydration, Na conservation occurs in the kidney,
premalignant conditions of gastric adenoCA) leading to renal tubular acidosis with subsequent
- most common benign tumor of the stomach aciduria
- 5 types:
1. Adenomatous: (+) malignant potential; 10-15% of all
gastric polyps POST-GASTRECTOMY PROBLEMS
2. hyperplastic (regenerative): most common gastric (1) Dumping Syndrome: phenomenon caused by the
polyp (75% of all gastric polyps); occurs in the destruction or bypass of the pyloric sphincter. Symptoms
setting of gastritis and has a low malignant potential are thought to be the result of the abrupt delivery of a
3. hamartomatous: benign hyperosmolar load into the small bowel. Medical therapy
4. inflammatory: benign consists of dietary management and somatostatin
5. heterotopic (e.g., ectopic pancreas): benign analogue (octreotide)
***Polyps that are symptomatic, >2 cm, large
a. Early Dumping: diaphoresis, weakness,
hyperplastic or adenomatous should be removed,
lightheadedness, tachycardia, crampy abdominal
usually by endoscopic snare polypectomy.
pain and diarrhea
I. GASTRIC VOLVULUS b. Late Dumping: hypoglycemia and
- is a twist of the stomach that usually occurs in hyperinsulinemia
association with a large hiatal hernia or unusually (2) Diarrhea: may be a result of truncal vagotomy, dumping or
mobile stomach without hiatal hernia. malabsorption
- Gastric volvulus is a chronic condition that can be (3) Gastric Stasis: may be due to a problem with gastric motor
surprisingly asymptomatic. function or be caused by an obstruction.
- Clinical manifestations: abdominal pain and pressure (4) Bile Reflux Gastritis
related to the intermittently distending and poorly (5) Roux Syndrome: post gastrectomy with great difficulty
emptying twisted stomach, dyspnea (due to pressure on with gastric emptying in the absence of mechanical
the lung), palpitations (due to pressure on the obstruction. These patients present with vomiting,
pericardium) and dysphagia (pressure on the epigastric pain and weight loss. Endoscopy may show
esophagus) bezoar formation, dilation of the gastric remnant and/or
- Management: dilatation of the Roux limb.
Vomiting and passage of a NGT may relieve (6) Gallstones
symptoms (7) Weight loss
Gastric infarction is a surgical emergency (8) Anemia
(9) Bone Disease
J. POSTGASTRECTOMY PROBLEMS

DUMPING SYNDROME REVIEW QUESTIONS


- occurs after bariatric surgery and PUD repair (after
pyloroplasty, pyloromyotomy or distal gastrectomy) 1. A patient with a vagotomy and pyloroplasty
- mechanism: there is accumulation of digested food in returns with a recurrent ulcer. The best method
the small intestine (or abrupt delivery of hyperosmolar for determining if there was an inadequate
load into the small bowel) leading to circumferential vagotomy performed is
expansion, additional accumulation of fluids emptying
from stomach to duodenum and sudden expulsion of a. Direct vagal stimulation
food to GIT possibly due to ablation of the pylorus or b. Stimulated gastric analysis
decreased gastric compliance with accelerated c. Stimulated PPI (pancreatic polypeptide)
emptying of liquids (after highly selective vagotomy) levels
- clinical manifestation: tachycardia, crampy abdominal d. None of the above there is no good test to
pain and diarrhea, dizziness, lightheadedness, determine inadequate vagotomy
diaphoresis, nausea and vomiting after ingestion of a
fatty or carbohydrate laden meal Answer: C
due to sudden shift in electrolytes and fluids Historically, gastric analysis was performed most
combined with increased blood flow to small commonly to test for the adequacy of vagotomy
intestine in postoperative patients with recurrent or
- treatment: persistent ulcer. Now this can be done by
decreasing fluid and food intake to small frequent assessing peripheral pancreatic polypeptide
portions levels in response to sham feeding. A 50%
avoid fatty and simple sugars increase in pancreatic polypeptide within 30
minutes of sham feeding suggests vagal integrity.
AFFERENT LIMB OBSTRUCTION (BLIND LOOP 2. Which of the following procedures for PUD has
SYNDROME) the highest incidence of postoperative diarrhea?
- occurs usually after a Billroth II procedure (distal
gastric resection followed by gastrojejunal a. Graham patch
anastomosis) b. Parietal cell vagotomy
- location of obstruction: at the limb associated with the c. Truncal vagotomy and pyloroplasty
gastric remnant going to the duodenum d. Distal gastrectomy without vagotomy
- clinical manifestations: severe epigastric pain following
eating, bilous emesis without food Answer: C
- treatment: convert Billroth II to roux en-Y gastric Parietal Truncal Truncal
bypass (possible problem: can delay gastric emptying) Cell vagotomy & vagotomy &
vagotomy pyloroplasty Antrectomy

TOPNOTCH MEDICAL BOARD PREP SURGERY SUPPLEMENT HANDOUT Page 63 of 85


For inquiries visit www.topnotchboardprep.com.ph or email us at topnotchmedicalboardprep@gmail.com
TOPNOTCH MEDICAL BOARD PREP SURGERY SUPPLEMENT HANDOUT - Jules Lopez,MD-MBA,Teddy
Carpio,MD-MBA
For inquiries visit www.topnotchboardprep.com.ph or email us at topnotchmedicalboardprep@gmail.com
Operative
mortality
0 <1 1 Cholecystokinin I cell Stimulates exocrine
rate (%) pancreatic secretion;
Ulcer 5-15 5-15 <2 Stimulates GB emptying;
recurrence
(%) Inhibits sphincter of Oddi
Dumping(%) contraction
Motilin M cell Stimulates intestinal motility
Mild <5 10 10-15
Glucagon-like L cell Stimulates intestinal
Severe 0 1 1-2 peptide 2 proliferation
Peptide YY L cell Inhibits intestinal motility &
Diarrhea
secretion
Mild <5 25 20

Severe 0 2 1-2
B. SMALL BOWEL OBSTRUCTION
- Epidemiology:
most frequently encountered surgical disorder
of the small intestine.
Lesions can be described as:
Intraluminal: foreign bodies, gallstones,
meconium
SMALL INTESTINE Intramural: tumors, Crohns disease
associated inflammatory strictures
A. Gross Anatomy and Histology Extrinsic: adhesions, hernias, carcinomatosis
B. Small bowel obstruction - Etiology:
C. Ileus & other disorders of intestinal motility Intra-abdominal adhesions related to prior
D. Crohns disease abdominal surgery: most common cause (75%
E. Intestinal fistulas of cases)
F. Small bowel neoplasms Hernias
G. Radiation enteritis Malignancy: due to extrinsic compression or
H. Meckels diverticulum invasion by advanced malignancies arising in
I. Acquired diverticulum organs other than the small bowel
J. Mesenteric Ischemia Crohn's disease.
K. Obscure GI bleeding Congenital abnormalities (i.e. midgut volvulus and
L. Intussuception intestinal malrotation) diagnosed at adulthood.
M. Short bowel syndrome superior mesenteric artery syndrome: rare;
compression of the 3rd portion of the duodenum by
the superior mesenteric artery as it crosses over
this portion of the duodenum; seen in young
A. GROSS ANATOMY AND HISTOLOGY asthenic individuals who have chronic symptoms
- raison d'tre of the GI tract because it is the principle suggestive of proximal small bowel obstruction.
site of nutrient digestion and absorption.
- Layers of the small intestine (from innermost to - Pathophysiology
outermost layers): mucosa, submucosa, muscularis Gas (usually from swallowed air) and fluid (from
propria and serosa swallowed liquids and GI secretions) accumulate
Contraction of the inner circular layer causes within the intestinal lumen proximal to the site of
results in luminal narrowing obstruction intestinal activity to overcome the
Contraction of the outer longitudinal layer results obstruction (seen as colicky pain and diarrhea)
in bowel shortening bowel distention intraluminal and intramural
Contraction of the muscularis mucosa contribute to pressures rise intestinal motility is eventually
mucosal or villus motility (but not peristalsis) reduced with fewer contractions If intramural
- Mucosal folds: plicae circulares / valvulae pressure becomes high enough impaired
conniventes intestinal microvascular perfusion intestinal
- Peyers patches: most commonly located in the ileum ischemia necrosis (strangulated bowel
which are aggregates of lymphoid follicles and is a local obstruction)
source of IgA With obstruction, the luminal flora of the small
- Difference between jejunum and ileum: jejunum has bowel (which is usually sterile) changes
larger circumference, thicker wall, less fatty Translocation of these bacteria to regional lymph
mesentery, and longer vasa recta nodes
- Calcium is primarily absorbed in the duodenum
through both transcellular transport and paracellular Partial SBO: only a portion of the intestinal lumen is
diffusion. occluded, allowing passage of some gas and fluid.

Complete SBO: complete occlusion


PHYSIOLOGY a
Closed loop obstruction: dangerous form of SBO, in
which a segment of intestine is obstructed both
Representative Regulatory Peptides produced in the
proximally and distally (e.g., with volvulus). In such
small Intestine:
cases, the accumulating gas and fluid cannot escape
either proximally or distally from the obstructed segment,
Hormone Source Actions
leading to a rapid rise in luminal pressure, and a rapid
Somatostatin D Cell Inhibits GI secretion, progression to strangulation.
motility & splanchnic
perfusion
- Clinical presentation
Secretin (1st S cell Stimulates exocrine Symptoms: colicky abdominal pain, nausea,
hormone discovered in
the human body)
pancreatic secretion; vomiting (a more prominent symptom with
stimulates intestinal proximal obstructions than distal; vomitus is
secretion usually feculent), and obstipation, continued

TOPNOTCH MEDICAL BOARD PREP SURGERY SUPPLEMENT HANDOUT Page 64 of 85


For inquiries visit www.topnotchboardprep.com.ph or email us at topnotchmedicalboardprep@gmail.com
TOPNOTCH MEDICAL BOARD PREP SURGERY SUPPLEMENT HANDOUT - Jules Lopez,MD-MBA,Teddy
Carpio,MD-MBA
For inquiries visit www.topnotchboardprep.com.ph or email us at topnotchmedicalboardprep@gmail.com
passage of flatus and/or stool beyond 6 to 12 hours - Tends to occur following non-abdominal procedures
after onset of symptoms (more for partial SBO than (i.e. cardiac surgery)
complete SBO) - Due to a neurologic dysfunction, electrolyte
Signs: abdominal distention (pronounced if the site abnormality and age
of obstruction is distal ileum & absent if the site of - Treatment: NGT, IV neostigmine, IV atropine (to
obstruction is in the proximal small intestine), counter bradycardia as SE of neostigmine), exploratory
initially hyperactive bowel sounds (maybe minimal laparotomy during worst case scenario)
towards the late stages of bowel obstruction)
Lab findings: hemoconcentration and electrolyte C. ILEUS & OTHER DISORDERS OF INTESTINAL
abnormalities (reflect intravascular volume MOTILITY
depletion) & Mild leukocytosis - Ileus is a temporary motility disorder
Features of strangulated SBO: abdominal pain - Postoperative ileus: most frequently implicated
often disproportionate to the degree of cause of delayed discharge following abdominal
abdominal findings ( suggestive of intestinal operations
ischemia), tachycardia, localized abdominal - Pathophysiology:
tenderness, fever, marked leukocytosis, & acidosis. Common etiologies: abdominal operations,
infection and inflammation, electrolyte
- Diagnosis abnormalities (K, &Mg, Na) & drugs
Confirmatory test: abdominal series (radiograph (anticholinergics, opiates, phenothiazine, CCB,
of the abdomen with the patient in a supine position, Tricyclic antidepressants)
upright position &radiograph of the chest with the Proposed mechanisms: surgical stress-induced
patient in an upright position) sympathetic reflexes, inflammatory response
Sensitivity of abdominal radiographs for mediator release, and anesthetic/analgesic effects
detecting SBO is 70-80% Normal temporal pattern of return of GI
Triad of dilated small bowel loops (>3 cm in motility : small intestinal motility (1st 24
diameter), air-fluid levels seen on upright hours), gastric motility (48 hours) and colonic
films, and a paucity of air in the colon is motility (3 to 5 days)
MOST SPECIFIC - Clinical presentation (usually resembles SBO):
CT scan Inability to tolerate liquids and solids by mouth, nausea,
80 to 90% sensitive and lack of flatus or bowel movements, vomiting,
70 to 90% specific abdominal distention & diminished or absent bowel
Apperance of closed-loop obstruction in sounds
CT: presence of U-shaped or C-shaped dilated - diagnosis: If ileus persists beyond 3 to 5 days
bowel loop associated with a radial postoperatively or occurs in the absence of
distribution of mesenteric vessels converging abdominal surgery, further investigation is warranted
toward a torsion point. to rule out possibility of mechanical obstruction
Appearance of strangulation in CT:
thickening of the bowel wall, pneumatosis CLINICAL PEARLS a
intestinalis (air in bowel wall), portal venous
gas, mesenteric haziness and poor uptake of IV Measures to REDUCE postoperative ileus:
contrast into the wall of the affected bowel. Intraoperative measures:
- minimize handling of bowel
- Treatment - laparascopic approach, if possible
Fluid resuscitation: isotonic replacement - avoid excessive intraoperative fluid administration
Broad spectrum antibiotics
NGT placement for decompression Postoperative measures
If complete SBO, perform surgery - early enteral feeding
If partial SBO, may be approached conservatively - epidural anesthesia, if indicated
given that there is no fever, tachycardia, - avoid excessive IV fluid administration
tenderness, or an increase in white cell count - correct electrolyte abnormalities
(indicates perforation)
- consider m-opiod antagonists (
most patients with partial small obstruction
whose symptoms do not improve within 48 hours ***Remember, though often recommended, the use of early
after initiation of nonoperative therapy should ambulation and routine NG intubation has NOT been
undergo surgery. demonstrated to be associated with earlier resolution of
Obstruction presenting in the early postoperative postoperative ileus.
period (particularly those undergoing pelvic
surgery, especially colorectal procedures) pose
the greatest risk for developing early D. INFLAMMATORY BOWEL SYNDROME: CROHNS
postoperative small bowel obstruction. DISEASE VS ULCERATIVE COLITIS
obstruction should be considered if Sx of
intestinal obstruction occur after the initial table 56. Inflammatory bowel syndrome
return of bowel function or if bowel function Crohns disease Ulcerative colitis
fails to return within the expected 3 to 5 description chronic, idiopathic Chronic inflammatory
days after abdominal surgery. transmural disease affecting only the
Regardless of etiology, the affected intestine should inflammatory disease colonic mucosa and
be examined, and nonviable bowel resected. with a propensity to submucosa
Criteria for viability: normal color affect the distal ileum
Etiology & -more common in Higher chance of leading
(pinkish), (+)peristalsis, and marginal
epidemiology Ashkenazi jews & to colorectal cancer
arterial pulsations. caucasaians, females,
has a bimodal age
distribution (3rd & 6th
decade), (+) strong
Ogilvie syndrome pattern of family
- massive idiopathic non-obstructive dilatation of the inheritance, smokers &
colon; acute colonic pseudo-obstruction higher Socio-eco status
- Distention of the abdomen leading to obstruction Pathology Focal transmural Inflammation is limited to
inflammation, mucosa and submucosa

TOPNOTCH MEDICAL BOARD PREP SURGERY SUPPLEMENT HANDOUT Page 65 of 85


For inquiries visit www.topnotchboardprep.com.ph or email us at topnotchmedicalboardprep@gmail.com
TOPNOTCH MEDICAL BOARD PREP SURGERY SUPPLEMENT HANDOUT - Jules Lopez,MD-MBA,Teddy
Carpio,MD-MBA
For inquiries visit www.topnotchboardprep.com.ph or email us at topnotchmedicalboardprep@gmail.com
aphthous ulcers only; lead pipe colon Decision to do perform surgery or do
(earliest lesion of (lacks haustral conservative treatment
Crohns), markings); no Surgeons usually do 2 to 3 months of
non casseating granulomas
conservative therapy before
granulomas,
cobblestoning, considering surgical intervention.
**fat wrapping This approach is based on evidence that
(encroachment of 90% of fistulas that are going to close,
mesenteric fat onto the close within a 5-week interval
serosal surface of the - Definitive management: surgery (if failure of
bowel): pathognomonic spontaneous closure during time period or with
of crohns complications and risk factors)
spares rectum,can
- rehabilitation
occur anywhere in the
GI tract, skip lesions, Primarily affects the MUST KNOW a
targets terminal ileum colon & rectum and is
continous; can also Remember FRIEND (factors that inhibit spontaneous closure
fistula, manifest with backwash of fistulas):
ileitis Foreign body within the fistula tract
Radiation enteritis
S/Sx Inisiduous onset with bloody diarrhea and
Infection/Inflammation at the fistula origin Epithelialization of
waxing and waning crampy abdominal pain.
course of abdominal Proctitis may produce the fistula tract
pain (usually RLQ), tenesmus; can proceed to Neoplasm at the fistula origin
nonbloody diarrhea & fulminant colitis and toxic Distal obstruction of the intestine
weight loss; megacolon

(+) extraintestinal F. SMALL BOWEL NEOPLASMS


manifestation: arthritis,
- Adenomas are the most common benign neoplasm of
uveitis, iritis, eythema
nodosum, pyoderma
the small intestine
gangrenosum, primary - Most common location for primary adenocarcinoma
sclerosing cholangitis, and adenomas of the small bowel is DUODENUM
nephrolithiasis (EXCEPT in patients with Crohns disease, which is
Diagnosis Endoscopy (skip Endoscopy & proctoscopy found mostly in the ileum)
lesions, cobblestoning, (earliest manifestation is - Primary small bowel cancers are rare; 1.1 to 2.4% of
abscess formation and mucosal edema; mucosal all GI malignancies
fistulas); histology friability ; ulceration; (+) Adenocarcinomas: 35 - 50%
demonstrate Pus and mucus)
Carcinoid tumors: 20 to 40%
granulomas; (+)
(pANCA) and anti Lymphomas: 10 to 15 %
Saccharomyces GISTs: most common location is STOMACH (60-
cerevisiae antibody 70%), 2nd most common location is small intestine
(ASCA (25-35%)
Treatment Sulfasalazine + steroids; Similar to Crohns; - Pathophysiology: proposed explanations for the low
surgery if unresponsive colectomy after 15 years frequency of small intestinal neoplasms
to aggressive medical of symptoms dilution of environmental carcinogens in the liquid
Tx
chyme present in the SI lumen
rapid transit of chime (limiting the contact time
E. INTESTINAL FISTULAS between carcinogens and the intestinal mucosa)
- abnormal communication between two epithelialized relatively low concentration of bacteria in small
surfaces intestinal chime (therefore, low concentration of
- can be internal (within GI tract or adjacent organs)or carcinogenic products of bacterial metabolism)
external (with communication to external environment) mucosal protection by secretory IgA and
- Kinds:
low output fistulas - drain less than 200 mL of hydrolases such as benzpyrene hydroxylase
fluid/day render carcinogens less active
high output fistulas - drain more than 500 mL of efficient epithelial cellular apoptotic mechanisms
fluid/day that serve to eliminate clones harboring genetic
- 80% of enterocutaneous fistulas are due to mutations.
iatrogenic complications
- Clinical presentation
usually become clinically evident between the 5th & - Clinical presentation
10th postop Partial SBO is the most common mode of
initial signs: Fever, leukocytosis, prolonged ileus, presentation
abdominal tenderness, and wound infection Only becomes symptomatic when it becomes large
(+) drainage of enteric material through the - Diagnosis: Because of the absent or nonspecific
abdominal wound or through existing drains: symptoms associated with most small intestinal
associated with intra-abdominal abscesses. neoplasms, these lesions rarely are diagnosed
- Diagnosis preoperatively
CT scan: most useful initial test - Treatment: surgical resection
small bowel series or enteroclysis examination: can
be obtained to demonstrate the fistula's site of G. RADIATION ENTERITIS
origin in the bowel. - An undesired side effect of radiation therapy is
Fistulogram: greater sensitivity in localizing the radiation-induced injury to the small intestine
fistula origin. - The SI is susceptible to radiation-induced injury
- Treatment because it has a high rate of rapidly proliferating
Should follow orderly steps (done to maximize cells compared to the other portions of the GI tract
spontaneous closure) - Pathophysiology
Stabilization: fluid & electrolyte resuscitation, principal mechanism of radiation-induced cell
TPN, antibiotics, death is believed to be apoptosis resulting from
Investigation: see diagnosis
TOPNOTCH MEDICAL BOARD PREP SURGERY SUPPLEMENT HANDOUT Page 66 of 85
For inquiries visit www.topnotchboardprep.com.ph or email us at topnotchmedicalboardprep@gmail.com
TOPNOTCH MEDICAL BOARD PREP SURGERY SUPPLEMENT HANDOUT - Jules Lopez,MD-MBA,Teddy
Carpio,MD-MBA
For inquiries visit www.topnotchboardprep.com.ph or email us at topnotchmedicalboardprep@gmail.com
free-radicalinduced breaks in double-stranded - Treatment: surgical
DNA diverticulectomy
The intensity of injury is related to the dose of If the indication for diverticulectomy is bleeding,
radiation administered segmental resection of ileum that includes both the
- Pathology diverticulum and the adjacent ileal peptic ulcer
acute injury: villus blunting, dense infiltrate of should be performed.
leukocytes and plasma cells within the crypts, Segmental ileal resection may also be necessary if
mucosal sloughing, ulceration, and hemorrhage the diverticulum contains a tumor or if the base of
chronic injury: progressive occlusive vasculitis that the diverticulum is inflamed or perforated.
leads to chronic ischemia and fibrosis that affects The management of incidentally found
all layers of the intestinal wall, rather than the (asymptomatic) Meckel's diverticula is
mucosa alone leading to strictures, abscesses, controversial.
and fistulas
- Clinical presentation I. ACQUIRED DIVERTICULUM
Acute: nausea, vomiting, diarrhea, and crampy - Considered as false diverticula (because their walls
abdominal pain. consist of mucosa and submucosa but lack a complete
Chronic: becomes evident within 2 years of muscularis)
radiation administration, most commonly presents - more common in the duodenum, near the ampulla
with partial small bowel obstruction with (periampullary, juxtapapillary, or peri-Vaterian
nausea, vomiting, intermittent abdominal diverticula)
distention, crampy abdominal pain, and weight loss - Diverticula in the jejunum tend to be large and
The terminal ileum is the most frequently affected accompanied by multiple other diverticula, whereas
segment those in the ileum tend to be small and solitary.
- Diagnosis - Pathophysiology
Enteroclysis: most accurate imaging test for Due to acquired abnormalities of intestinal smooth
diagnosing chronic radiation enteritis, muscle or dysregulated motility leading to
CT scan findings are neither very sensitive nor herniation of mucosa and submucosa through
specific for chronic radiation enteritis; should be weakened areas of muscularis.
obtained to rule out the presence of recurrent - Clinical presentation
cancer (because of overlap in clinical Acquired diverticula are asymptomatic unless
manifestations) associated complications arise
- Treatment: supportive therapy Complications (6 to 10% of patients): intestinal
obstruction, diverticulitis, hemorrhage,
H. MECKELS DIVERTICULUM perforation, and malabsorption.
- most prevalent congenital anomaly of the GI tract - Diagnosis
- considered a true diverticula Most acquired diverticula are discovered
- location is usually found in the ileum within 100 cm of incidentally on radiographic imaging, during
the ileocecal valve endoscopy, or at the time of surgery.
- 60% of Meckel's diverticula contain heterotopic Enteroclysis is the most sensitive test for detecting
mucosa (most common: gastric mucosa 60%; jejunoileal diverticula
others: Pancreatic acini, Brunner's glands, pancreatic - Treatment
islets, colonic mucosa, endometriosis, and hepatobiliary If asymptomatic, observe
tissues). If (+)complications, such as bleeding and
diverticulitis: segmental intestinal resection for
MUST KNOW a diverticula located in the jejunum or ileum.
rule of TWOs of Meckels diverticulum:
J. MESENTERIC ISCHEMIA
2% prevalence (affects 2% of the population)
2:1 female predominance Table 57: comparison of acute vs chronic mesenteric ischemia
location 2 ft proximal to the ileocecal valve in adults Acute mesenteric ischemia Chronic mesenteric
usually 2 inches in length ischemia
Causes: - results from atherosclerotic
sympatomatic lesions usually present before 2 years of age
-arterial embolus: most lesions in the main splanchnic
common cause; most arteries (celiac, superior
- Pathophysiology common source: heart; most mesenteric, and inferior
Failure of the the omphalomesenteric common location: SMA mesenteric arteries
(vitelline) duct to undergo obliteration during -arterial thrombosis: occur -rarely leads to infarction
the 8th week of gestation in proximal mesenteric
- Littres hernia: Meckel's diverticula found in an arteries -Postprandial abdominal
inguinal or femoral hernia sacs; when incarcerated, -vasospasm (nonocclusive pain is the most prevalent
mesenteric ischemia): symptom, producing a
can cause intestinal obstruction
diagnosed in critically ill characteristic aversion to food
- Clinical presentation patients receiving vasopressor ("food-fear") and weight loss
most common presentations associated with agents. (can be mistaken as a
symptomatic Meckel's diverticula: bleeding (most -venous thrombosis: involves symptom of malignancy)
common in pediatric age), intestinal the superior mesenteric vein
obstruction (most common in adults), and in 95% of cases; associated
diverticulitis with heritable or acquired
- Diagnosis coagulation DO
Usually discovered incidentally on radiographic Golden period: 3 hours
imaging, during endoscopy, or at the time of intestinal sloughing; 6 hours:
surgery. full thickness intestinal
CT scan: low sensitivity and specificity infarction
Enteroclysis: has 75% accuracy but u not
applicable during acute presentations Hallmark of acute
Radionuclide scans (99mTc-pertechnetate): positive mesenteric ischemia: Severe
only when the diverticulum contains associated abdominal pain, out of
proportion to the degree of
ectopic gastric mucosa that is capable of uptake of
tenderness on examination
the tracer

TOPNOTCH MEDICAL BOARD PREP SURGERY SUPPLEMENT HANDOUT Page 67 of 85


For inquiries visit www.topnotchboardprep.com.ph or email us at topnotchmedicalboardprep@gmail.com
TOPNOTCH MEDICAL BOARD PREP SURGERY SUPPLEMENT HANDOUT - Jules Lopez,MD-MBA,Teddy
Carpio,MD-MBA
For inquiries visit www.topnotchboardprep.com.ph or email us at topnotchmedicalboardprep@gmail.com
a. Gastrectomy
b. Gastric bypass
c. Ileal resection
d. ALL OF THE ABOVE

Answer: D
Vitamin B12 (cobalamin) malabsorption can
result from a variety of surgical manipulations.
SUPERIOR MESENTERIC ARTERY SYNDROME
The vitamin is initially bound by saliva-derived R
protein. In the duodenum, R protein is
Compression of the third, or transverse, portion of the
hydrolyzed by pancreatic enzymes, allowing free
duodenum between the aorta and the superior mesenteric
cobalamin to bind to gastric parietal cell-derived
artery
intrinsic factor. The cobalamin-intrinsic factor
complex is able to escape hydrolysis by
The superior mesenteric artery usually forms an angle of
pancreatic enzymes, allowing it to reach the
approximately 45 (range, 38-56) with the abdominal aorta,
terminal ileum, which expresses specific
and the third part of the duodenum crosses caudal to the
receptors for intrinsic factor. Subsequent events
origin of the superior mesenteric artery, coursing between
in cobalmin absorption are poorly characterized,
the superior mesenteric artery and aorta. Any factor that
but the intact complex probably enters
sharply narrows the aortomesenteric angle to approximately
enterocytes through translocation. Because each
6-25 can cause entrapment and compression of the third
of these steps is necessary for cobalamin
part of the duodenum as it passes between the superior
assimilation, gastric resection, gastric bypass and
mesenteric artery and aorta, resulting in superior mesenteric
ileal resection can each result in Vitamin B 12
artery syndrome.
insufficiency.
In addition, the aortomesenteric distance in superior
mesenteric artery syndrome is decreased to 2-8 mm (normal 2. Which of the following is the LAST to recover
is 10-20 mm). Alternatively, other causes implicated in from postoperative ileus?
superior mesenteric artery syndrome include high insertion
of the duodenum at the ligament of Treitz, a low origin of the a. Stomach
superior mesenteric artery, and compression of the b. Small Bowel
duodenum due to peritoneal adhesion. c. Colon
d. NONE of the above recovery is simultaneous

K. INTUSSUCEPTION Answer: C
- refers to a condition where one segment of the intestine The return of normal motility generally follows a
becomes drawn in to the lumen of the proximal characteristic temporal sequence, with small
segment of the bowel intestinal motility returning to normal within the
- usually is seen in the pediatric population 1st 24 hours after laparotomy and gastric and
- Adult intussusceptions are rare; usually with distinct colonic motility returning to normal by 48 hours
pathologic lead point (which can be malignant) and 3 to 5 days, respectively. Because small bowel
- commonly present with a history of intermittent motility is returned before colonic and gastric
abdominal pain and signs and symptoms of bowel motility, listening for bowel sounds is not a reliable
obstruction indicator that ileus has fully resolved. Functional
- CT scan: diagnostic of choice evidence of coordinated GI motility in the form of
Finding: "target sign" passing flatus or bowel movement is a more useful
- Treatment: surgical resection of the involved segment indicator.
and the lead point, which needs to undergo pathologic
evaluation to rule out an underlying malignancy.

L. SHORT BOWEL SYNDROME APPENDIX


- presence of less than 200 cm of residual small bowel in
adult patients A. Anatomy
- usually acquired (s/p intestinal resection) B. Acute appendicitis
- can result to malabsorptive symptoms: diarrhea, C. Appendiceal tumors - Carcinoid
dehydration, and malnutrition,
- most common etiologies:
adults: acute mesenteric ischemia, malignancy, and
A. ANATOMY
Crohn's disease
- Function: immunologic organ; a GALT tossie that
pedia: intestinal atresias, volvulus, and necrotizing
secrete immunoglobulins
enterocolitis
- The base of the appendix can always be found at the
- Pathophysiology
confluence of the taenia
Normal: Resection of less than 50% of the small
- Tip of the appendix varies: retroceccal (most common),
intestine is generally well tolerated.
pelvic, subcecal, preileal, or right pericolic position
Symptomatic when greater than 50 to 80% of the
***The location of the tip of the appendix
small intestine has been resected.
determine the location of physical findings
Malabsorption in patients who have undergone
produced by irritation of parietal peritoneum
massive small bowel resection is exacerbated by a
- the luminal capacity if the normal appendix is 0.1 cc.
characteristic hypergastrinemia-associated
secretion of as little as 0.5 cc of fluid distal to the
gastric acid hypersecretion that persists for 1 to
obstruction raises intraluminal pressure to 60 cm H20.
2 years postoperatively
- Treatment:
B. ACUTE APPENDICITIS
TPN & enteral nutrition
- Etiology
Pharmacotherapy
Fecalith: most common cause of appendiceal
obstruction; usually in adults
REVIEW QUESTIONS
Lymphoid hyperplasia: most common cause of
appendiceal obstruction in patients of pediatric age
1. Vitamin B12 deficiency can occur after

TOPNOTCH MEDICAL BOARD PREP SURGERY SUPPLEMENT HANDOUT Page 68 of 85


For inquiries visit www.topnotchboardprep.com.ph or email us at topnotchmedicalboardprep@gmail.com
TOPNOTCH MEDICAL BOARD PREP SURGERY SUPPLEMENT HANDOUT - Jules Lopez,MD-MBA,Teddy
Carpio,MD-MBA
For inquiries visit www.topnotchboardprep.com.ph or email us at topnotchmedicalboardprep@gmail.com

- Pathogenesis: Luminal obstruction bacterial Gynecologic conditions: ruptured ectopic


overgrowth, active mucosal secretion & increased pregnancy, PID, ruptured grafian follicle, twisted
luminal pressure ovarian cyst
- Natural history: rarely resolves; ultimately lead to Intussusception: patients age, type of pain,
gangrene and perforation palpable mass in the lower quadrant and passage
- Clinical presentation of currant jelly stool may help with diagnosis;
Hallmark of appendicitis: poorly localized pain (due barium enema offers both diagnostic and
to distension stimulates visceral afferent pain therapeutic option for intussusception.
fibers) that is referred to the periumbilical region
- Special conditions:
(via autonomic innervations) Right lower
quadrant pain and tenderness (via somatic
AP IN THE YOUNG
innervations due to involvement of the parietal
- Diagnostic accuracy in these age group is lower
peritoneum)
compared to adults due to imprecise history and
Vomiting usually follows abdominal pain
nonspecific abdominal complaints
Anorexia is a constant symptom; if not present,
- Hx: vomiting, fever and diarrhea are common
question diagnosis; usually precedes abdominal
complaints
pain
- PE: abdominal distention, maximal tenderness in the
Variations in the anatomic location of the
right lower quadrant, the inability to walk or walking
appendiceal tip account for the different
with a limp, and pain with percussion, coughing, and
manifestation of the abdominal pain
hopping
Retrocecal: flank or back pain
- Gangrene and rupture are more common in these age
Pelvis: findings maybe absent; painful DRE
group because of delays in diagnosis
exam
- PE maneuvers:
Dumphys sign: increased pain during coughing or AP IN THE ELDERLY
jumping - Usually with atypical presentation: fever, leukocytosis
Rovsings sign: pain in the RLQ when pressure is and RLQ pain maybe minimal or absent
applied on the LLQ; this suggests peritoneal - Have 60-90% rupture rate
irritation - The atrophic omentum is less capable of walling off a
Psoas sign: pain on extension of the right thigh
with the patient lying on the left side; this is due to perforated appendix diffuse peritonitis or distant
the pain elicited by the stretched psoas muscle intra-abdominal abscess are expected
irritating the inflamed appendix - If patient is older than 60yo, always rule out cancer
Obturators sign: pain with passive rotation of the because the definitive treatment for that is right
flexed right hip; suggests that the inflamed tip lies hemicolectomy (if affecting the cecum)
in the appendix
- Diagnosis: usually based on history and physical AP IN PREGNANCY
examination even in the absence of laboratories and - Most common surgical emergency in pregnancy
imaging - In pregnancy, the gravid uterus pushes the appendix
Laboratory findings superiorly and the tip medially
1. Moderate leukocytosis with - Most consistent sign of AP in pregnant women: pain in
polymorphonuclear predominance (if above the right side of the abdomen
18,000 suspect abscess or perforation) - Common occurrence of abdominal pain, nausea and
2. Can also have normal WBC count (1/3 of leukocytosis in the normal course of pregnancy makes
patients) diagnosis difficult
3. Minimal albuminuria, (+) WBC and RBC in - Most cases occur during 2nd trimester
urine if appendix is retrocecal - Fetal mortality is 2-8.5%; increases to 35% with
4. Anemia in elderly should raise suspicion of rupture
carcinoma of the cecum
Imaging AP IN HIV OR AIDS PATIENTS
1. Plain abdominal films: fecalith, localized - Similar presentation to non-infected patients
ileus on the RLQ & loss of peritoneal fat - Risk of appendiceal rupture is higher for these patients
strip - DDx: CMV enteritis, typhilitis, fungal, protozoal and
2. UTZ: tubular, immobile and mycobacterial infections
noncompressible appendix,wall thickness of
Treatment: appendectomy
>2mm and outer diameter of at least 6 mm
are indicative of appendicitis
C. APPENDICEAL TUMORS - CARCINOID
3. CT scan: thickened by more than 5-
- most common location is appendix (50%), ileum (25%)
7mm&fluid filled, periappendiceal
then rectum (20%)
inflammation along with fat stranding, fluid
- ileal carcinoid has the highest potential for metastasis
collections & phlegmons
(arounf 35%) vs appendiceal carcinoid which has
- Differential diagnosis
lowest potential for metastasis (3%)
Acute mesenteric adenitis: associated with URTI
- Gross appearance: small, firm, circumscribed, yellowish
and presents with a more diffused abdominal painl
tumor
also with generalized lymphadenopathy with
- Treatment:
lymphocytosis
<2cm at distal appendix: appendectomy
Acute gastroenteritis: crampy abdominal pain
>2cm or at base: right hemicolectomy
with watery stools, nausea and vomiting
Diverticulitis: of cecum or perforated carcinoma
REVIEW QUESTIONS
of cecum is difficult to distinguish clinically from
appendicitis; diagnosis is usually done
1.
A patient suspected of having appendicitis
intraoperatively
underwent exploration, Crohns disease was found.
Epiploic apendagitis: infarction of the appendage
Which of the following are true?
due to torsion; pain shift is unusal and patient is
a. The normal appendix should always be
usually not ill
removed
b. All grossly involved bowel, including the
appendix, should be resected.
TOPNOTCH MEDICAL BOARD PREP SURGERY SUPPLEMENT HANDOUT Page 69 of 85
For inquiries visit www.topnotchboardprep.com.ph or email us at topnotchmedicalboardprep@gmail.com
TOPNOTCH MEDICAL BOARD PREP SURGERY SUPPLEMENT HANDOUT - Jules Lopez,MD-MBA,Teddy
Carpio,MD-MBA
For inquiries visit www.topnotchboardprep.com.ph or email us at topnotchmedicalboardprep@gmail.com
c. An inflamed appendix, cecum and terminal - can be both therapeutic and diagnostic
ileum, should be resected - length: 25 cm
d. Perforated bowel and advanced Crohns - 15-19 mm diameter proctoscope is useful for diagnostic
disease with obstruction should be resected. examination
- useful for polypectomy, electrocoagulation, detorsion of
Answer: D sigmoid volvulus
If a normal appendix is found at the time of
laparotomy, other causes should be sought. If 3. Flexible sigmoidoscopy and colonoscopy
Crohns disease is encountered and the cecum - provides excellent visualization of colon and rectum
and base of the appendix are normal, an - can be both diagnostic and therapeutic
appendectomy should be performed. If the base - length:
is involved with Crohns disease and the 60 cm: sigmoidoscope
appendix is normal, appendectomy should not be 100-160 cm: colonoscope
performed. If the finding of Crohns disease is - full length insertion:
uncomplicated by perforation or obstruction, may allow visualization as far as splenic flexure:
ileal resection is not indicated. However, in the sigmoidoscope
case of perforation or Crohns disease with may allow visualization as far as terminal ileum:
obstruction, the involved bowel should be colonoscope
resected.
IMAGING:

1. Plain x-ray and contrast studies


- plain x-rays of abdomen (upright, supine and
COLON, RECTUM, ANUS diaphragmatic views) are useful for detecting free intra-
abdominal air, bowel gas patterns suggestive of small or
large bowel obstruction and volvulus
A. Embryology
- contrast studies are useful for evaluationg obstructive
B. Diagnostic evaluation of Colon, Rectum and Anus
symptoms, delineating fistulous tracts and diagnosing
C. Evaluation of Common symptoms
small perforations or anastomotic leaks.
D. Diverticular disease
- Gastrografin (water soluble contrast agent) is
E. Colorectal adenocarcinoma
F. Colorectal carcinoid tumors recommended if perforation or leak is suspected
G. Anal intraepithelial neoplasia (Bowens disease) - Double contrast barium enema is 70-90% sensitive
H. Volvulus for the detection of mass lesions greater than 1 cm in
I. Colonic pseudoobstruction (Ogilvies syndrome) diameter
J. Hemorrhoids If a small, non obstructing lesion is considered,
K. Anal fissure colonoscopy is the preferred imaging modality of
L. Anorectal abcess choice
M. Fistula in ano
2. CT
- the utility of CT is in the detection of extraluminal
disease, such as intra-abdominal abscesses and
pericoloic inflammation and in staging colorectal
A. EMBRYOLOGY
carcinoma (because of its sensitivity in detecting
- Embryonic GI tract begins developing during 4th week
hepatic metastasis)
of gestation
REMEMBER: a standard CT scan is INSENSITIVE
Table 57. Embryology of GI tract
for detection of intraluminal lesions
FOREGUT MIDGUT HINDGUT - If considering a perforation / anastomotic leak:
Esophagus, small intestine, distal transverse check for extravasation of oral or rectal contrast
stomach, pancreas, ascending colon, colon, descending - Bowel wall thickening / mesenteric stranding
liver, duodenum and proximal colon, rectum, and suggests inflammatory bowel disease, enteritis/colitis
transverse colon proximal anus or ischemia
Celiac artery SMA IMA
***distal anus is derived from the ectoderm; BS: internal pudendal 3. MRI
artey - the main use of MRI in colorectal DO is in the evaluation
of pelvic lesions
- The colon has 5 distinct layers: mucosa, submucosa,
- more sensitive than CT for detecting bony involvement
inner circular muscle, outer longitudinal muscle, and
or pelvic sidewall extension of rectal tumors.
serosa - Can be useful in the detection and delineation of
complex fistulas in ano.
MUST KNOW
4. Positron Emission Tomography
Most common bacterium within the colon is B. fragilis - useful for imaging tissues with high levels of anaerobic
followed by E. coli and Enterococcus sp. glycolysis, such as malignant tumors
- F-fluorodeoxyglucose is injected as a tracer its
metabolism results in positron emission
B. DIAGNOSTIC EVALUATION OF COLON, RECTUM AND - Used as an adjunct to CT in staging colorectal cancer
ANUS
5. Angiography
ENDOSCOPY: - used for the detection of bleeding within the colon or
small bowel
1.Anoscopy - to visualize hemorrhage angiographically, bleeding
- useful instrument for the examination of the anal canal must be relatively brisk (0.5 to 1 cc per minute)
- not attempted without anesthesia if patient complains - if extravasation of contrast is identified, infusion with
of severe perianal pain and does not tolerate digital vasopressin or angiographic embolization can be
rectal examination therapeutic.
2. Proctoscopy
- useful for the examination of the rectum and distal 6. Endorectal and Endoanal UTZ
sigmoid colon
TOPNOTCH MEDICAL BOARD PREP SURGERY SUPPLEMENT HANDOUT Page 70 of 85
For inquiries visit www.topnotchboardprep.com.ph or email us at topnotchmedicalboardprep@gmail.com
TOPNOTCH MEDICAL BOARD PREP SURGERY SUPPLEMENT HANDOUT - Jules Lopez,MD-MBA,Teddy
Carpio,MD-MBA
For inquiries visit www.topnotchboardprep.com.ph or email us at topnotchmedicalboardprep@gmail.com
- is used primarily to evaluate the depth of invasion of - insert NGT (1st test that should be performed) since the
neoplastic lesions in the rectum and detecting most common cause of bleeding can either be
sphincter defects & outlining complex anal fistulas esophageal, gastric or duodenal
- normal rectal wall can be seen as a 5 layer structure if (+) return of bile suggests that bleeding is
- UTZ can reliably differentiate benign polyps from distal to the ligament of Treitz
invasive tumors based upon the integrity of the if bloody/non-bile secretions suggests an upper
submucosal layer. intestinal source; do EGD right away
- Accuracy in detecting depth of mural invasion is 81- - technetium-99-tagged RBC scan: highly sensitive (as
94% little as 0.1 cc/hour of bleeding can be detected);
however location is imprecise perform angiography
PHYSIOLOGIC AND PELVIC FLOOR INVESTIGATIONS: to localize bleeding
useful in the evaluation of patients with incontinence, - if sharp, knife-like pain + bright red rectal bleeding with
constipation, rectal prolapse, obstructed defecation and bowel movements anal fissure
other pelvic floor disorders - if painless, bright red rectal bleeding secondary to
bowel movements internal hemorrhoids
1. Manometry
- procedure: pressure-sensitive catheter is placed in the 3. Constipation and obstructed defecation
lower rectum catheter is withdrawn through the anal - A very common problem
canal and pressures recorded - rule out an underlying metabolic, pharmacologic,
- values: endocrine, psychological and neurologic causes first
resting pressure (normal: 40-80 mmHg): reflects before work up
the function of the internal anal sphincter - a stricture or mass lesion should be excluded by
Squeeze pressure (normal: 40-80 mmHg above colonoscopy or barium enema
resting pressure): maximum voluntary contraction - once other causes have been ruled out, perform transit
pressure minus resting pressure, reflects the studies
function of the external anal sphincter - Medical management is the mainstay of treatment for
High pressure zone (normal: 2-4 cm): estimates constipation (High fiber, increase fluids & laxatives)
the kength of the anal canal
- Absence of rectoanal inhibitory reflex is 4. Diarrhea and irritable bowel syndrome
characteristic of Hirschsprungs disease - Acute bloody diarrhea and pain can be due to infection
or inflammation
2. Neurophysiology - chronic diarrhea has a more difficult diagnostic
- Neurophysiologic testing assesses function of the dilemma since causes are myriad (ulcerative colitis,
pudendal nerve and recruitment of puborectalis muscle crohns colitis, malabsorption, short-gut syndrome,
fibers carcinoid, islet cell tumors, etc)

LABORATORY STUDIES: 5. Incontinence


- ranges in severity from occasional leakage of gas and
1. Fecal Occult Blood testing (FOBT) liquid stool to daily loss of solid stool
- is a screening test for colonic neoplasms in - can be neurogenic or anatomic
asymptomatic, average-risk individuals Neurogenic: diseases of CNS, spinal cord, pudendal
- occult bleeding from any GI source will produce a nerve injury
positive result (since it is a non specific test for Anatomic: congenital abnormalities, procidentia,
peroxidase contained in hemoglobin) overflow incontinence secondary to impaction,
- any positive FOBT mandates further investigation, neoplasm or trauma
usually by colonoscopy - Most common traumatic cause of incontinence is injury
to the anal sphincter during vaginal delivery
2. Stool studies
- helpful in the evaluation of etiology of diarrhea D. DIVERTICULAR DISEASE
- wet mount examination: (+) fecal leukocytes indicate - Diverticulosis: presence of diverticula without
colonic inflammation or presence of invasive organisms inflammation.
(such as E. coli or Shigella) Common in patients with low fiber diet
- Sudan red stain to stool sample: to evaluate steatorrhea Majority tend to occur after the age of 85
Sigmoid colon: most common site of
3. CEA: tumor marker diverticulosis
- elevated in 60-90% of patients with colorectal cancer; Common symptom: massive LGIB
however, not an effective screening agent for colorectal - Diverticulitis: inflammation and infection associated
CA with diverticula.
- serial monitoring used after curative-intent surgery is 5% of complicated diverticulitis develop a fistula to
done an adjacent organ. Most common of which is a
C. EVALUATION OF COMMON SYMPTOMS colovesical fistula.
1. Pain - More common is false diverticula type
- abdominal pain related to colon and rectum can result Only mucosa and muscularis mucosa have
from obstruction (inflammatory or neoplastic), herniated (also called pulsion diverticula) through
inflammation, perforation or ischemia the colonic wall, in between taenia coli (area of
- pelvic pain can originate from distal colon and rectum weakness)
or adjacent urogenital structures - True diverticula, comprises all layers of the bowel, is
tenesmus: due to proctitis or from rectal or congenital and rare
rectrorectal mass - Clinical manifestations of diverticulitis:
cyclical pain + menses + rectal bleeding: Uncomplicated diverticulitis: left-sided abdominal
endometriosis pain, with or without fever, mass and leukocytosis
- anorectal pain is most often secondary to anal fissure, complicated diverticulitis: abscess, obstruction,
perirectal abscess and/or fistula, or a thrombosed diffuse peritonitis (free perforation), or fistulas
hemorrhoids (most common is colovesical fistula)
- Hinchey staging for complicated diverticulitis
2. Lower GI bleeding Stage I: colonic inflammation with an associated
- first goal in managing a patient with GI hemorrhage: pericolic abscess
ADEQUATE RESUSCITATION
TOPNOTCH MEDICAL BOARD PREP SURGERY SUPPLEMENT HANDOUT Page 71 of 85
For inquiries visit www.topnotchboardprep.com.ph or email us at topnotchmedicalboardprep@gmail.com
TOPNOTCH MEDICAL BOARD PREP SURGERY SUPPLEMENT HANDOUT - Jules Lopez,MD-MBA,Teddy
Carpio,MD-MBA
For inquiries visit www.topnotchboardprep.com.ph or email us at topnotchmedicalboardprep@gmail.com
Stage II: colonic inflammation with a retroperitoneal or Hyperplastic polyp: most common type of
pelvic abscess all polyps; usually small, multiple and sessile;
Stage III: purulent peritonitis occur frequently in the rectosigmoid region
Stage IV: fecal peritonitis. Pseudopolyps (or inflammatory polyps):
- Diagnosis: CT scan occur most commonly in the context of
Appears as pericolic soft tissue stranding, inflammatory bowel disease, amebic colitis,
colonic wall thickening, and/or phlegmon ischemic colitis, and schistosomal colitis; not
- Treatment: premalignant, but they cannot be
Uncomplicated diverticulitis: outpatient therapy distinguished from adenomatous polyps based
with broad-spectrum oral antibiotics for 7-10 days upon gross appearance & therefore should be
& low-residue diet; failure to improve within 48-72 removed.
hours indicates abscess formation Hamartomas: similar appearance to
If 2nd episode of uncomplicated diverticulitis or 1st adenomatous polyps but is not considered to
episode of complicated diverticulitis: elective be premalignant
sigmoid colectomy is recommended Familial juvenile polyposis: autosomal
Small abscesses (<2 cm diameter) may be treated dominant DO in which patients develop
with parenteral antibiotics. hundreds of polyps in the colon and
Larger abscesses are best treated with CT-guided rectum; degenerate into adenomas
percutaneous drainage carcinoma
Peutz Jeghers syndrome: characterized
E. COLORECTAL ADENOCARCINOMA by polyposis of the small intestine and, to
- Most common malignancy of the GIT a lesser extent, of the colon and rectum. ;
- Risk factors: Characteristic melanin spots are noted on
1. Aging: dominant risk factor for colorectal cancer; the buccal mucosa and lips of these
incidence increases after 50 yo patients.
2. Known FH of cancer: accounts for 20% of cases Cronkite-Canada syndrome: GI
3. Diet: high in animal fat and low in fiber polyposis + alopecia + cutaneous
4. Inflammatory bowel syndrome pigmentation + atrophy of the fingernails
5. Cigarette smoking and toenails; SSx: Diarrhea, vomiting,
malabsorption, and protein-losing
- Pathogenesis: genetic defects enteropathy
Figure 34. Schematic diagram showing progression from normal
Cowden syndrome: autosomal dominant
colonic mucosa to carcinoma of colon disorder with hamartomas of all three
embryonal cell layers; Facial
trichilemmomas, breast cancer, thyroid
disease, and GI polyps are typical of the
syndrome.
Neoplastic polyps
Tubular adenomas: most common type of
APC neoplastic polyps; asymptomatic,
- Tumor suppressor gene located at chromosome 5 pedunculated, less than 1 cm in size and occur
- Function: the protein product of APC is for maintain commonly in the rectosigmoid region
cellular adhesions and suppressing neoplastic growth <1cm: rare chance for malignany
- APC inactivation leads to sporadic colorectal cancer 1-2cm: 10% chance for malignancy
- Mutated in individuals with familial adenomatous >2 cm: 30% chance for malignancy
polyposis (FAP) Tubulovillous adenoma: mixed; 22% chance
for malignancy
K-RAS Villous adenoma: sessile, larger and
- Proto-oncogene located in chromosome 12 symptomatic, can cause malignancy by 40-
- Function: encodes for plasma membrane based protein 50%; highest risk of cancer
involved in transduction of growth and differential Sessile adenomas are more likely to harbor
signals malignancy compared to pedunculated ones
- Mutation leads to uncontrolled cell division
- K-RAS activation leads to colorectal cancer - Inherited colorectal carcinoma
- Familial adenomatosis polyposis
rare autosomal dominant condition accounts
DCC for only about 1% of all colorectal
- Tumor suppressor gene located at chromosome 18 adenocarcinomas.
- Function: encodes for a protein responsible for cell to Due to mutation in the APC gene, located on
cell contact chromosome 5q
- Loss of DCC gene (or inactivation) tend to present in Clinically, patients develop hundreds to
more advanced carcinomas thousands of adenomatous polyps shortly
- Present in 70% of colorectal carcinomas after puberty.
The lifetime risk of colorectal cancer in FAP
P53 patients approaches 100% by age 50 years.
- Tumor suppressor gene located at chromosome 17 Flexible sigmoidoscopy of first-degree
- Mutations of this gene are the most common genetic relatives of FAP patients beginning at age 10
abnormality found in human cancer genes to 15 years
- Function: crucial for initiating apoptosis in cells with FAP may be associated with extraintestinal
irreparable genetic damage. manifestations such as congenital hypertrophy
- Mutations in p53 are present in 75% of colorectal of the retinal pigmented epithelium, desmoid
cancers tumors, epidermoid cysts, mandibular
- p53 inactivation leads to colorectal cancer osteomas (Gardner's syndrome), and central
nervous system tumors (Turcot's syndrome).
- Polyps: HNPCC or Lynch syndrome
Non-neoplastic polyps (no malignant potential) Rare autosomal dominant disorder arising
from errors in mismatch repair
TOPNOTCH MEDICAL BOARD PREP SURGERY SUPPLEMENT HANDOUT Page 72 of 85
For inquiries visit www.topnotchboardprep.com.ph or email us at topnotchmedicalboardprep@gmail.com
TOPNOTCH MEDICAL BOARD PREP SURGERY SUPPLEMENT HANDOUT - Jules Lopez,MD-MBA,Teddy
Carpio,MD-MBA
For inquiries visit www.topnotchboardprep.com.ph or email us at topnotchmedicalboardprep@gmail.com
is characterized by the development of APR: tumors less than 5 cm from ananl verge and if
colorectal carcinoma at an early age (average recurrent cancer at LAR site
age: 40 to 45 years).
The risk of synchronous or metachronous F. COLORECTAL CARCINOID TUMORS
colorectal carcinoma is 40%. - Result of a neuroendocrine tumor that secretes
HNPCC also may be associated with neurotransmitters (serotonin, ACTH, histamine,
extracolonic malignancies, including dopamine, tryptophan, substance P, bradykinin)
endometrial (most common), ovarian, - occur most commonly in the GI tract
pancreas, stomach, small bowel, biliary, and - most common location is appendix (50%), ileum (25%)
urinary tract carcinomas. then rectum (20%)
Diagnosis: Amsterdam criteria for clinical - ileal carcinoid has the highest potential for metastasis
diagnosis of HNPCC are three affected - appendiceal has lowest potential for metastasis
relatives with histologically verified - Carcinoid tumors in the proximal colon are less
adenocarcinoma of the large bowel (one must common and are more likely to be malignant.
be a 1st degree relative of one of the others) in - Can be part of MEN type I
2 successive generations of a family with 1 - risk of malignancy increases with size (more than 60%
patient diagnosed before age 50 years. of tumors greater than 2 cm in diameter are associated
Screening colonoscopy is recommended with distant metastases)
annually for at-risk patients beginning at - clinical manifestation:
either age 20 to 25 years or 10 years younger triad of flushing (due to excess bradykinin),
than the youngest age at diagnosis in the diarrhea (due to excess serotonin) & valculhar
family, whichever comes first. heart disease (primarily affects the mitral valve
from excess serotonin)
Familial colorectal cancer others: hypotension, tachycardia, alcohol
Nonsyndromic familial colorectal cancer intolerance
accounts for 10 to 15% of patients with - diagnosis:
colorectal cancer 24 hour 5-HIAA collection (5-HIAA is a metabolite
Screening colonoscopy is recommended every of serotonin)
5 years beginning at age 40 years or beginning - treatment:
10 years before the age of the earliest Small carcinoids can be locally resected, either
diagnosed patient in the pedigree. transanally or using transanal endoscopic
microsurgery.
- Routes of spread & natural history: Larger tumors or tumors with obvious invasion
Regional lymph node involvement is the most into the muscularis require more radical resection
common form of spread of colorectal carcinoma Medical: somatostatin analogues (octreotide)
and usually precedes distant metastasis RT
T stage (depth of invasion) is the single most
significant predictor of lymph node spread G. ANAL INTRAEPITHELIAL NEOPLASIA (BOWENS
The number of lymph nodes with metastases DISEASE)
correlates with the presence of distant disease and - refers to squamous cell carcinoma in situ of the
inversely with survival: 4 or more involved lymph anus.
nodes predict a poor prognosis - precursor to an invasive squamous cell carcinoma
most common site of distant metastasis from (epidermoid carcinoma)
colorectal cancer is the liver (via hematogenous - may appear as a plaque-like lesion, or may only be
spread to the portal venous system) apparent with high-resolution anoscopy and
application of acetic acid or Lugol's iodine solution.
- Screening: - associated with HPV infection types 16 and 18.
annual DRE at age 40 - Incidence has increased dramatically among HIV-
FOB at age 50 positive, homosexual men.
Flexible signoidoscopy every 5 years at age 50 - Treatment:
Colonoscopy if with risk factors Ablation
- Clinical presentation: change in bowel habits, rectal Topical immunomodulators such as imiquimod,
bleeding, melena, unexplained anemia, or weight Topical 5-FU
loss
- Staging and treatment H. VOLVULUS
- occurs when an air-filled segment of the colon twists
Table 58. Duke staging of colorectal cancer & treatment about its mesentery
Stage description 5 year Treatment - clinical manifestations: similar to SBO; abdominal
survival distention, nausea, and vomiting; can rapidly progress
Stage A Cancer limited 90% Wide to generalized abdominal pain and tenderness; Fever
to mucosa & resection of
submucosa colon with
and leukocytosis are heralds of gangrene and/or
Stage B Cancer 70% sampling of perforation
invades the LN (to rule
muscularis out er Table 59. Comparison of sigmoid vs cecal volvulus
propria disease Sigmoid volvulus Cecal volvulus
stage); stage B -90% of cases -<20% of cases
can also -plain abdominal xray: bent -plain abdominal xray: kidney-
employ inner tube or coffee bean shaped, air-filled structure in
RT+CT appearance with the convexity the LUQ
Stage C Invasion of 30% Surgery + of the loop lying in the RUQ
local LN chemotx (5- (opposite site of obstruction)
fluorouracil + -gastrografin enema: birds beak
leucovorin) + (pathognomonic)
RT Initial management: fluid -surgical exploration once
Stage D Distant Limited palliative resuscitation followed by diagnosis is made; no room for
metastasis survival endoscopic detorsion (rigid endoscopic detorsion
proctoscope); if suspecting
gangrene or perforation, perform
LAR: for tumors located 5-10 cm from anal verge immediate surgical exploration

TOPNOTCH MEDICAL BOARD PREP SURGERY SUPPLEMENT HANDOUT Page 73 of 85


For inquiries visit www.topnotchboardprep.com.ph or email us at topnotchmedicalboardprep@gmail.com
TOPNOTCH MEDICAL BOARD PREP SURGERY SUPPLEMENT HANDOUT - Jules Lopez,MD-MBA,Teddy
Carpio,MD-MBA
For inquiries visit www.topnotchboardprep.com.ph or email us at topnotchmedicalboardprep@gmail.com
- Fissures located laterally should arouse suspicion of
Crohns, UC, syphilis, TB,leukemia
- Clinical manifestation: tearing pain with defecation
MUST KNOW a and hematochezia; often too tender to tolerate DRE
- Treatment
Birds beak is also seen in barium esophagogram of Initially, can be managed conservatively with
achalasia! lubricants, warm sitz bath and bulk laxatives
(treatment of choice)
Surgery: lateral subQ partial internal
sphincterectomy
I. COLONIC PSEUDOOBSTRUCTION (OGILVIES Posterior fissurectomy & sphincterectomy
SYNDROME) can lead to keyhole defect & constant
- Distention of the abdomen leading to colonic soiling
obstruction (even if there is no obvious signs of
obstruction) L. ANORECTAL ABCESS
- most commonly occurs in hospitalized patients and is - Perianal abscess: most common manifestation and
associated with the use of narcotics, bedrest, and appears as a painful swelling at the anal verge
comorbid disease. - Ischiorectal abscess: happens when there is Spread
- Due to a neurologic dysfunction, electrolyte through the external sphincter below the level of the
abnormality and age puborectalis; may become extremely large and may not
- Treatment: NGT, IV neostigmine, IV atropine (to be visible externally; DRE will reveal a painful swelling
counter bradycardia as SE of neostigmine), exploratory laterally in the ischiorectal fossa
laparotomy during worst case scenario) - Intersphincteric abscess: occur in the intersphincteric
space and are notoriously difficult to diagnose; causes
J. HEMORRHOIDS deep pain in the rectum without external manifestation
- are cushions of submucosal tissue containing venules, - Pelvic and superior levator abscess: rare; may result
arterioles, and smooth-muscle fibers that are located in from extension of an intersphincteric or ischiorectal
the anal canal abscess upward, or extension of an intraperitoneal
- Excessive straining, increased abdominal pressure, and abscess downward
hard stools lead to further prolapsed of hemorrhoids - Horseshoe abscess: bilateral ischiorectal, supralevator
- Difference between internal and external hemorrhoids or perianal abscesses that communicate; begins as a
posterior midline infection
- Treatment: drainage with local anesthesia
Table 60: comparison of internal and external hemorrhoids - Signs of (+) fistula: nonhealing of an abscess wounds or
Internal hemorrhoids External hemorrhoids recurrence of an abscess at the same location
-Exaggerated submucosal - are dilated veins of the inferior
vascular cushions normally hemorrhoidal plexus located
M. FISTULA IN ANO
located above dentate line; below the dentate line and
covered by insensate covered by anoderm; can cause
- Classification (based on relationship to the anal
transitional mucosa of the anal significant pain sphincter complex)
canal and not by anoderm; only 1. Intersphincteric (most common): tracks
become painful when already through the distal internal sphincter and
thrombosed / necrosis intersphincteric space to an external opening near
the anal verge
-prolapsing hemorrhoids: are
internal hemorrhoids beyond the 2. Transsphincteric: often results from an ischiorectal
dentate line abscess and extends through both the internal and
external sphincters
3. Suprasphincteric: originates in the intersphincteric
- Types of internal hemorrhoids: graded according to
plane and tracks up and around the entire external
extent of prolapsed sphincter
4. Extrasphincteric: originates in the rectal wall and
1st degree: bulge into the anal canal and may prolapse tracks around both sphincters to exit laterally,
beyond the dentate line on straining usually in the ischiorectal fossa
2nd degree: prolapse through the anus but reduce
- Goodsalls rule : states that, if the external opening
spontaneously
is anterior to the imaginary line drawn between the
3rd degree: prolapse through the anal canal and require
ischial tuberosities, the fistula runs directly into the
manual reduction
anal canal. If the external opening is posterior, the
4th degree: prolapse but cannot be reduced and are at
tract curves to the posterior midline.
risk for strangulation
- Combined internal and external hemorrhoids:
***EXCEPTION: if an anterior external opening is
straddle the dentate line; mixed characteristics
greater than 3 cm from the anal margin, these
- Post partum hemorrhoids: result from straining
fistulas usually track to the posterior midline!!!
during labor, which results in edema, thrombosis,
and/or strangulation.
Figure 35. Goodsalls rule
- Treatment:
Dietary fiber, stool softeners, OFI, avoid straining:
for 1st & 2nd degree hemorrhoids
Rubber band ligation: for persistent 1st & 2nd
degree hemorrhoids & selected 3rd degree
hemorrhoids
If thrombosed hemorrhoids, perform excision
Most common complication of
hemorrhoidectomy: urinary retention

K. ANAL FISSURE
- is a tear in the anoderm distal to the dentate line
- 90% of fissures are located at the posterior midline, - Treatment: fistulotomy with adequate drainage or seton
an area where the anoderm is least supported by the placement
sphincter
TOPNOTCH MEDICAL BOARD PREP SURGERY SUPPLEMENT HANDOUT Page 74 of 85
For inquiries visit www.topnotchboardprep.com.ph or email us at topnotchmedicalboardprep@gmail.com
TOPNOTCH MEDICAL BOARD PREP SURGERY SUPPLEMENT HANDOUT - Jules Lopez,MD-MBA,Teddy
Carpio,MD-MBA
For inquiries visit www.topnotchboardprep.com.ph or email us at topnotchmedicalboardprep@gmail.com

REVIEW QUESTIONS

1. Which of the following is important in


maintaining the integrity of the colonic mucosa?

a. short-chain fatty acids


b. alanine
c. medium-chain fatty acids
d. glutamine

Answer: A
Short chain fatty acids are produced by bacterial
fermentation of dietary carbohydrates. Short
chain fatty acids are an important source of
energy for the colonic mucosa, and metabolism
by colonocytes provides energy for processes
such as active transport of sodium. Lack of a
dietary source for production of short chain fatty
acids, or diversion of the fecal stream by an A.Midline -are used because of the flexibility offered by
incision this approach in establishing adequate
ileostomy or colostomy, may result in mucosal exposure.
atrophy and diversion colitis. -the incision in the fused midline aponeurotic
tissue (linea alba) is simple and requires no
2. Match the organs in the left hand column with division of skeletal muscle.
the location of their referred pain in the right B.paramedian -made longitudinally 3 cm off the midline,
hand column. (items in the right may be used incision through the rectus abdominis sheath
more than once) structures, and have largely been abandoned
in favor of midline or nonlongitudinal access
methods
A. Gallbladder a. epigastrium
C.right -Subcostal incisions on the right (Kocher
B. Jejunum b. periumbilical subcostal incision for cholecystectomy) or left (for
C. Rectum c. hypogastrium incision splenectomy) are archetypal muscle-dividing
D. Pancreas d. shoulder D.bilateral incisions that generally result in the
E. Appendix subcostal transaction of some or all of the rectus
abdominis muscle fibers and investing
Answer: A a,d; B b; C c; D a; E b aponeuroses.
-These incisions generally are closed in two
The visceral peritoneum is innervated by C fibers layers (anterior aponeurotic sheath of the
rectus muscle medially, transitioning to
coursing with the autonomic ganglia. C fibers are
external oblique muscle and aponeurosis
unmyelinated, slow-conducting (0.5-5.0 m/s), more laterally & posterior, deeper layer
polymodal nociceptors that travel bilaterally with the consists of internal oblique and 75ransverses
sympathetic and parasympathetic fibers. Visceral pain abdominis muscle)
is a response to injury of the visceral peritoneum. E.Rocky davis Right lower quadrant incision or muscle
Distension, stretch, traction, compression, torsion, incision splitting incision for appendectomy
ischemia and inflammation trigger visceral pain F.McBurney it begins 2 to 5 centimeters above the anterior
fibers. Abdominal organs are insensate to heat, incision superior iliac spine and continues to a point
cutting and electrical stimulation. one-third of the way to the umbilicus
(McBurney's point). Thus, the incision is
parallel to the external oblique muscle
Visceral pain is typically vague and crampy and is G.Transverse Similar to kocher incision (subcostal incision).
perceived in the region of oprigin of the incision Preferred for newborns and infants because
embryologically derived autonomic ganglia. Foregut more abdominal exposure is gained per length
organs (proximal to the ligament of treitz) refer pain of the incision compared to vertical exposure
to the celiac chain, and the pain is felt in the H.Pfannenstiel Pfannenstiel incision, used commonly for
epigastrium. The organs of the midgut (small incision pelvic procedures, is distinguished by
intestine, ascending colon) refer pain to the superior transverse skin and anterior rectus sheath
mesenteric chain (periumbilical chain) and those of incisions, followed by rectus muscle retraction
and longitudinal incision of the peritoneum.
the hindgut (transverse and descending colon,
sigmoid colon and rectum) to the inferior mesenteric
B. RECTUS ABDOMINIS DIASTASIS
ganglia and hypogastrium.
- Other name: diastasis recti
- is a clinically evident separation of the rectus abdominis
muscle pillars resulting to a characteristic bulging of
ABDOMINAL WALL & HERNIAS the abdominal wall in the epigastrium (sometimes
mistaken for a ventral hernia)
A. Abdominal Incisions - may be congenital
B. Rectus abdominis diastasis - can be associated with advancing age, in obesity, or
C. Rectus sheath hematoma after pregnancy
D. Abdominal wall hernias - In the postpartum setting, rectus diastasis tends to
E. Incisional hernias occur in women who are of advanced maternal age, who
F. Retroperitoneal fibrosis have a multiple or twin pregnancy, or who deliver a
G. Inguinal hernias high-birth-weight infant.
H. Femoral hernia - Diagnosis:
CT scan: can differentiate rectus diastasis from a
true ventral hernia
- Treatment: surgery
A. ABDOMINAL INCISIONS
C. RECTUS SHEATH HEMATOMA
Figure 36. Abdominal incisions

TOPNOTCH MEDICAL BOARD PREP SURGERY SUPPLEMENT HANDOUT Page 75 of 85


For inquiries visit www.topnotchboardprep.com.ph or email us at topnotchmedicalboardprep@gmail.com
TOPNOTCH MEDICAL BOARD PREP SURGERY SUPPLEMENT HANDOUT - Jules Lopez,MD-MBA,Teddy
Carpio,MD-MBA
For inquiries visit www.topnotchboardprep.com.ph or email us at topnotchmedicalboardprep@gmail.com
- As a result of hemorrhage from any of the network of bilateral), or diminished lower extremity pulses
collateralizing vessels (superior and inferior epigastric (unilateral or bilateral).
arteries or veins) within the rectus sheath and muscles - Diagnosis:
- History: trauma, sudden contraction of the rectus ESR, BUN & creatinine
muscles with coughing, sneezing, or any vigorous Most definitive noninvasive diagnostic test:
physical activity. intravenous pyelography
- Clinical manifestations: sudden onset of unilateral - Treatment: corticosteroids with or without surgery
abdominal pain that increases with contraction of the (only indicated when renal function is compromised)
rectus muscles; palpable tender mass
(+)Fothergill's sign: palpable abdominal mass F. INGUINAL HERNIAS
that remains unchanged with contraction of the
rectus muscles Table 61. comparison of Inguinal hernias
- Diagnosis: Indirect inguinal Direct inguinal hernia
Abdominal UTZ may show a solid or cystic mass hernia
within the abdominal wall etiology - usually congenital; -Usually acquired;
CT scan: most definitive study to establish the due to patent processus weakness in the
vaginalis abdominal wall
correct diagnosis and to exclude other disorders
musculature
- Treatment: nonoperative; surgery is indicated in Risk Strenuous physical activity, obesity, ehlers danlos,
instances of expensing hematoma and hemodyanamic factors: smoking
instability anatomy protrude lateral to the protrusions medial to
inferior epigastric the inferior epigastric
D. ABDOMINAL WALL HERNIAS vessels, through the vessels, in Hesselbach's
- This is due to defects in the parietal abdominal wall deep inguinal ring triangle
fascia and muscle through which intra-abdominal or PE: cough impulse is Cough impulse is
preperitoneal contents can protrude inguinal controlled; felt on manifest; felt on the
occlusion the dorsum of fingertip
- ACQUIRED HERNIAS
test
may develop through slow architectural fingertip
deterioration of the muscular aponeuroses or they
may develop from failed healing of an anterior
abdominal wall incision (incisional hernia). MUST KNOW a
most common finding is a mass or bulge on the
anterior abdominal wall, which may increase in Hesselbachs triangle:
size with a Valsalva maneuver Inferior: inguinal ligament
PE reveals a bulge on the anterior abdominal wall Medial: rectus abdominis
that may reduce spontaneously, with recumbency, Superolateral border: inferior epigastric vessels
or with manual pressure
Treatment: if incarcerated (cannot be reduced) or
strangulated (BS is compromised) do surgical MUST KNOW a
correction
- PRIMARY VENTRAL HERNIAS Femoral hernias are more prevalent in females compared
Non incisional or true ventral hernias to males but the most common type of groin hernia in
Examples: females is still indirect inguinal hernia.

Epigastric hernias: congenital due to defective


midline fusion of lateral abdominal wall; occurs in
multiples and are small; located in the midline - Clinical manifestations: groin pain
between the xiphoid process and the umbilicus; - Diagnosis: usually employed for ambiguous diagnosis
found to contain omentum or a portion of the (i.e. obese patients)
falciform ligament. - Treatment: definitive treatment is surgical repair

Umbilical hernias: due to a patent umbilical ring; Bassini repair: anterior approach, nonprosthetic,
more common in premature infants;spontaneous hernia reduced and the defect oversewn, &
closure can occur at age of 5, no closure by that reconstruction the site of weakness; disadvantage: (+)
time, do elective surgical repair tension on the reconstructed tissue

Spigelian hernias: occur anywhere along the Shouldice repair: anterior approach, nonprosthetic,
length of the Spigelian line or zonean multilayer (4-layer suture repair) reconstruction
aponeurotic band of variable width at the lateral distributes the tension, effectively resulting in a
border of the rectus abdominis. tension-free repair; lowest recurrence rate

E. RETROPERITONEAL FIBROSIS Potts repair: high ligation of the sac only, with no
- class of disorders characterized by hyperproliferation of repair of the inguinal canal; used for indirect hernias
fibrous tissue in the retroperitoneum only
- if primary, it is known as Ormond disease
- may be secondary to inflammatory process, malignancy, McVay repair: anterior approach, nonprosthetic; the
or medication (methysergide, ergotamine, conjoined tendon is sutured to the coopers ligament
hydralazine, methyldopa and B blockers) laterally; can be used for indirect, direct & femoral
- Men are twice as likely to be affected as women hernias
- primarily affects individuals in the 4th-6th decades of ***problem with anterior non prosthetic
life. approaches: high recurrence rates
- Clinical manifestations:
Sx: insidious onset of dull, poorly localized Lichenstein tension free repair: addition of a mesh
abdominal pain, unilateral leg swelling, prosthesis effected a reconstruction of the posterior
intermittent claudication, oliguria, hematuria, & inguinal canal, without placing tension on the floor itself
dysuria.
PE: hypertension, the palpation of an abdominal or Read-rives repair: anterior preperitoneal approach
flank mass, lower extremity edema (unilateral or

TOPNOTCH MEDICAL BOARD PREP SURGERY SUPPLEMENT HANDOUT Page 76 of 85


For inquiries visit www.topnotchboardprep.com.ph or email us at topnotchmedicalboardprep@gmail.com
TOPNOTCH MEDICAL BOARD PREP SURGERY SUPPLEMENT HANDOUT - Jules Lopez,MD-MBA,Teddy
Carpio,MD-MBA
For inquiries visit www.topnotchboardprep.com.ph or email us at topnotchmedicalboardprep@gmail.com
Rives, stoppa, wanz repair: giant prosthetic c. Medical therapy to control ascites
reinforcement of the visceral sac; preperitoneal d. Transjugular intrahepatic portocaval shunt
approach followed by umbilical hernia repair

Kugel repair: maximize on the preperitoneal approach Answer: C


while minimizing on the length of the skin and fascia Treatment and control of the ascited with diuretic,
incision dietary management and paracentesis is the most
appropriate initial therapy. Patients with refractory
Laparoscopic repair: uses preperitoneal approach ascited may be candidates for transjugular
with small incisions; can asses and repair unilateral or intrahepatic portocaval shunting or eventual liver
bilateral inguinal hernias transplantation. Umbilical hernia repair should be
Emergent inguinal hernia repair: reserved for deferred until after the ascites is controlled.
strangulated, incarcerated and sliding hernias
2. In the setting of an equivocal examination,
which of the following has the greatest
Figure 37: treatment algorithm for hernia repair sensitivity in diagnosing an inguinal hernia?

a. Repeat examination by a second surgeon


b. Ultrasound
c. CT scan
d. MRI

Answer: D
Although Ct scan is useful in ambiguous clinical
presentations, little data exist to support its routine
use in diagnosis. The use of MRI in assessing groin
hernias was examined in a group of 41 patients
scheduled to undergo laparoscopic inguinal hernia
repair. Preoperatively, all patients underwent US and
MRI. Laparoscopic confirmation of the presence of
inguinal hernia was deemed as gold standard.
Physical examination was found to be the least
sensitive. False positives were low on physical
examination and MRI (one finding), but higher with
US (four findings). With further refinement of
technology, radiologic techniques qill continue to
improve sensitivity and specificity rates of diagnosis,
thereby serving a supplementary role in cases of
undertain diagnosis

DIAGNOSTIC PERITONEAL LAVAGE

A catheter is placed infraumbilically and a 10mL syringe is


connected. The aspirate is considered to show positive findings LIVER, PORTAL VENOUS SYSTEM & GALLBLADDER
if >10 mL of blood is aspirated. If <10 mL is withdrawn, a liter of
normal saline is instilled. The effluent is withdrawn via A. Anatomy
siphoning and sent to the laboratory for analysis B. Liver function tests
C. Radiographic evaluation
Anterior D. Liver cirrhosis
Thoracoabdominal E. Portal Hypertension
Abdominal Stab
Stab Wounds F. Budd-Chiari syndrome
Wounds
RBC Count >100,000/mL >10,000/mL G. Infections of the liver
WBC Count >500/mL >500/mL H. Benign neoplasms of the liver
I. Malignant tumors
Amylase level >19 IU/L >19 IU/L
J. Gallstone disease
Alkaline
K. Acute cholecystitis
Phosphatase >2 IU/L >2 IU/L
L. Choledocholithiases
level
M. Cholangitis
Bilirubin level >0.01mg/dL >0.01 mg/dL
N. Biliary pancreatitis
O. Acalculous cholecystitis
P. Biliary or choledochal cysts
Q. Sclerosing cholangitis
REVIEW QUESTIONS

1. Which of the following is the most important


initial therapy for a patient with portal A. ANATOMY
hypertension, ascites, and a tense umbilical
hernia? Liver
- Largest organ, weighing approximately 1500 g
a. Primary repair with concurrent placement of a
- Hepatoduodenal ligament contains the porta
peritoneal venous shunt
b. Emergency primary repair to avoid hernia hepatis (portal vein, hepatic artery and common bile
rupture duct)
TOPNOTCH MEDICAL BOARD PREP SURGERY SUPPLEMENT HANDOUT Page 77 of 85
For inquiries visit www.topnotchboardprep.com.ph or email us at topnotchmedicalboardprep@gmail.com
TOPNOTCH MEDICAL BOARD PREP SURGERY SUPPLEMENT HANDOUT - Jules Lopez,MD-MBA,Teddy
Carpio,MD-MBA
For inquiries visit www.topnotchboardprep.com.ph or email us at topnotchmedicalboardprep@gmail.com
Pringle maneuver: used to clamp this ligament in ***factor VIII: not synthesized exclusively in
the event of injury to the right hepatic artery the liver; has the shortest half life; useful for
during cholecystectomy determining liver failure
Relationship: CBD is located at the right of the AST (formarly -Indicators of integrity of hepatocellular
SGOT) & ALT membranes; increased levels reflect
hepatic artery and anterior to the portal vein
(formarly SGPT) hepatocellular injury with leakage
From the right and deep to the porta hepatis is the -AST: can also be found in liver, cardiac
foramen of winslow (or epiploic foramen) muscle, skeletal muscle, kidney, brain,
- Segmental anatomy pancreas, lungs, and red blood cells and
Cantlies line: plane from the gallbladder fossa to thus is less specific
the IVC that separates the livers right and left -ALT: more specific for liver disease
lobes grossly. -AST:ALT ratio of >2:1 alcoholic liver
Falciform ligament: separates the left lateral and disease
moderate increases: viral hepatitis
left medial segments along the umbilical fissure
-in the thousands ischemia, toxin
and anchors the liver to the anterior abdominal ingestion (acetaminophen), fulminant
wall; does not separate the liver to right and left hepatitis
lobes Indirect -elevations point to intrahepatic
Couinauds segments: divides liver into 8 (unconjugated) cholestasis, hemolytic disorders (hemolytic
segments, in clockwise direction with caudate lobe bilirubin anemia, resoprtion of hematomas),
as segment 1 bilirubin defects in hepatic uptake or
conjugation (acquired or inherited)
Direct -elevations point to extrahepatic or
Figure 38. Segmental anatomy of liver (conjugated) obstructive cholestasis, inherited or
bilirubin acquired disorders of intrahepatic
excretion or extrahepatic obstruction
Alkaline -found in liver and bones;
phosphatase -indicative of biliary obstruction
- since half life of AP is 7 days, it may take
several days for the levels to normalize
even after resolution of biliary obstruction
GGTP -early marker and sensitive test for
hepatobiliary disease
-nonspecific; can also be elevated in
overdose of certain medications, alcohol
Notes to figure
abuse, pancreatic disease, myocardial
Segments part Corresponding side Venous drainage infarction, renal failure, & obstructive
Segment I Caudate lobe IVC pulmonary disease
Segment II Left lateral superior segment Left lobe Left hepatic vein - interpret GGTP elevations with other
Segment III Left lateral inferior segment Left lobe Left hepatic vein
enzyme abnormalities
Segment IV Left medial segment (quadate Left lobe Middle hepatic
lobe outdated) vein
Segment V Right anterior inferior Right lobe Right & middle
segment hepatic vein BIOCHEMISTRY
Segment VI Right posterior inferior Right lobe Right hepatic vein
segment
Segment Right posterior superior Right lobe Right hepatic vein
AST (aspartate transaminase): an enzyme in gluconeogenesis
VII segment that transfers amino groups from aspartic acid to
Segment Right anterior superior Right lobe Right & middle
VIII segment hepatic vein
ketoglutaric acid to produce oxaloacetate.

- The hepatic veins divides the liver into 4 sectors ALT (alanine transaminase): an enzyme in gluconeogenesis
- The liver has dual blood supply: that transfers amino groups from alanine to ketoglutaric
hepatic actery: 25% acid to produce pyruvic acid
branch of celiac artery
most common variation: right hepatic artery
from SMA
portal vein: 75% (majority)
confluence of splenic vein and SMV
- normal pressure: 3-5 mmHg
- communication of portal vein and systemic circulation
(important for location of varices & bleeding in portal
hypertension): gastroesophageal junction, anal C. RADIOGRAPHIC EVALUATION
canal, falciform ligament, splenic venous bed and
left renal vein, and retroperitoneum ULTRASOUND
- Biliary tree
Hepatic ducts follow arterial branching of the liverLiver
Left hepatic duct has a longer extrahepatic course - Useful initial test imaging test of the liver because it
is inexpensive, involves no radiation exposure, and is
B. LIVER FUNCTION TESTS well tolerated by patients
- Term used to frequently measure the levels of group of - It is excellent for diagnosing biliary pathology and liver
serum markers for evaluation of liver dysfunction. lesions.
- A misnomer because the panel measures cell damage, - Limitations:
and not liver function Incomplete imaging: dome or beneath the ribs on
the surface, lesion boundaries are not as visualized
Table 62. Different components of liver function tests Obesity
Serum albumin, -Measures livers synthetic function Overlying gas bowels
prothrombin - prothrombin time and INR: best test - If a mass is detected, further evaluation by CT or MRI is
time & clotting among the 3 to measure the livers
required since UTZ has lower sensitivity and specificity
factors (except synthetic function
factor VIII) - PT is prolonged with conditions such as - Intraoperative ultrasound:
vitamin K deficiency or warfarin therapy Gold standard for diagnosing liver lesions
(because vitamin K is involved in the Y- Useful for tumor staging, visualization of
carboxylation of factors used to measure intrahepatic vascular structures, guidance of
prothrombin time) resection plane by assessment of relationship of
mass to vessels, for biopsy of tumors and tumor
ablation
TOPNOTCH MEDICAL BOARD PREP SURGERY SUPPLEMENT HANDOUT Page 78 of 85
For inquiries visit www.topnotchboardprep.com.ph or email us at topnotchmedicalboardprep@gmail.com
TOPNOTCH MEDICAL BOARD PREP SURGERY SUPPLEMENT HANDOUT - Jules Lopez,MD-MBA,Teddy
Carpio,MD-MBA
For inquiries visit www.topnotchboardprep.com.ph or email us at topnotchmedicalboardprep@gmail.com
Gallbladder 2. Venous or portal dominant phase (60 to
- UTZ is the initial investigation used for any patient 70 seconds after contrast injection) the
suspected of disease in the biliary tree. phase where there is optimal enhancement
- UTZ will show gallbladder stones with sensitivity and of normal liver parenchyma and
specificity of >90% hypovascular lesions (will appear
Appearance of GB Stones: (+) acoustic shadow, attenuated in contrast with brighter normal
move with changes in position (vs polyps: may liver parenchyma)
also have a shadow but does not move with
changes in position) gallbladder
If a stone obstructs the neck of the GB: large GB but - It is the test of choice in evaluating patients with
thin walled suspected malignancy of the GB, extrahepatic
- if acute cholecystitis: (+) edema within the wall of the biliary system or nearby organs, in particular, the
GB or between the GB and liver in association with head of pancreas
localized tenderness - Abdominal CT scan is inferior to UTZ in diagnosing
- if chronic cholecystitis: contracted thick-walled GB gallstones
- Extrahepatic ducts are well visualized using UTZ
(except for retroduodenal portion)
- Dilation of the ducts + stones in the GB + jaundiced PERCUTANEOUS TRANSHEPATIC CHOLANGIOGRAPHY
patient think extrahepatic obstruction - Useful in patients with bile duct strictures and
Periampullary tumors can be difficult to diagnose tumors, as it defines the anatomy of the biliary tree
on UTZ proximal to the affected segment
UTZ is useful for evaluating tumor invasion and - Mechanism: intrahepatic ducts are accessed
flow in the portal vein an important guideline in percutaneously with a small needle under fluoroscopic
the resectability of periampullary and pancreatic guidance catheter is placed cholangiogram
head tumors performed can do therapeutic interventions as
well (biliary drain insertion, stent placement)
ORAL CHOLECYSTOGRAPHY - Very little role in management of uncomplicated
- Considered as a diagnostic procedure of choice for gallstone disease
gallstones but it largely replaced now by UTZ.
- Mechanism: oral administration of radiopaque
compound that is absorbed and excreted by the liver, MAGNETIC RESONANCE IMAGING
passed into the GB stones are noted on a film as a
filling defect in a visualized, opacified GB Liver
- Also uses contrast agent, just like in CT scan, to
differentiate normal and pathologic lesion in the liver
BILIARY RADIONUCLIDE SCANNING (HIDA SCAN) - Types:
Gadopentate dimeglumine behaves in a manner
- Provides a noninvasive evaluation of the liver, GB, bile similar to iodine in CT
ducts and duodenum with both anatomic and functional Feruxomide excretion of kuppfer cells
information Iminoacetic acid-derivative radionuclide
- Mechanism: Technetium-labeled derivatives of secretion in bile by hepatocytes
dimethyl iminodiacetic acid (HIDA) are injected IV
cleared by Kuppfer cells in the liver excreted in Gallbladder
the bile - MRI with MRCP (magnetic resonance
10 minutes: time it takes for the liver to detect it cholangiopancreatography) offers a single
60 minutes: time it takes for the GB, bile ducts and noninvasive test for the diagnosis of bliary tract and
duodenum to detect it pancreatic disease
- the primary use of biliary scintigraphy is the
diagnosis of acute cholecystitis
appearance: nonvisualized GB, with prompt filling ENDOSCOPIC RETROGRADE CHOLANGIOGRAPHY (ERCP)
of the common bil duct and duodenum, biliary - It is the diagnostic and therapeutic procedure of
obstruction choice for stones in the CBD associated with
sensitivity & specificity: 95% obstructive jaundice, cholangitis and gallstone
- can also detect obstruction of the ampulla pancreatitis
appearance: filling of the GB and CBD with delayed - Provides direct visualization of the bilary and
and absent filling of the duodenum pancreatic ducts, particularly the ampullary region and
- can also be used for detection of biliary leaks as a distal common bile duct
complication of GB surgery - Therapeutic interventions include sphincterotomy,
stone extraction if indicated

COMPUTED TOMOGRAPHY
Liver POSITRON EMISSION TOMOGRAPHY
- Contrast medium is routinely used for liver evaluation liver
because of the similar densities of most pathologic - PET offers functional imaging of tissues with high
liver masses and normal hepatic parenchyma. metabolic activity, including most types of metastatic
Uses dual or triple phase bolus of IV contrast tumors
Exploits the dual blood supply of the liver: most - With high value for colorectal cancer with liver
liver tumors receive their blood supply from metastases
the hepatic artery and normal hepatic 20% of patients with colorectal cancer present
parenchyma from portal vein initially with liver metastasis
2 phases: presence of extrahepatic disease is a poor
1. Arterial dominant phase (20 to 30 prognosticator and precludes surgical intervention
seconds after beginning of contrast valuable tool for the diagnostic work up of
injection) the phase where hepatic tumors patient with potentially resectable hepatic
and other hypervascular lesions are well disease
delineated. must be combined with CT to improve
diagnostic accuracy
TOPNOTCH MEDICAL BOARD PREP SURGERY SUPPLEMENT HANDOUT Page 79 of 85
For inquiries visit www.topnotchboardprep.com.ph or email us at topnotchmedicalboardprep@gmail.com
TOPNOTCH MEDICAL BOARD PREP SURGERY SUPPLEMENT HANDOUT - Jules Lopez,MD-MBA,Teddy
Carpio,MD-MBA
For inquiries visit www.topnotchboardprep.com.ph or email us at topnotchmedicalboardprep@gmail.com
Most significant clinical finding:
D. LIVER CIRRHOSIS gastroesophageal varices
- final sequela of chronic hepatic insult, is characterized Major BS of GE varices: anterior branch of the
by the presence of fibrous septa (due to left gastric or coronary vein
accumulation to ECM matrix or scar tissue) May present with splenomegaly, hemorrhoids,
throughout the liver subdividing the parenchyma into ascites, caput medusa & upper GI bleeding due to
hepatocellular nodules variceal bleeding (leading cause of morbidity
- 2 consequences: hepatocellular failure and portal and mortality)
hypertension - Diagnosis: most accurate method of determining portal
- Classification hypertension is hepatic venography
Micronodular cirrhosis: characterized by thick - Management
regular septa, small uniform regenerative nodules, Prevention of variceal bleeding: improve liver
and involvement of virtually every hepatic lobule function (avoid alcohol), avoid aspirin & NSAID,
Macronodular cirrhosis: frequently has septa and beta blockers
regenerative nodules (irregularly sized Management of acute variceal bleeding
hepatocytes with large nuclei and cell plates of Specifics:
varying thickness) ICU admission: must!
Mixed cirrhosis: present when regeneration is Blood resuscitation: goal is Hgb of
occurring in a micronodular liver and over time 8g/dL and above
converts to a macronodular pattern FFP and platelets for patients with
- Etiology: viral, autoimmune, drug-induced, cholestatic, severe coagulopathy
and metabolic diseases Short term prophylactic antibiotics:
- Clinical manifestation ceftriaxone 1g/day (proven to decrease
Fat stores and muscle mass are reduced the rate of bacterial infections and
resting energy expenditure is increased increase survival)
(+) Muscle cramps: respond to administration of Vassopressin at 0.2 to 0.8 units/min IV
quinine sulfate and human albumin for vasoconstriction (most potent)
increased CO & HR Octreotide/somatostatin for splanchnic
Prone to infections due to impaired phagocytic vasoconstriction
activity of the RES Endoscopic variceal ligation (EVL)
Balloon tamponade using sengstaken-
- Diagnosis blakemore tube
mild normocytic normochromic anemia. Shunt therapy (surgical shunts or
Decreased WBC & PC TIPS)
bone marrow: macronormoblastic Even with aggressive pharmacologic and
prothrombin time is prolonged & does not respond endoscopic therapy, 10-20% of patients
to vitamin K tx with variceal bleeding will continue to
serum albumin level is decreased rebleed
serum levels of bilirubin, transaminases, and Shunt therapy (surgical shunt or TIPS),
alkaline phosphatase are all elevated on the other hand, has been shown to
control refractory variceal bleeding in
- CHILD-TURCOTTE-PUGH SCORE: evaluate the risk of >90% of treated individuals
portocaval shunt procedures secondary to portal Surgical shunt: CTP class A
hypertension and also useful in predicting surgical TIPS: CTP class B & C
risks of other intra-abdominal operations Balloon tamponade using sengstaken-
performed on cirrhotic patients blakemore tube can control refractory
bleeding in >80% of patients
Table 63. Child-Turcotte-Pugh Score
variable 1 point 2 points 3 points Complication: aspiration, esophageal
Bilirubin < 2 mg/dL 2-3 mg/dL >3 mg/dL perforation
Albumin >3.5 g/dL 2.8-3.5 g/dL <2.8 g/dL
INR <1.7 1.7-2.2 >2.2 Table.64 Comparison of Surgical shunts vs TIPS
Encephalopathy none controlled uncontrolled Surgical shunts (can be TIPS (Transjugular
Ascites none controlled uncontrolled selective or non selective Intrahepatic Portosystemic
shunts) Shunt)
Child-Turcotte-Pugh Class & overall surgical mortality rates
Class A = 5-6 points 10% -aim: reduce portal venous -considered as a nonselective
Class B= 7-9 points 30% pressure, maintain total hepatic shunt
Class C= 10-15 points 75-80% and portal blood flow and avoid -involves implantation of a
a high incidence of metallic stent between an
complicating hepatic intrahepatic branch of the portal
E. PORTAL HYPERTENSION encephalopathy vein and a hepatic vein radical
- definition: direct portal venous pressure that is >5 -TIPS can control variceal
mmHg greater than the IVC pressure, a splenic pressure -non-selective shunt (ex. bleeding in >90% of cases
of >15 mmHg, or a portal venous pressure measured at portacaval shunt or eck fistula: refractory to medical
surgery of >20 mmHg joins the portal vein to the IVC in treatment
- normal portal venous pressure: 5 to 10 mmHg an end-to-side fashion & disrupts -disadvantages: bleeding either
at this pressure, very little blood is shunted from portal vein flow to the liver, or intra-abdominally or via the
joins it in a side-to-side fashion biliary tree, infections, renal
the portal venous system into the systemic and maintains partial portal failure, decreased hepatic
circulation venous flow to the liver; non function, and er hepatic
as portal venous pressure increases, the selective; rarely performed now encephalopathy (because it is a
communication with the systemic circulation dilate because it has a higher non selective shunt)
Large amount of blood is shunted around the incidence of hepatic
encephalopathy and decreased
liver and into the systemic circulation liver function resulting from the
complications reduction of portal perfusion;
A portal pressure of >12 mmHg is necessary for controls bleeding effectively
varices to form and subsequently bleed
- Etiology: most common cause is cirrhosis -selective shunt (ex. Warren
(intrahepatic) shunt distal splenorenal & left
gastric caval shunt) have er
- Clinical manifestation
TOPNOTCH MEDICAL BOARD PREP SURGERY SUPPLEMENT HANDOUT Page 80 of 85
For inquiries visit www.topnotchboardprep.com.ph or email us at topnotchmedicalboardprep@gmail.com
TOPNOTCH MEDICAL BOARD PREP SURGERY SUPPLEMENT HANDOUT - Jules Lopez,MD-MBA,Teddy
Carpio,MD-MBA
For inquiries visit www.topnotchboardprep.com.ph or email us at topnotchmedicalboardprep@gmail.com
incidence of hepatic - Clinical manifestation: RUQ pain + fever + hepatomegaly
encephalopathy + travel to an endemic area
- Diagnosis:
F. BUDD-CHIARI SYNDROME most common biochemical abnormality: AP
- uncommon congestive hepatopathy characterized by level.
the obstruction of hepatic venous outflow due to (+) Leukocytosis
endoluminal venous thrombosis (primary) or transaminase levels and jaundice are unusual.
compressive lesion external to the veins (secondary) (+) fluorescent antibody test for E. histolytica
- risk factors: coagulopathies, thrombotic disease Ultrasound and CT scanning: very sensitive but
- most patients are women nonspecific for the detection of amebic abscesses
- mean age of diagnosis: 30 yo Appears to be as a well-defined low-density round
- clinical manifestations: abdominal pain (RUQ), ascites, lesions that have enhancement of the wall, ragged
and hepatomegaly or long standing portal hypertension in appearance with a peripheral zone of edema; has
- diagnosis a central cavity with septations & fluid levels
- abdominal UTZ: initial investigation of choice - Treatment
check for absence of hepatic vein flow, spider web Metronidazole 750 mg tid for 7 to 10 days is the
hepatic veins & collateral circulation treatment of choice and is successful in 95% of
- definitive imaging: hepatic venography cases.
- initial treatment: anticoagulation Defervescence usually occurs in 3 to 5 days.
Time of resolution of abscess: 30 to 300 days from
G. INFECTIONS OF THE LIVER presentation
Aspiration of the abscess is rarely needed and
PYOGENIC LIVER ABSCESS should be reserved for patients with large
- most common liver abscesses seen in the United abscesses, abscesses that do not respond to
States. medical therapy, abscesses that appear to be
- Risk factors: IV drug abuse, teeth cleaning, diverticulitis, superinfected, and abscesses of the left lobe of the
Crohn's disease, subacute bacterial endocarditis, (+) liver that may rupture into the pericardium
infected indwelling catheters & immunocompromised
states
- may be single or multiple HYDATID DISEASE
- more frequently found in the right lobe of the liver - due to the larval or cyst stage of infection by the
- causative organisms: tapeworm Echinococcus granulosus (causative agents)
monomicrobial: 40% ; polymicrobial: 40%; intermediate hosts: Humans, sheep, and cattle
culture negative: 20% definitive host: dogs
most common: gram-negative organisms - commonly involve the right lobe of the liver, usually
(Escherichia coli 2/3; Streptococcus faecalis, the anterior-inferior or posterior-inferior segments
Klebsiella, and Proteus vulgaris are also common) - clinical manifestations: dull RUQ or abdominal
Anaerobic organisms (ex. Bacteroides fragilis) are distention; can be clinically silent; if ruptured, may lead
also seen frequently to an allergic or anaphylactic reaction.
If (+) endocarditis / indwelling catheter: think - Diagnosis:
Staphylococcus and Streptococcus (+)ELISA for echinococcal antigens; maybe (-) if
- Clinical manifestations: RUQ pain, fever & jaundice (1/3 cyst has not leaked or does not contain scolices, or
of patients) if the parasite is no longer viable
- Diagnosis: Eosinophilia of >7% is found is approximately 30%
Leucocytosis, ESR & AP (most common of infected patients.
laboratory findings) UTZ & CT scan of the abdomen: sensitive for
Blood cultures reveal the causative organism in detecting hydatid cysts.
approximately 50% of cases. hydatid cysts: appear as well-defined
Liver UTZ: round or oval hypoechoic lesions hypodense lesions with a distinct wall; (+)
with well-defined borders and a variable Ring-like calcifications of the pericysts
number of internal echoes. (present in 20 to 30% of cases); healing occurs
CT scan: highly sensitive in the localization;
appear as hypodense mass with air-fluid levels
the entire cyst calcifies densely, and a
lesion with this appearance is usually dead or
(indicating a gas-producing organisms) &
inactive. Daughter cysts: occur in a peripheral
peripheral enhancement
location & are slightly hypodense compared
- Treatment: cornerstones of treatment include
with the mother cyst.
correction of the underlying cause, percutaneous needle
- Treatment:
aspiration, and IV antibiotic therapy
Unless the cysts are small or the patient is not a
Initial antibiotic therapy needs to cover gram-
suitable candidate for surgery, treatment of
negative as well as anaerobic organisms; must
hydatid disease is surgically based (laparoscopic
be continued for at least 8 weeks.
or open complete cyst removal + instillation of
If aspiration and IV antibiotics fail, undergo
scolicidal agent)
surgical therapy (either laparoscopic or open
caution must be exercised to avoid rupture of
drainage)
the cyst with release of protoscolices into the
Anatomic surgical resection is reserved for patients
peritoneal cavity.
with recalcitrant abscesses.
Peritoneal contamination can result in an acute
Always rule out necrotic hepatic malignancy
anaphylactic reaction or peritoneal
implantation of scolices with daughter cyst
formation and inevitable recurrence
AMEBIC ABSCESS
Medical treatment of choice: albendazole - initial
- most common type of liver abscesses worldwide.
treatment for small, asymptomatic cysts.
- Causative agent: Entamoeba histolytica
- can be single or multiple
H. BENIGN NEOPLASMS OF THE LIVER
- most commonly located in the superior-anterior
aspect of the right lobe of the liver near the diaphragm
HEPATIC CYST
- Gross: necrotic central portion that contains a thick,
- most common benign lesion found in the liver is the
reddish brown, pus-like material (anchovy paste or
congenital or simple cyst
chocolate sauce)
TOPNOTCH MEDICAL BOARD PREP SURGERY SUPPLEMENT HANDOUT Page 81 of 85
For inquiries visit www.topnotchboardprep.com.ph or email us at topnotchmedicalboardprep@gmail.com
TOPNOTCH MEDICAL BOARD PREP SURGERY SUPPLEMENT HANDOUT - Jules Lopez,MD-MBA,Teddy
Carpio,MD-MBA
For inquiries visit www.topnotchboardprep.com.ph or email us at topnotchmedicalboardprep@gmail.com
- female:male ratio is approximately 4:1 - diagnosis:
- Clinical manifestation: asymptomatic if small; Large biphasic CT scan: well circumscribed with a
simple cysts may cause abdominal pain, epigastric typical central scar
fullness, and early satiety. Occasionally the affected Arterial phase contrast: intense homogeneous
patient presents with an abdominal mass. enhancement
- Diagnosis: appear as thin-walled, homogeneous, Venous phase contrast: isodense or invisible
fluid-filled structures with few to no septations. MRI scans: hypointense on T1-weighted images &
- Treatment: isointense to hyperintense on T2-weighted images
Observation if asymptomatic After gadolinium administration, lesions are
If symptomatic, perform UTZ- or CT-guided hyperintense but become isointense on
percutaneous cyst aspiration followed by delayed images.
sclerotherapy nuclear imaging: (+) uptake by Kupffer cells.
excised cyst wall is sent for pathologic analysis to - Treatment: surgical resection only if symptomatic
rule out carcinoma, and the remaining cyst wall
must be carefully inspected for evidence of BILE DUCT HAMARTOMA
neoplastic change. - small liver lesions (2 - 4 mm)
- usually visualized on the surface of the liver at
HEMANGIOMA laparotomy.
- consist of large endothelial-lined vascular spaces and - Gross appearance: firm, smooth, and whitish yellow in
represent congenital vascular lesions that contain appearance.
fibrous tissue and small blood vessels which eventually - can be difficult to differentiate from small metastatic
grow lesions
- most common solid benign masses that occur in the - excisional biopsy often is required to establish the
liver diagnosis
- more common in women
- clinical manifestation: I. MALIGNANT TUMORS
most common symptom is abdominal pain
can be asymptomatic as well HEPATOCELLULAR CARCINOMA (HCC)
- diagnosis: - 5th most common malignancy worldwide
biphasic contrast CT scan: asymmetrical nodular - Risk factors: viral hepatitis (B or C), alcoholic cirrhosis,
peripheral enhancement that is isodense with hemochromatosis, and nonalcoholic steatohepatitis
large vessels and exhibit progressive centripetal - HCCs are typically hypervascular with blood supplied
enhancement fill-in over time predominantly from the hepatic artery
MRI: hypointense on T1-weighted images and - Most common site of metastasis is lungs
hyperintense on T2-weighted images - Clinical manifestations: jaundice, pruritus,
Caution should be exercised in ordering a liver hepatosplenomegaly, bleeding diathesis, cachexia,
biopsy if the suspected diagnosis is hemangioma encephalopathy, asterixis, ascites and varices
because of the risk of bleeding from the biopsy site - Diagnosis:
- treatment: Surgical resection (enucleation or CT scan: appears hypervascular during the arterial
formal hepatic resection) only if symptomatic; phase of CT studies & relatively hypodense during
observation if asymptomatic the delayed phases due to early washout of the
contrast medium by the arterial blood.
MRI: HCC is variable on T1-weighted images and
HEPATIC ADENOMA usually hyperintense on T2-weighted images; HCC
- benign solid neoplasms of the liver enhances in the arterial phase after gadolinium
- most commonly seen in young women (aged 20-40) injection because of its hypervascularity and
- typically solitary becomes hypointense in the delayed phases due to
- risk factors: Prior or current use of estrogens (oral contrast washout
contraceptives) (+) thrombus in portal vein is highly suggestive
- Gross appearance: soft and encapsulated and are tan to of HCC
light brown. AST,ALT,AFP
- Histology: does not contain Kuppfer cells - treatment options for liver cancer
- (+) risk of malignant transformation to a well- hepatic resection: reserved for patients without
differentiated HCC cirrhosis & Child's class A cirrhosis with preserved
- Clinical manifestation: liver function and no portal hypertension
carry a significant risk of spontaneous rupture liver transplantation: if with poor liver function
with intraperitoneal bleeding. and the HCC meets the Milan criteria (one nodule
The clinical presentation may be abdominal pain <5 cm, or two or three nodules all <3 cm, no gross
- Diagnosis: vascular invasion or extrahepatic spread)
CT scan: with sharply defined borders; can be Chemoembolization can also be of benefit
confused with metastatic tumors 5 year survival after complete resection: 30%
venous phase contrast: hypodense or isodense
(in comparison with background liver
arterial phase contrast: subtle hypervascular CHOLANGIOCARCINOMA
enhancement -
2nd most common primary malignancy within the liver
MRI: hyperintense on T1-weighted images and -
It is the adenocarcinoma of the bile ducts that forms in
enhance early after gadolinium injection. the biliary epithelial cells
nuclear imaging: "cold; no uptake of - Most commonly occurs at the bifurcation of the
radioisotope common hepatic duct
- Treatment: surgical resection - Subclassification:
peripheral (intrahepatic) bile duct cancer
tumor mass is within the lobe or peripheral of
FOCAL NODULAR HYPERPLASIA the liver
- A benign, solid neoplasm of the liver less common that extrahepatic bile duct
- more common in women of childbearing age Cancer
- FNH lesions usually do not rupture spontaneously central (extrahepatic) bile duct cancer
and have no significant risk of malignant if it is proximally located, referred to as a hilar
transformation. cholangiocarcinoma (Klatskin's tumor).
TOPNOTCH MEDICAL BOARD PREP SURGERY SUPPLEMENT HANDOUT Page 82 of 85
For inquiries visit www.topnotchboardprep.com.ph or email us at topnotchmedicalboardprep@gmail.com
TOPNOTCH MEDICAL BOARD PREP SURGERY SUPPLEMENT HANDOUT - Jules Lopez,MD-MBA,Teddy
Carpio,MD-MBA
For inquiries visit www.topnotchboardprep.com.ph or email us at topnotchmedicalboardprep@gmail.com
presents with obstructive and painless by the ingrowths of mucosal glands into the
jaundice rather than an actual liver mass muscle layer (epithelial sinus formation)
- treatment: 6. symptomatic granulomatous polyps
surgical resection is the treatment of choice - Gallstone formation
hilar cholangiocarcinoma + primary Cholesterol stones (80% of gallstones)
sclerosing cholangitis: surgical resection has multiple, variable size, may be hard and
no role & transplantation provided dismal faceted or irregular, mulberry-shaped, and
results soft; colors range from whitish yellow and
neoadjuvant chemoradiation has a role green to black
Most cholesterol stones are radiolucent
formation is due to supersaturation of bile
GALLBLADDER CANCER with cholesterol
- rare aggressive tumor with a very poor prognosis. Pigment stones (15-20% of gallstones)
- Cholithiasis is the most important risk factor for dark because of the presence of calcium
gallbladder carcinoma bilirubinate
- 80-90% of gallbladder tumors are adenocarcinomas Black pigment stones: small, brittle, black,
- signs and symptoms of GB carcinoma are and sometimes speculated; In Asian countries
indistinguishable from cholecystitis and cholelithiasis such as Japan, black stones account for a much
- sensitivity of UTZ in detecting GB carcinoma ranges higher percentage of gallstones than in the
from 70-100%. Western hemisphere; typically occur in
- Treatment: surgery is the only curative option for patients with cirrhosis and hemolysis
gallbladder cancer Brown pigment stones: <1 cm in diameter,
reoperation for an incidental finding of gallbladder brownish-yellow, soft, and often mushy; they
cancer after cholecystectomy (central liver are formed usually due to secondary to
resection, hilar lymphadenectomy, and evaluation bacterial infection (ex. E. coli)caused by
of cystic duct stump) bile stasis.; associated with stasis secondary
reoperation should be considered for all patients to parasite infection
who have T2 or T3 tumors or for whom the - Clinical presentation
accuracy of staging is in question Abdominal pain: epigastrium or RUQ, constant,
radical resection in patients with advanced disease increasing in severity, episodic, usually after a fatty
usually with dismal results if already with (+) meal, nausea, vomiting
hilar LN Hydrops of gallbladder: manifests as a palpable
nontender gallbladder
METASTATIC COLORECTAL CANCER Usually due to impacted stone without
- Over 50% of patients diagnosed with colorectal cancer cholecystis (pathophysio: bile gets absorbed,
will develop hepatic metastases during their lifetime. but the gallbladder epithelium continues to
- Resection is the preferred treatment for liver secrete mucus, and the gallbladder becomes
metastases from colorectal CA, provided that patient distended with mucinous material)
has adequate liver reserve, no extrahepatic metastases, Is usually an indication for cholecystectomy
total hepatic involvement and advanced crirhosis, vena - Diagnosis
cava or portal vein invasion Abdominal UTZ: standard diagnostic test for
- volume of future liver remnant and the health of the gallstones
background liver, and not actual tumor number, as the Presence of hyperechoic intraluminal focus
primary determinants in selection for an operative Shadowing posterior to the focus
approach. Movement of the focus with positional changes
of the patient
J. GALLSTONE DISEASE - Management: Patients with symptomatic gallstones
- Prevalence and incidence should be advised to have elective laparoscopic
most common problems affecting the digestive cholecystectomy
tract
Women are 3x more likely to develop gallstones K. ACUTE CHOLECYSTITIS
than men - Pathogenesis:
risk factors: Obesity, pregnancy, dietary factors, Acute cholecystitis is secondary to gallstones in 90
Crohn's disease, terminal ileal resection, gastric to 95% of cases
surgery, hereditary spherocytosis, sickle cell In <1% of acute cholecystitis, the cause is a tumor
disease, and thalassemia obstructing the cystic duct (leads to gallbladder
- Natural history distention, inflammation, and edema of the
Most patients will remain asymptomatic gallbladder wall)
prophylactic cholecystectomy in asymptomatic Gross appearance: gallbladder wall is grossly
persons with gallstones is rarely indicated thickened & reddish with subserosal hemorrhages;
cholecystectomy is advisable for the ff (+) pericholecystic fluid often; mucosal hyperemia
asymptomatic patients: & patchy necrosis
1. elderly patients with diabetes When the gallbladder remains obstructed and
2. individuals isolated from medical care for secondary bacterial infection supervenes an
extended periods of time
3. in populations with increased risk of acute gangrenous cholecystitis develops
gallbladder cancer (porcelain gallbladder abscess or empyema forms within the gallbladder;
premalignant lesion) can also lead to perforation of ischemic areas
4. symptomatic Cholesterolosis: emphysematous gallbladder : (+) gas may be seen
accumulation of cholesterol in macrophages in the gallbladder lumen and in the wall of the
in the gallbladder mucosa, either locally or gallbladder on abdominal radiographs and CT
as polyps; produces the classic macroscopic scans due to gas-forming organisms as part of the
appearance of a "strawberry gallbladder." secondary bacterial infection
5. symptomatic Adenomyomatosis or - clinical manifestations:
cholecystitis glandularis proliferans: unremitting epigastric or RUQ pain, may persist for
characterized on microscopy by several days, may radiate to the right upper part of
hypertrophic smooth muscle bundles and the back or the interscapular area; febrile,
anorexia, nausea, and vomiting, reluctant to move,
TOPNOTCH MEDICAL BOARD PREP SURGERY SUPPLEMENT HANDOUT Page 83 of 85
For inquiries visit www.topnotchboardprep.com.ph or email us at topnotchmedicalboardprep@gmail.com
TOPNOTCH MEDICAL BOARD PREP SURGERY SUPPLEMENT HANDOUT - Jules Lopez,MD-MBA,Teddy
Carpio,MD-MBA
For inquiries visit www.topnotchboardprep.com.ph or email us at topnotchmedicalboardprep@gmail.com
(+) focal tenderness and guarding on the RUQ, (+) UTZ: (+)gallbladder stones, dilated ducts
Murphy's sign (an inspiratory arrest with deep ERC: Definitive diagnosis
palpation in the right subcostal area) is - Treatment
characteristic IV antibiotics: initial management; cover for
mirizzis syndrome: Severe jaundice due to gram (-)
common bile duct stones or obstruction of the bile Fluid resuscitation: initial management
ducts by severe pericholecystic inflammation Emergency biliary decompression: if failed to
secondary to impaction of a stone in the improve with IV antibiotics and resuscitation
infundibulum of the gallbladder that mechanically measures
obstructs the bile duct
in elderly patients and in those with diabetes N. BILIARY PANCREATITIS
mellitus, acute cholecystitis may have a subtle - Obstruction of the pancreatic duct by an impacted stone
presentation resulting in a delay in diagnosis. or temporary obstruction by a stone passing through
- Laboratory diagnosis: the ampulla leads to this condition
A mild to moderate leukocytosis (12,000 to 15,000 - Diagnosis: UTZ of biliary tree
cells/mm3) - Treatment: ERC with sphincterotomy and stone
if high WBC (above 20,000): suggests a extraction + cholecystectomy (upon resolution of
complicated form of cholecystitis such as pancreatitis during same admission)
gangrenous cholecystitis, perforation, or
associated cholangitis. O. ACALCULOUS CHOLECYSTITIS
mild elevation of serum bilirubin, <4 mg/mL - develops in critically ill patients in ICU (patients on
mild elevation of alkaline phosphatase, parenteral nutrition with extensive burns, sepsis, major
transaminases, and amylase. operations, multiple trauma, or prolonged illness with
- diagnosis: multiple organ system failure)
UTZ: most useful radiologic test for diagnosing - histopathology: reveals edema of the serosa and
acute cholecystitis muscular layers, with patchy thrombosis of arterioles
Is 95% sensitive and specific and venules
Appears as thickening of the gallbladder wall - clinical manifestations:
and (+) pericholecystic fluid alert patient: right upper quadrant pain and
(+) sonographic murphys sign tenderness, fever, and leukocytosis
Biliary radionuclide scanning (HIDA scan): sedated or unconscious patient: fever and elevated
most accurate in the diagnosis of acute WBC count, as well as elevation of alkaline
cholecystitis phosphatase and bilirubin
- Treatment - diagnosis:
IV fluids UTZ: diagnostic test of choice; appears as distended
Antibiotics: should cover Gram (-) aerobes + gallbladder with thickened wall, biliary sludge,
anaerobes - 3rd generation cephalosporin or 2nd pericholecystic fluid, and (+) abscess formation
generation cephalosporin + metronidazole - Treatment of choice: Percutaneous ultrasound- or CT-
Analgesia guided cholecystostomy
Cholecystectomy: definitive treatment
Laparoscopic cholecystectomy: procedure of choice P. BILIARY or CHOLEDOCHAL CYSTS
- congenital cystic dilatations of the extrahepatic and/or
L. CHOLEDOCHOLITHIASES intrahepatic biliary tree
- Common bile duct stones - rare
- Common over the age of 60 - more common in women
- clinical manifestations: may be silent or incidental; if - more frequently diagnosed during childhood
symptomatic, may cause pain, nausea and vomiting with - types:
mild epigastric or RUQ tenderness + mild icterus type I: cystic dilatation of the extrahepatic bile
- diagnosis: duct; most common type
of serum bilirubin, alkaline phosphatase, and type II: diverticulum of the CBD
transaminases type III: a choledochocele extending from the distal
UTZ: dilated common bile duct (>8 mm in duct into the duodenum
diameter) type IV: combined intrahepatic and extrahepatic cysts
Endoscopic cholangiography: gold standard for type V: cystic disease confined to intrahepatic ducts
diagnosing CBD stones; can be therapeutic as well
IOC can be done to evaluate CBD stones - clinical manifestations: jaundice or cholangitis (for
- Treatment: sphincterotomy and ductal clearance of the adults);less than of patients present with the classic
stones is appropriate, followed by a laparoscopic clinical triad of abdominal pain, jaundice, and a mass
cholecystectomy - diagnosis: Ultrasonography or CT scanning will confirm
the diagnosis, but endoscopic, transhepatic, or MRC is
M. CHOLANGITIS required to assess the biliary anatomy and to plan the
- Complication of choledochal stones appropriate surgical treatment
- Gallstones are the most common cause of obstruction in - treatment: complete cyst excision with roux-en-Y
cholangitis hepaticojejunostomy
- Normal: bile is sterile
- Causative organisms: E. coli, Klebsiella pneumoniae, Q.
SCLEROSING CHOLANGITIS
Streptococcus faecalis, Enterobacter, and -
is a progressive disease that eventually results in
Bacteroidesfragilis secondary biliary cirrhosis characterized by
- Clinical manifestations : inflammatory strictures involving the intrahepatic and
most common presentation is fever, epigastric or extrahepatic biliary tree
right upper quadrant pain, and jaundice - associated with ulcerative colitis, Riedel's thyroiditis
(Charcot's triad) and retroperitoneal fibrosis
charcots triad + septic shock + mental status - increased risk for developing cholangiocarcinoma.
- mean age of presentation is 30 to 45 years
changes reynauds pentad
- men are affected twice as commonly as women
- diagnosis:
- clinical manifestations: jaundice, fatigue, weight loss,
Leukocytosis, hyperbilirubinemia, and elevation of
pruritus, and abdominal pain; usually with cyclic
alkaline phosphatase and transaminases are seen
remissions and excacerbations
TOPNOTCH MEDICAL BOARD PREP SURGERY SUPPLEMENT HANDOUT Page 84 of 85
For inquiries visit www.topnotchboardprep.com.ph or email us at topnotchmedicalboardprep@gmail.com
TOPNOTCH MEDICAL BOARD PREP SURGERY SUPPLEMENT HANDOUT - Jules Lopez,MD-MBA,Teddy
Carpio,MD-MBA
For inquiries visit www.topnotchboardprep.com.ph or email us at topnotchmedicalboardprep@gmail.com
- diagnosis:
elevated ALP & bilirubin
ERCP: confirmatory test
multiple dilatations and strictures
(beading) of both the intra- and
extrahepatic biliary tree

REVIEW QUESTIONS

1. A patient presents with biliary colic. On


ultrasound there are multiple small gallstones in
the gallbladder and the common bile duct
measures 9mm in diameter. No stone is
visualized in the common bile duct. Which of the
following is the most reasonable next step?

a. Repeat UTZ in 24-48 hours


b. MRCP with contrast
c. Percutaneous cholangiography
d. Laparoscopic cholecystectomy and
intraoperative cholangiography

Answer: D
For patients with symptomatic gallstones and
suspected CBD stones, either preoperative
endoscopic cholangiography or an intraoperative
cholangiogram will document the bile duct stones. If
an endoscopic cholangiogram reveals stones,
sphincterotomy and ductal clearance of the stones is
appropriate, followed by a laparoscopic
cholecystectomy. An intraoperative cholangiogram at
the time of cholecystectomy will also document the
presence or absence of bile duct stones. Laparoscopic
common bile duct exploration via the cystic duct or
with formal choledochotomy allows the stones to be
retrieved in the same setting. If the expertise and/or
the instrumentation for laparoscopic common bile
duct exploration are not available, a drain shuld be
left adjacent to the cystic duct and the patient
scheduled for endoscopic sphincterotomy the
following day. An open common bile duct exploration
is an option of the endoscopic method has already
been tried or is, for some reason, not feasible.

2. Which hepatic cells provides the primary


defense against lipopolysaccharide (LPS)?

a. Hepatocytes
b. Kuppfer cells
c. Bile duct epithelial cells
d. Intrahepatic endothelial cells

Answer: B
The complications of Gram negative sepsis is
initiated by the endotoxin LPS. The liver is the
main organ in the clearance of LPS in the
bloodstream and plays a critical role in the
identification and processing of LPS. Kuppfer
cells are the resident macrophages in the liver
and have been shown to participate in LPS
clearance.

TOPNOTCH MEDICAL BOARD PREP SURGERY SUPPLEMENT HANDOUT Page 85 of 85


For inquiries visit www.topnotchboardprep.com.ph or email us at topnotchmedicalboardprep@gmail.com

Вам также может понравиться